P2 Exam Practice Kit

July 16, 2017 | Author: immaculate79 | Category: Overdraft, Labour Economics, Banks, Demand, Cost
Share Embed Donate


Short Description

Practice kit...

Description

CIMA - PAPER P2

Performance Management Exam Practice Kit

Tutor contact details Tufal Choudhury [email protected] 07790 904122

1|Page

Paper P2 PERFORMANCE STRATEGY Syllabus overview While Paper P2 continues the analytic theme of Paper P1 Performance Operations (for example in terms of identifying relevant costs), its main focus is on the application of information in the management processes of decision-making and control, so as to optimise performance. The first two sections deal respectively with the key contributors to operational performance – revenue (decisions of what to produce, at what price) and costs (how to manage them to maximise profitability). The role of control in monitoring and improving performance then comes to the fore in the final two sections, dealing with principles and practices in the use of responsibility centres and budgeting. Syllabus structure The syllabus comprises the following topics and study weightings: A B C D

Pricing and Product Decisions Cost Planning and Analysis for Competitive Advantage Budgeting and management Control Control and Performance Measurement of Responsibility Centres

30% 30% 20% 20%

Assessment strategy There will be a written examination paper of three hours, plus 20 minutes of preexamination question paper reading time. During the 20 minutes you can: read the question paper and annotate or highlight the question paper. However you will not be allowed to: open the answer book; write in the answer book; add any loose sheets/supplements to your answer book; or use calculators. Failure to comply with these rules will be considered as a serious breach of the exam regulations.

2|Page

The examination paper will have the following sections.

Section A

50 marks

Five compulsory medium answer questions, each worth ten marks. Short scenarios may be given, to which some or all questions relate.

Section B

50 marks

One or two compulsory questions. Short scenarios may be given, to which questions relate.

Recommended study hours Based on a study duration of 3-4 months, we recommend; 1 2 3

4 study sessions per week. Each study session 1.5 to 2 hours. Total hours per week between 6 to 8 hours.

The study time per week recommended, should be 80% question practice.

Exam strategy Within your 20 minutes of official reading time allowed before the exam starts, read through all requirements and information carefully. You can write on the exam paper during this time so concentrate heavy on answer plans for the section A type questions, look at Section B type questions only if you have time. You can write on the exam paper during this time so produce brief outline answer plans and highlight key requirements and scenario information.   

You have 1.8 mins per mark (3 hours or 180 mins divided by 100 marks). Allow 18 minutes per question for the five compulsory questions within section A (total time 90 minutes). Allow 90 minutes (the remaining time you have) to complete section B.

The examiner has said that it is unlikely section B will ever contain one compulsory question and therefore more likely to contain two compulsory questions worth 25 marks each.

3|Page

The section B questions are not just 45 minutes of time per question attempted; you have to break down the 45 minutes of time allotted for each requirement before you begin. This ensures you don’t run out of time and this same principle should also be applied to section A requirements. Tackle all the easiest parts for numerical calculations and then attempt all the other aspects of the requirement in the time you have. It is crucial that you manage your time properly in the exam. Don’t be afraid to stop calculating even though your answer is not complete, otherwise over running will seriously prejudice other marks due to inadequate time and dramatically reduce your chances of passing the exam. Question requirements     

Read them carefully. Break down each requirement into headings and include and underline these headings within your answer sheet. Examine any information in the scenario for the question (if provided) constantly to generate ideas to meet the requirements. Ensure you deal with all aspects to a question requirement. Always write a brief conclusion (sometimes additional marks are included in the post exam guide if a candidate has does this).

Answer presentation       

Page clearly labelled as to what part of the question you are completing Start a fresh page for every part to a question. Watch for ‘write a report or memo etc’. Let the question requirement drive your headings. Write in brief explanations 5-6 lines then a gap or ‘white space’ every 5-6 lines. Diagrams should be neat and spacious about 1/3 page in size. Marks will be balanced according to the weighting of the syllabus.

Discussions    

Make the relevant point. Explain that point in as much detail as possible. Relate to the scenario. Give your recommendation where required.

4|Page

Learning objectives Learning objectives are defined at the back of every CIMA exam on exam day, a copy of the table below will be provided on exam day. Exam requirements will BOLD these verbs below to help you decide the correct approach to a question. The common learning verbs include Explain, Discuss, Describe, Identify and Construct. Familiarise yourself with these commonly used verbs.

Knowledge

Comprehension

Application

Analysis

Evaluation

List State Define

Make a list of Express, fully or clearly, the details/facts of Give the exact meaning of

Describe Distinguish Explain Identify Illustrate

Communicate the key features of Highlight the differences between Make clear, state the meaning of Recognise, or select after consideration Use an example to describe or explain

Apply Calculate/compute Demonstrate Prepare Reconcile Solve Tabulate

Put to practical use Ascertain mathematically Prove with certainty by practical means Make or get ready for use Prove consistent/compatible Find an answer to Arrange in a table

Analyse Categorise Compare and contrast Construct Discuss Interpret Produce

Examine in detail Place in a defined class Show the similarities and/or differences between Build up or compile Examine in detail by argument Translate into familiar terms Create or bring into existence

Advise Evaluate Recommend

Counsel, inform or notify Appraise or assess the value of Advise on the course of action

5|Page

CONTENTS

Section A type questions – 10 marks each Part A Pricing and Product Decisions Relevant chapters from study manual 1, 3, 5 and 8 Question number A1 - 1 A1 - 2 A1 – 3 A1 – 4 A1 - 5 A1 – 6 A1 – 7 A1 – 8

Title

EXE (P2) MNP (P2) VBJ (P2) QXY plc (P2) RST (P2) HS (P2) Bank charges (P2) WX

Exam year

Pilot Paper May 2005 May 2005 Nov 2006 May 2007 Nov 2007 May 2008 May 2011

Question page number 13 14 15 19 19 20 21 22

Answer page number 103 104 105 106 108 109 112 113

Question page number 24 25 27 29 31 33 35 36 37 39 41

Answer page number 117 121 124 127 131 137 139 144 146 151 154

Section B type questions – 25 marks each Part A Pricing and Product Decisions Relevant chapters from study manual 1, 3, 5 and 8 Question number A2 – 1 A2 – 2 A2 – 3 A2 – 4 A2 – 5 A2 – 6 A2 - 7 A2 – 8 A2 – 9 A2 – 10 A2 – 11

Title

TQ (P2) QP plc (P2) ZP plc (P2) AVX plc (P2) GHK (P2) H (P2) DFG (P2) Highly skilled workers (P2) RT LM Hotel

Exam year

Pilot Paper Nov 2005 Nov 2005 May 2006 May 2006 May 2007 Nov 2007 May 2008 May 2010 Nov 2010 May 2011

6|Page

Section A type questions – 10 marks each Part B Cost Planning and Analysis for Competitive Advantage Relevant chapters from study manual 2, 4 and 6 Question number B1 – 1 B1 – 2 B1 – 3 B1 – 4 B1 – 5 B1 – 6 B1 – 7 B1 – 8 B1 – 9 B1 – 10 B1 – 11 B1 – 12 B1 – 13 B1 – 14 B1 – 15 B1 - 16 B1 – 17 B1 – 18 B1 – 19 B1 – 20 B1 – 21 B1 – 22 B1 – 23 B1 - 24 B1 - 25 B1 – 26 B1 – 27 B1 – 28

Title

SWAL (P2) X group (P2) ML (P2) PK plc (P2) Financial advisors (P2) Compliance v conformance (P2) AVN (P2) W (P2) New product (P2) New small company (P2) XY (P2) Inventory levels (P2) Workshop (P2) Out-turn performace report PQ Timber products LMN Production manager CAL QW Accountancy services PT TQM and JIT (P1) Standard costing (P1) Marginal v throughput (P1) MRPS (P1) JIT (P1) Key features of TQM (P1)

Exam year

Pilot Paper May 2005 Nov 2005 Nov 2005 May 2006 May 2006 Nov 2006 Nov 2006 May 2007 Nov 2007 May 2008 May 2008 May 2008 May 2010 May 2010 May 2010 May 2010 Nov 2010 Nov 2010 Nov 2010 Nov 2010 May 2011 Pilot Paper May 2006 May 2006 May 2007 May 2007 May 2008

Question page number 44 44 45 46 46 47 47 48 49 50 51 51 52 53 54 54 55 56 57 58 58 59 60 60 60 60 60 61

Answer page number 157 159 160 161 163 165 167 170 171 172 174 175 177 178 180 181 182 183 185 186 187 188 189 190 191 191 192 192

7|Page

Section B type questions – 25 marks each Part B Cost Planning and Analysis for Competitive Advantage Relevant chapters from study manual 2, 4 and 6 Question number B2 – 1 B2 – 2 B2 – 3 B2 – 4

Title

The Q organisation (P2) F plc (P1) KL (P2) Retail outlet (P2)

Exam year

May 2005 May 2005 Nov 2006 Nov 2007

Question page number 62 63 64 65

Answer page number 194 198 202 206

8|Page

Section A type questions – 10 marks each Part C Budgeting and management Control Relevant chapters from study manual 4 Question number C1 – 1 C1 - 2 C1 – 3 C1 – 4 C1 – 5 C1 – 6 C1 – 7 C1 – 8 C1 – 9 C1 – 10 C1 - 11 C1 – 12 C1 – 13 C1 – 14 C1 - 15 C1 – 16 C1 - 17 C1 - 18 C1 – 19 C1 – 20 C1 – 21 C1 – 22

Title

Solicitors firm DW JYT DVD SFG Feedback and feedforward (P1) Profit centre managers (P1) Balance scorecard (P1) Particiaption in budgets (P1) Beyond budgeting (P1) J Limited (P1) ST plc (P1) W Limited (P1) T plc (P1) Product M (P1) QBD (P1) Budgetary planning and control (P1) JIT systems (P1) Feedback and forward (P1) Nursing homes (P1) Participative budgeting (P1) Rolling budgets (P1)

Exam year

May 2010 Nov 2010 May 2011 May 2011 May 2011 Pilot Paper Pilot Paper May 2005 May 2005 May 2005 Nov 2005 Nov 2005 Nov 2005 May 2006 May 2007 Nov 2007 Nov 2007 Nov 2007 Nov 2007 Nov 2007 May 2008 May 2008

Question page number 69 70 70 71 72 72 72 73 73 73 74 74 74 75 75 76 77 77 77 77 78 78

Answer page number 210 211 212 213 215 216 217 217 219 220 221 222 223 224 225 226 227 227 228 228 230 231

9|Page

Section B type questions – 25 marks each Part C Budgeting and management Control Relevant chapters from study manual 4 Question number C2 – 1 C2 – 2 C2 – 3 C2 – 4

Title

M plc (P1) RF Ltd (P1) Trackit (P1) X plc (P1)

Exam year

May 2006 May 2007 May 2008 Nov 2006

Question page number 79 80 82 85

Answer page number 232 236 240 244

Section A type questions – 10 marks each Part D Control and Performance Measurement of Responsibility Centres Relevant chapters from study manual 9 Question number D1 – 1 D1 – 2 D1 – 3 D1 – 4 D1 – 5 D1 – 6 D1 – 7

Title

EVA and RI (P1) Controllability principle (P1) Transfer pricing (P1) EVA (P1) WD, PD & TD (P1) G group (P1) Digital equipment (P1)

Exam year

Pilot Paper Pilot Paper Pilot Paper May 2005 May 2005 May 2007 May 2008

Question page number 87 87 87 87 87 88 89

Answer page number 248 249 249 251 252 254 255

10 | P a g e

Section B type questions – 25 marks each Part D Control and Performance Measurement of Responsibility Centres Relevant chapters from study manual 9 Question number D2 – 1 D2 – 2 D2 – 3 D2 – 4 D2 – 5 D2 – 6 D2 – 7

Title

Y and Z (P1) FP (P1) ZZ group (P1) Computer manufacturer (P1) Perfumes and cosmetics SWZ DE company

Exam year

Nov 2005 May 2006 Nov 2006 Nov 2007 May 2010 Nov 2010 May 2011

Question page number 90 91 93 95 96 99 100

Answer page number 257 262 266 269 273 276 279

11 | P a g e

Mock exams to be submitted Once all chapters have been completed, four mock exams are to be attempted under timed conditions and sent to the address Tufal Choudhury, 57 Toms Lane, Kings Langley WD4 8NA. Please ensure you put your name and address on your mock exam when submitting by post. Alternatively scanned scripts by e-mail can be sent directly to [email protected] You should allow 20 minutes reading time before you begin and then 3 hours maximum to complete each mock exam. Make sure you have a strategy and plan for how to manage your time.

Mock One P2 Specimen (2010) exam paper

Mock Two P2 September 2010 exam paper

Mock Three P2 March 2011 exam paper

Mock Four P2 September 2011 exam paper

Mock Five P2 November 2011 exam paper The above exam questions and solutions can be downloaded via your ‘mycima’ account which contains access to all past exam papers and solutions, alternatively contact [email protected] if you would like to receive PDF copies.

12 | P a g e

Questions – Section A Part A Pricing and Product Decisions A1 -1 EXE (CIMA P2 Pilot paper 2005) You have received a request from EXE to provide a quotation for the manufacture of a specialised piece of equipment. This would be a one-off order, in excess of normal budgeted production. The following cost estimate has already been prepared: Note

$

1 2

50 20

Skilled 25 hours @ $8.00 per hour Semi-skilled 10 hours @ $5.00 per hour

3 4

200 50

35 hours @ $10.00 per hour

5

350

Estimating time

6

Administration overhead @ 20% of production cost

7

100 770 154 924

Profit @ 25% of total cost

8

Direct materials: Steel 10m2 @ $5.00 per m2 Brass fittings Direct labour:

Overhead

Selling price

231 1,155

Notes: 1. 1 The steel is regularly used, and has a current stock value of $5.00 per square metre. There are currently 100 square metres in stock. The steel is readily available at a price of $5.50 per square metre. 2. The brass fittings would have to be bought specifically for this job: a supplier has quoted the price of $20 for the fittings required. 3. The skilled labour is currently employed by your company and paid at a rate of $8.00 per hour. If this job were undertaken it would be necessary either to work 25 hours’ overtime, which would be paid at time plus one half, OR in order to carry out the work in normal time, reduce production of another product that earns contribution of $13.00 per hour. 4. The semi-skilled labour currently has sufficient paid idle time to be able to complete this work. 13 | P a g e

5. The overhead absorption rate includes power costs which are directly related to machine usage. If this job were undertaken, it is estimated that the machine time required would be ten hours. The machines incur power costs of $0.75 per hour. There are no other overhead costs that can be specifically identified with this job. 6. The cost of the estimating time is that attributed to the four hours taken by the engineers to analyse the drawings and determine the cost estimate given above. 7. It is company policy to add 20% to the production cost as an allowance for administration costs associated with the jobs accepted. This is the standard profit added by your company as part of its pricing policy. Required: Prepare on a relevant cost basis, the lowest cost estimate that could be used as the basis for a quotation. Explain briefly your reasons for using EACH of the values in your estimate. (12 marks) A1 – 2 MNP (CIMA P2 May 2005) Z manufactures three joint products (M, N and P) from the same common process. The following process account relates to the common process last month and is typical of the monthly results of operating this process: Common Process Account Litres $ Litres $ Opening work in process Materials Conversion costs: Variable Fixed Abnormal loss

1,000 5,320 100,000 250,000 100,000 180,000 5,200 101,000 535,320

Normal loss Output M Output N Output P Closing WIP

10,000 25,000 15,000 45,000 800 101,000

20,000 141,875 85,125 255,375 3,533 29,412 535,320

Each one of the products can be sold immediately after the common process, but each one of them can be further processed individually before being sold. The following further processing costs and selling prices per litre are expected:

14 | P a g e

Product

M N P

Selling price after common process $/litre 6·25 5·20 6·80

Selling price after further processing $/litre 8·40 6·45 7·45

Further variable processing cost $/litre 1·75 0·95 0·85

Required: (a) State the method used to apportion the common costs between the products M, N and P and comment on its acceptability. Explain why it is necessary to apportion the common costs between each of the products. (5 marks) (b) Evaluate the viability of the common process, and determine the optimal processing plan for each of the three products showing appropriate calculations. (5 marks) (Total = 10 marks) A1 – 3 VBJ (CIMA P2 May 2005) The CS group is planning its annual marketing conference for its sales executives and has approached the VBJ Holiday company (VBJ) to obtain a quotation. VBJ has been trying to win the business of the CS group for some time and is keen to provide a quotation which the CS group will find acceptable in the hope that this will lead to future contracts. The manager of VBJ has produced the following cost estimate for the conference: $ Coach running costs 2,000 Driver costs 3,000 Hotel costs 5,000 General overheads 2,000 Sub total 12,000 Profit (30%) Total

3,600 15,600

You have considered this cost estimate but you believe that it would be more appropriate to base the quotation on relevant costs. You have therefore obtained the following further information:

15 | P a g e

Coach running costs represent the fuel costs of $1,500 plus an apportionment of the annual fixed costs of operating the coach. No specific fixed costs would be incurred if the coach is used on this contract. If the contract did not go ahead, the coach would not be in use for eight out of the ten days of the conference. For the other two days a contract has already been accepted which contains a significant financial penalty clause. This contract earns a contribution of $250 per day. A replacement coach could be hired for $180 per day. Driver costs represent the salary and related employment costs of one driver for 10 days. If the driver is used on this contract the company will need to replace the driver so that VBJ can complete its existing work. The replacement driver would be hired from a recruitment agency that charges $400 per day for a suitably qualified driver. Hotel costs are the expected costs of hiring the hotel for the conference. General overheads are based upon the overhead absorption rate of VBJ and are set annually when the company prepares its budgets. The only general overhead cost that can be specifically identified with the conference is the time that has been spent in considering the costs of the conference and preparing the quotation. This amounted to $250. Required: Prepare a statement showing the total relevant cost of the contract. Explain clearly the reasons for each of the values in your quotation and for excluding any of the costs (if appropriate). (10 marks)

16 | P a g e

A1 – 4 QXY plc (CIMA P2 Nov 2006) You are the Assistant Management Accountant of QXY plc, a food manufacturer. The Board of Directors is concerned that its operational managers may not be fully aware of the importance of understanding the costs incurred by the business and the effect that this has on their operational decision making. In addition, the operational managers need to be aware of the implications of their pricing policy when trying to increase the volume of sales. You are scheduled to make a presentation to the operational managers tomorrow to explain to them the different costs that are incurred by the business, the results of some research that has been conducted into the implications for pricing and the importance of understanding these issues for their decision making. The diagram on the opposite page has already been prepared for the presentation. Required: You are required to interpret the diagram and explain how it illustrates issues that the operational managers should consider when making decisions. (Note: your answer mustinclude explanations of the Sales Revenue, Total Cost and Fixed Cost lines, and the significance of each of the activity levels labelled A, B, C, D). (10 marks)

17 | P a g e

Diagram showing costs and revenues over a range of activity levels

A1 – 5 Z (CIMA P2 May 2007) 18 | P a g e

A1 – 5 RST (CIMA P2 May 2007) Z is one of a number of companies that produce three products for an external market. The three products, R, S and T may be bought or sold in this market. The common process account of Z for March 2007 is shown below: Kg Inputs: Material A Material B Material C Direct labour Variable overhead Fixed cost Totals

1,000 2,000 1,500

4,500

$ 3,500 2,000 3,000 6,000 2,000 1,000 17,500

Kg Normal loss Outputs: Product R Product S Product T

$

500

0

800 2,000 1,200

3,500 8,750 5,250

4,500 17,500

Z can sell products R, S or T after this common process or they can be individually further processed and sold as RZ, SZ and TZ respectively. The market prices for the products at the intermediate stage and after further processing are: Market prices per kg:

R S T RZ SZ TZ

$ 3.00 5.00 3.50 6.00 5.75 6.75

The specific costs of the three individual further processes are: Process R to RZ variable cost of $1.40 per kg, no fixed costs Process S to SZ variable cost of $0.90 per kg, no fixed costs Process T to TZ variable cost of $1.00 per kg, fixed cost of $600 per month Required: (a) Produce calculations to determine whether any of the intermediate products should be further processed before being sold. Clearly state your recommendations together with any relevant assumptions that you have made. (3 marks)

19 | P a g e

(b) Produce calculations to assess the viability of the common process: (i) assuming that there is an external market for products R, S and T; and (ii) assuming that there is not an external market for products R, S and T. State clearly your recommendations. (7 marks) (Total = 10 marks) A1 – 6 HS (CIMA P2 Nov 2007) HS manufactures components for use in computers. The business operates in a highly competitive market where there are a large number of manufacturers of similar components. HS is considering its pricing strategy for the next twelve weeks for one of its components. The Managing Director seeks your advice to determine the selling price that will maximise the profit to be made during this period. You have been given the following data: Market Demand The current selling price of the component is $1,350 and at this price the average weekly demand over the last four weeks has been 8,000 components. An analysis of the market shows that for every $50 increase in selling price the demand reduces by 1,000 components per week. Equally, for every $50 reduction in selling price the demand increases by 1,000 components per week. Costs The direct material cost of each component is $270. This price is part of a fixed price contract with the material suppliers and the contract does not expire for another year. Production labour and conversion costs, together with other overhead costs and the corresponding output volumes, have been collected for the last four weeks and they are as follows: Week 1 2 3 4

Output volume (units) 9,400 7,600 8,500 7,300

$000 7,000 5,688 6,334 5,446

No significant changes in cost behaviour are expected over the next twelve weeks.

20 | P a g e

Required: (a) Calculate the optimum (profit maximising) selling price of the component for the period. Note: If Price = a - bq then Marginal Revenue = a - 2bq (6 marks) (b) Identify and explain two reasons why it may be inappropriate for HS to use this theoretical pricing model in practice. (4 marks) (Total = 10 marks) A1 – 7 Bank charges (CIMA P2 May 2008) A bank is reviewing the bank account it offers to its business customers and the charges it makes for routine transactions (for example paying into the account, writing cheques, making electronic payments and transfers). Currently, the bank’s charges to its business customers are £0⋅60 per routine transaction. The bank pays interest to the customer at 0⋅1% per year on any balance in the account. According to the bank’s records, there are currently one million business customers. Each customer makes one thousand routine transactions each year; 45% of business customers maintain an average balance of £2,000 in their account. The accounts of the other 55% of business customers are overdrawn with an average overdraft balance of £4,000. Interest on overdrawn accounts is charged at 20% per year. In addition, the bank has a number of savings account customers which, together with the bank’s business customers, result in a balance of net funds that are invested by the bank and yield an annual return by 3% per year. The bank is concerned about a growing tendency for its competitors to provide routine transactions free of charge to their business customers. As a result the bank is considering twoaccount options: Account Option One An account that charges the business customer a fixed fee of £10 per month, with no further charges for any routine transactions. Interest would be paid to the business customer at 0⋅5% per year on any balances in the account. The bank expects that if it adopts this charging structure, it will increase the number of business customers by 5%from its present level;

21 | P a g e

Account Option Two An account that does not charge the customer for any routine transactions, but pays no interest on any balances in the account. The bank expects that if it adopts this charging structure, this will increase the number of business customers by 10% from its present level. The bank does not expect the profile of new business customers to be different from existing business customers in terms of the balances in their accounts or the number of routine transactions they make. Interest will continue to be charged at 20% per year on overdrawn accounts. The bank does not expect that either of these options will result in any changes to its existing staffing or other resources. The bank also expects that if it takes no action and continues with its existing bank account that the number of business customers will fall by 20%. Required: (a) Recommend which course of action the bank should take by preparing calculations to show the annual profits from: (i) continuing with the existing bank account (ii) each of the two account options described above. (12 marks) A1 – 8 WX (CIMA P2 May 2011) WX is reviewing the selling price of one of its products. The current selling price of the product is $25 per unit and annual demand is forecast to be 150,000 units at this price. Market research indicates that the level of demand would be affected by any change in the selling price. Detailed analysis from this research shows that for every $1 increase in selling price, annual demand would reduce by 25,000 units and that for every $1 decrease in selling price, annual demand would increase by 25,000 units. A forecast of the annual costs that would be incurred by WX in respect of this product at differing activity levels is as follows: Annual production (units)

Direct materials Direct labour Overhead

100,000

160,000

200,000

$000 200 600 880

$000 320 960 1,228

$000 400 1,200 1,460

22 | P a g e

The cost behaviour patterns represented in the above forecast will apply for the whole range of output up to 300,000 units per annum of this product. Required: (a) (i) Calculate the total variable cost per unit. (2 marks) (ii) Calculate the selling price of the product that will maximise the company’s profits. (4 marks) Note: If Price (P) = a - bx then Marginal Revenue = a - 2bx (b) Explain TWO reasons why the company might decide NOT to use this optimum selling price. (4 marks) (Total = 10 marks)

23 | P a g e

Questions – Section B Part A Pricing and Product Decisions A2-1 TQ (CIMA P2 Pilot Paper 2005) (a) TQ manufactures and retails second generation mobile (cell) phones. The following details relate to one model of phone:

Budgeted selling price Budgeted variable cost Budgeted fixed cost

$/unit 60 25 10

Period Budgeted production and sales (units)

1 520

2 590

3 660

Fixed overhead volume variance

$1,200 (A)

$1,900 (A)

$2,600 (A)

There was no change in the level of stock during any of periods 1 to 3. The Board of Directors had expected sales to keep on growing but, instead, they appeared to have stabilised. This has led to the adverse fixed overhead volume variances. It is now the start of period 4 and the Board of Directors is concerned at the large variances that have occurred during the first three periods of the year. The Sales and Marketing Director has confirmed that the past trend of sales is likely to continue unless changes are made to the selling price of the product. Further analysis of the market for the mobile phone suggests that demand would be zero if the selling price was raised to $100 or more. Required: (i) Calculate the price that TQ should have charged for the phone assuming that it wished to maximise the contribution from this product. Note: If price = a – bx then marginal revenue = a – 2bx (7 marks) (ii) Calculate the difference between the contribution that would have been earned at the optimal price and the actual contribution earned during period 3, assuming the variable costs per unit were as budgeted. (3 marks)

24 | P a g e

(b) TQ is currently developing a third generation mobile phone. It is a “state of the art” new handheld device that acts as a mobile phone, personal assistant, digital camera (pictures and video), and music player. The Board of Directors seeks your advice as to the pricing strategy that it should adopt for such a product. The company has incurred a significant level of development costs and recognises that the technology for these products is advancing rapidly and that the life cycle for the product is relatively short. Required: Prepare a report, addressed to the Board of Directors that discusses the alternative pricing strategies available to TQ. (15 marks) (Total = 25 marks) A2 – 2 QP plc (CIMA P2 Nov 2005) QP plc is a food processing company that produces pre-prepared meals for sale to consumers through a number of different supermarkets. The company specialises in three particular pre-prepared meals and has invested significantly in modern manufacturing processes to ensure a high quality product. The company is very aware of the importance of training and retaining high quality staff in all areas of the company and, in order to ensure their production employees’ commitment to the company, the employees are guaranteed a weekly salary that is equivalent to their normal working hours paid at their normal hourly rate of £7 per hour. The meals are produced in batches of 100 units. Costs and selling prices per batch are as follows: Meal

TR £/batch

PN £/batch

BE £/batch

Selling Price

340

450

270

Ingredient K (£5/kg) Ingredient L (£10/kg) Ingredient M (£15/kg)

150 70 30

120 90 75

90 40 45

Labour (£7/hour)

21

28

42

Factory costs absorbed

20

80

40

QP plc has adopted throughput accounting for its short-term decisions.

25 | P a g e

Required: (a) State the principles of throughput accounting and the effects of using it for short-term decision making. (6 marks) (b) QP plc is preparing its production plans for the next three months and has estimated the maximum demand from its customers to be as follows: TR 500 batches PN 400 batches BE 350 batches These demand maximums are amended figures because a customer has just delayed its request for a large order and QP has unusually got some spare capacity over the next three months. However, these demand maximums do include a contract for the delivery of 50 batches of each to an important customer. If this minimum contract is not satisfied then QP plc will have to pay a substantial financial penalty for non-delivery. The Production Director is concerned at hearing news that two of the ingredients used are expected to be in short supply for the next three months. QP plc does not hold inventory of these ingredients and although there are no supply problems for ingredient K, the supplies of ingredients L and M are expected to be limited to: Ingredient L 7,000 kilos Ingredient M 3,000 kilos The Production Director has researched the problem and found that ingredient V can be used as a direct substitute for ingredient M. It also costs the same as ingredient M. There is an unlimited supply of ingredient V. Required: Prepare calculations to determine the production mix that will maximise the profit of QP plc during the next three months. (10 marks) (c) The World Health Organisation has now announced that ingredient V contains dangerously high levels of a chemical that can cause life-threatening illnesses. As a consequence it can no longer be used in the production of food.

26 | P a g e

As a result, the production director has determined the optimal solution to the company’sproduction mix problem using linear programming. This is set out below: Objective function value TR value PN value BE value TR slack value PN slack value BE slack value L value M value

110,714 500 357 71 0 43 279 3 28

Required: Explain the meaning of each of the values contained in the above solution. (9 marks) (Total = 25 marks) A2 – 3 ZP plc (CIMA P2 Nov 2007) ZP plc is a marketing consultancy that provides marketing advice and support to small and medium sized enterprises. ZP plc employs 4 full time marketing consultants who each expect to deliver 1,500 chargeable hours per year and each receive a salary of £60,000 per year. In addition the company employs 6 marketing support/administration staff whose combined total salary cost is £120,000 per year. ZP plc has estimated its other costs for the coming year as follows: £000 Office premises: rent, rates, heating

50

Advertising

5

Travel to clients

15

Accommodation whilst visiting clients

11

Telephone, fax, communications

10

27 | P a g e

ZP plc has been attributing costs to each client (and to the projects undertaken for them) by recording the chargeable hours spent on each client and using a single cost rate of £75 per chargeable hour. The same basis has been used to estimate the costs of a project when preparing a quotation for new work. ZP plc has reviewed its existing client database and determined the following three average profiles of typical clients: Client profile Chargeable hours per client

D 100

E 700

F 300

Distance (miles) to client

50

70

100

Number of visits per client

3

8

3

Number of clients in each profile

10

5

5

The senior consultant has been reviewing the company’s costing and pricing procedures. He suggests that the use of a single cost rate should be abandoned and, where possible, activities should be costed individually. With this is mind he has obtained the following further information: 

It is ZP plc’s policy that where a visit is made to a client and the distance to the client is more than 50 miles, the consultant will travel the day before the visit and stay in local accommodation so that the maximum time is available for meeting the client the following day.



The cost of travel to the client is dependent on the number of miles travelled to visit the client.



Other costs are facility costs – at present the senior consultant cannot identify an alternative basis to that currently being used to attribute costs to each client.

Required: (a) Prepare calculations to show the cost attributed to each client group using an activity based system of attributing costs. (7 marks) (b) Discuss the differences between the costs attributed using activity based costing and those attributed by the current system and adviser whether the senior consultant’s suggestion should be adopted. (9 marks) 28 | P a g e

(c) In a manufacturing environment activity based costing often classifies activities into those that are: unit; batch; product sustaining; and facility sustaining. Discuss, giving examples, how similar classifications may be applied to the use of the technique in consultancy organisations such as ZP plc. (9 marks) (Total = 25 marks) A2 – 4 AVX plc (CIMA P2 May 2006) AVX Plc assembles circuit boards for use by high technology audio video companies. Due to the rapidly advancing technology in this field, AVX Plc is constantly being challenged to learn new techniques. AVX Plc uses standard costing to control its costs against targets set by senior managers. The standard labour cost per batch of one particular type of circuit board (CB45) is set out below: £ Direct labour 50 hours @ £10 /hour 500 The following labour efficiency variances arose during the first six months of the assembly of CB45: Month November December January February March April

Number of batches assembled and sold 1 1 2 4 8 16

Labour Efficiency Variance (£) Nil 170.00 Favourable 452.20 Favourable 1,089.30 Favourable 1,711.50 Favourable 3,423.00 Favourable

An investigation has confirmed that all of the costs were as expected except that there was a learning effect in respect of the direct labour that had not been anticipated when the standard cost was set. Required: (a) (i) Calculate the monthly rates of learning that applied during the six months; (ii) Identify when the learning period ended and briefly discuss the implications of your findings for AVX Plc. (10 marks)

29 | P a g e

AVX Plc initially priced each batch of CB45 circuit boards on the basis of its standard cost of £960 plus a mark up of 25%. Recently the company has noticed that, due to increasing competition, it is having difficulty maintaining its sales volume at this price. The Finance Director has agreed that the long run unit variable cost of the CB45 circuit board is £672.72 per batch. She has suggested that the price charged should be based on an analysis of market demand. She has discovered that at a price of £1,200 the demand is 16 batches per month, for every £20 reduction in selling price there is an increase in demand of 1 batch of CB45 circuit boards, and for every £20 increase in selling price there is a reduction in demand of 1 batch. Required: (b) Calculate the profit maximising selling price per batch using the data supplied by the Finance Director (8 marks) Note: If Price (P) = a-bx then Marginal Revenue (MR) = a-2bx The Technical Director cannot understand why there is a need to change the selling price. He argues that this is a highly advanced technological product and that AVX Plc should not reduce its price as this reflects badly on the company. If anything is at fault, he argues, it is the use of Standard Costing and he has asked whether Target Costing should be used instead. Required: (c) (i) Explain the difference between standard costs and target costs; (ii) Explain the possible reasons why AVX Plc needs to re-consider its pricing policy now that the CB45 circuit board has been available in the market for six months. (7 marks) (Total = 25 marks)

30 | P a g e

A2 – 5 GHK (CIMA P2 May 2006) GHK manufactures four products from different combinations of the same direct materials and direct labour. An extract from the flexible budgets for next quarter for each of these products is as follows : Product Units Revenue Direct Material A (note 1) Direct Material B (note 2) Direct labour (note 3) Overhead (note 4)

G 3,000 $000 30 9

5,000 $000 50 15

H 3,000 $000 60 12

5,000 $000 100 20

J 3,000 $000 45 4.5

5,000 $000 75 7.5

K 3,000 $000 90 18

5,000 $000 150 30

6

10

6

10

13.5

22.5

36

60

6

10

24

40

22.5

37.5

9

15

6

8

13

19

11

17

11

17

Notes 1. Material A was purchased some time ago at a cost of $5 per kg. There are 5,000 kgs in inventory. The costs shown in the flexible budget are based on this historical cost. The material is in regular use and currently has a replacement cost of $7 per kg. 2. Material B is purchased as required ; its expected cost is $10 per kg. The costs shown in the flexible budget are based on this expected cost. 3. Direct labour costs are based on an hourly rate of $10 per hour. Employees work the number of hours necessary to meet production requirements. 4. Overhead costs of each product include a specific fixed cost of $1,000 per quarter which would be avoided if the product was to be discontinued. Other fixed overhead costs are apportioned between the products but are not affected by the mix of products manufactured. GHK has been advised by the only supplier of material B that the quantity of material B that will be available during the next quarter will be limited to 5,000 kgs. Accordingly the company is being forced to reconsider its production plan for the next quarter. GHK has already entered into contracts to supply one of its major customers with the following : 500 units of product G 1,600 units of product H 800 units of product J 400 units of product K

31 | P a g e

Apart from this, the demand expected from other customers is expected to be 3,600 units of product G 3,000 units of product H 3,000 units of product J 4,000 units of product K The major customer will not accept partial delivery of the contract and if the contract with this major customer is not completed in full, then GHK will have to pay a financial penalty of $5,000. Required : (a) For each of the four products, calculate the relevant contribution per $ of material B for the next quarter. (6 marks) (b) It has been determined that the optimum production plan based on the data above is to produce 4,100 units of product G, 4600 units of product H, 800 units of product J, and 2,417 units of product K. Determine the amount of financial penalty at which GHK would be indifferent between meeting the contract or paying the penalty. (5 marks) (c) Calculate the relevant contribution to sales ratios for each of the four products. (2 marks) (d) Assuming that the limiting factor restrictions no longer apply, prepare a sketch of a multi product profit volume chart by ranking the products according to your contribution to sales ratio calculations based on total market demand. Your sketch should plot the products using the highest contribution to sales ratio first. (6 marks) (e) Explain briefly, stating any relevant assumptions and limitations, how the multi product profit volume chart that you prepared in (d) above may be used by the manager of GHK to understand the relationships between costs, volume and profit within the business. (6 marks) (Total = 25 marks)

32 | P a g e

A2 – 6 H (CIMA P2 May 2007) H, a printing company, uses traditional absorption costing to report its monthly profits. It is seeking to increase its business by winning work from new customers. It now has theopportunity to prepare a quotation for a large organisation that currently requires a new catalogue of its services. A technical report on the resource requirements for the catalogues has been completed at a cost of $1,000 and its details are summarised below: Production period It is expected that the total time required to print and despatch the catalogue will be one week. Material A 10,000 sheets of special printing paper will be required. This is a paper that is in regular use by H and the company has 3,400 sheets in inventory. These originally cost $1.40 per sheet but the current market price is $1•50 per sheet. The resale price of the sheets held in inventory is $1.20 per sheet. Material B This is a special ink that H will need to purchase at a cost of $8 per litre. 200 litres will be required for this catalogue but the supplier has a minimum order size of 250 litres. H does not foresee any other use for this ink, but will hold the surplus in inventory. H’s inventory policy is to review slow moving items regularly. The cost of any inventory item that has not been used for more than 6 months is accounted for as an expense of the period in which that review occurs. Direct labour Sufficient people are already employed by H to print the catalogue, but some of the printing will require overtime working due to the availability of a particular machine that is used on other work. The employees are normally paid $8 per hour, the order will require 150 hours of work and 50 of these hours will be in excess of the employees’ normal working week. A rate of $10 per hour is paid for these overtime hours. Employees are paid using an hourly rate with a guaranteed minimum wage for their normal working week. Supervision An existing supervisor will take responsibility for the catalogue in addition to her existing duties. She is not currently fully employed and receives a salary of $500 per week.

33 | P a g e

Machinery Two different types of machine will be required: Machine A will print the catalogues. This is expected to take 20 hours of machine time. The running cost of machine A is $5 per hour. There is currently 30 hours of unused time on machine A per week that is being sold to other printers for $12 per hour. Machine B will be used to cut and bind the catalogues. This machine is being used to full capacity in the normal working week and this is why there is a need to work overtime. The catalogue will require 25 machine hours and these have a running cost of $4 per hour. Despatch There will be a delivery cost of $400 to transport the catalogues to the customer. Fixed overhead costs H uses a traditional absorption costing system to attribute fixed overhead costs to its work. The absorption rate that it uses is $20 per direct labour hour. Profit mark-up H applies a 30% mark-up to its costs to determine its selling prices. Required: (a) In order to assist the management of H in preparing its quotation, prepare a schedule showing the relevant costs for the production of the catalogues. State clearly your reason for including or excluding each value that has been provided in the above scenario. (15 marks) (b) Explain how the use of relevant costs as the basis of setting a selling price may be appropriate for short-term pricing decisions but may be inappropriatefor long-term pricing decisions. Your answer should also discuss the conflict between reporting profitability within a traditional absorption costing system and the use of relevant cost based pricing. (10 marks) (Total = 25 marks)

34 | P a g e

A2 – 7 DFG (CIMA P2 Nov 2007) DFG manufactures two products from different combinations of the same resources. Unit selling prices and unit cost details for each product are as follows: Product Selling price

D £/unit 115

G £/unit 120

Direct material A (£5 per kg) Direct material B (£3 per kg) Skilled labour (£7 per hour) Variable overhead (£2 per machine hour) Fixed overhead*

20 12 28 14 28

10 24 21 18 36

Profit

13

11

*Fixed overhead is absorbed using an absorption rate per machine hour. It is an unavoidable central overhead cost that is not affected by the mix or volume of products produced. The maximum weekly demand for products D and G is 400 units and 450 units respectively and this is the normal weekly production volume achieved by DFG. However, for the next four weeks the achievable production level will be reduced due to a shortage of available resources. The resources that are expected to be available are as follows: Direct material A Direct material B Skilled labour Machine time

1,800kg 3,500kg 2,500 hours 6,500 machine hours

Required: (a) Using graphical linear programming identify the weekly production schedule for products D and G that maximises the profits of DFG during the next four weeks. (15 marks) (b) The optimal solution to part (a) shows that the shadow prices of Skilled labour and Direct material A are as follows: Skilled labour £Nil Direct material A £5.82 Explain the relevance of these values to the management of DFG. (6 marks) 35 | P a g e

(c) Using the graph you have drawn in part (a) explain how you would calculate by how much the selling price of Product D could rise before the optimal solution would change. Note: Assume that demand is not affected by the selling price. You are not required to perform any calculations. (4 marks) (Total = 25 marks) A2 – 8 Highly skilled workers (CIMA P2 May 2008) An engineering company manufactures a number of products and components, using a team of highly skilled workers and a variety of different metals. The current supplier has announced that the amount of M1, one of the materials it currently supplies, will be limited to 1,000 square metres in total for the next three-month period because there will be insufficient M1 to satisfy demand. The only items manufactured using M1 and their production costs and selling prices (where applicable) are shown below:

Selling price

Product P4 $/unit 125

Product P6 $/unit 175

Component C3 $/unit n/a

Component C5 $/unit n/a

Direct materials: M1 * M2 Direct labour Variable overhead Fixed overhead **

15 10 20 10 20

10 20 30 15 30

5 15 16 8 16

10 20 10 5 10

Total cost

75

105

60

55

* Material M1 is expected to be limited in supply during the next three months. These costs are based on M1 continuing to be available at a price of $20 per square metre. ** Fixed overhead is absorbed on the basis of direct labour cost. Products P4 and P6 are sold externally. Components C3 and C5 are used in other products made by the company. These other products do not require any further amounts of material M1.

36 | P a g e

The estimated total demand for these products and components during the next three months is as follows: P4 P6 C3 C5

2,000 units 1,500 units 500 units 1,000 units

Components C3 and C5 are essential components. They would have to be bought in if they could not be made internally. They can be purchased from external suppliers for $75 and $95 per unit respectively. The bought in components are of the same quality as those manufactured by the company. The products they are used in have sufficient margins to remain financially worthwhile if C3 and C5 are bought in at these prices. Required: (a) Prepare calculations to show the most profitable course of action for the company for the next three months, assuming that there are no other suppliers of material M1, and advise the company on THREE other factors that it should consider before making its decision. (14 marks) (b) Calculate the maximum prices that the company should pay to obtain further supplies of material M1 from an alternative supplier, and the quantities of material M1 to which each of these prices apply. (6 marks) The company has now become aware of a contract that it has already accepted, for the immediate delivery of 500 units of P4 at a selling price of $125 per unit. This contract has a financial penalty clause for non-delivery. This contract is in addition to the 2,000 units of estimated demand for P4 stated previously. Assume that there is no alternative supplier of material M1. (c) Calculate the minimum financial penalty that would change your recommendation. (5 marks) (Total = 25 marks) A2 – 9 RT (CIMA P2 May 2010) RT produces two products from different quantities of the same resources using a just-intime (JIT) production system. The selling price and resource requirements of each of the products are shown below:

37 | P a g e

Product

R

T

Unit selling price ($)

130

160

Resources per unit: Direct labour ($8 per hour)

3 hours

5 hours

Material A ($3 per kg)

5 kgs

4 kgs

Material B ($7 per litre)

2 litres

1 litre

Machine hours ($10 per hour)

3 hours

4 hours

Market research shows that the maximum demand for products R and T during June 2010 is 500 units and 800 units respectively. This does not include an order that RT has agreed with a commercial customer for the supply of 250 units of R and 350 units of T at selling prices of $100 and $135 per unit respectively. Although the customer will accept part of the order, failure by RT to deliver the order in full by the end of June will cause RT to incur a $10,000 financial penalty. At a recent meeting of the purchasing and production managers to discuss the production plans of RT for June, the following resource restrictions for June were identified: Direct labour hours Material A Material B Machine hours

7,500 hours 8,500 kgs 3,000 litres 7,500 hours

Required: (a) Assuming that RT completes the order with the commercial customer, prepare calculations to show, from a financial perspective, the optimum production plan for June 2010 and the contribution that would result from adopting this plan. (6 marks) (b) Prepare calculations to show, from a financial perspective, whether RT should complete the order from the commercial customer (3 marks) You have now presented your optimum production plan to the purchasing and production managers of RT. During your presentation it became clear that the predicted resource restrictions were rather optimistic. In fact the managers agreed that the availability of all of the resources could be as much as 10% lower than their original predictions.

38 | P a g e

(c) Assuming that RT completes the order with the commercial customer, andusing graphical linear programming, prepare a graph to show the optimum production plan for RT for June 2010 on the basis that the availability of all resources is 10% lower than originally predicted. (11 marks) (d) Discuss how the graph in your solution to (c) above can be used to help to determine the optimum production plan for June 2010 if the actual resource availability lies somewhere between the managers’ optimistic and pessimistic predictions. (5 marks) (Total = 25 marks) A2 – 10 LM (CIMA P2 Nov 2010) LM produces two products from different quantities of the same resources using a just-intime (JIT) production system. The selling price and resource requirements of each of these two products are as follows: Product Unit selling price ($)

L 70

M 90

Variable costs per unit: Direct labour ($7 per hour) Direct material ($5 per kg) Machine hours ($10 per hour)

28 10 10

14 45 20

Fixed overheads absorbed

12

6

Profit per unit

10

5

Fixed overheads are absorbed at the rate of $3 per direct labour hour. Market research shows that the maximum demand for products L and M during December 2010 will be 400 units and 700 units respectively. At a recent meeting of the purchasing and production managers to discuss the company’s production plans for December 2010, the following resource availability for December 2010 was identified: Direct labour Direct material Machine hours

3,500 hours 6,000 kg 2,000 hours

39 | P a g e

Required: (a) Prepare calculations to show, from a financial perspective, the optimum production plan for December 2010 and the contribution that would result from adopting your plan. (6 marks) (b) You have now presented your optimum plan to the purchasing and production managers of LM. During the presentation, the following additional information became available: (i) The company has agreed to an order for 250 units of product M for a selling price of $90 per unit from a new overseas customer. This order is in addition to the maximum demand that was previously predicted and must be produced and delivered in December 2010; (ii) The originally predicted resource restrictions were optimistic. The managers now agree that the availability of all resources will be 20% lower than their original predictions. Required: Construct the revised resource constraints and the objective function to be used to identify, given the additional information above, the revised optimum production plan for December 2010. (6 marks) (c) The resource constraints and objective function requested in part (b) above have now been processed in a simplex linear programming model and the following solution has been printed: Product L Product M Direct labour Direct material ($) Machine hours Contribution ($)

400 194 312 1.22 312 10,934.00

Product L other value Product M other value

0 506

Required: Analyse the meaning of each of the above eight values in the solution to the problem. Your answer should include a proof of the five individual values highlighted in bold. (13 marks) (Total = 25 marks)

40 | P a g e

A2 – 11 Hotel (CIMA P2 May 2011) The management of a hotel is planning for the next year. The hotel has 100 bedrooms. The price of a room night includes breakfast for the guests. Other services (a snack service and a bar and restaurant) are available but are not included in the price of the room night. These additional services are provided to hotel guests only.

For planning purposes the hotel divides the year (based on 360 days) into three seasons: peak, mid and low. Details of the hotel and its services and forecasts for the next year are given below. 1. Seasons, room charges, room occupancy, guests per room and room revenue The hotel charges a price per room per night (including breakfast) irrespective of the number of guests per room. The price charged is different in each of the seasons. Season

Peak

Number of days 90 Price charged per room per night ($) 100.00 Hotel room occupancy % 95 Average number of guests per room 1.8 Total room revenue ($) 855,000

Mid

Low

120 80.00 75 1.5 720,000

150 55.00 50 1.2 412,500

2. Guest related costs The hotel incurs some costs, including providing breakfast, that are directly related to the number of guests in the hotel. These are $12 per guest per night in all seasons. 3. Room related costs The hotel incurs some costs that are directly related to the number of rooms occupied. These include cleaning and laundry costs of $5 per occupied room per night regardless of season. There are also power and lighting costs of $3 in the peak season, $4 in the mid season and $6 in the low season per occupied room per night. 4. Hot snacks The hotel offers a 24 hour hot snacks service to the guests. Past records show that this service has been used by 30% of its guests in the mid and low seasons but only 10% in the peak season. It is forecast that the average spend per guest per night will be $10. The hotel earns a 30% gross contribution from this income. The hotel employs a cook on a salary of $20,000 per year to provide this service. All of the costs for the hot snacks service, except for the cook’s salary, are variable. The cook could be made redundant with no redundancy costs.

41 | P a g e

5. Restaurant & Bar Past records show that the usage of the restaurant and bar is seasonal. The restaurant and bar are particularly popular with the hotel’s business guests. The forecast usage is shown below.

Season Peak Mid Low

Daily demand 30% of hotel guests spend an average of $15 each 50% of hotel guests spend an average of $20 each 70% of hotel guests spend an average of $30 each

The hotel earns a 25% gross contribution from this income and employs two chefs on a combined salary of $54,000 per year to provide this facility. All of the costs in the restaurant and bar, except for the salaries of the chefs, are variable. The two chefs could be made redundant with no redundancy costs. 6. General hotel costs. These include the costs of reception staff, the heating and lighting of the common areas and other facility related costs. The forecast costs for next year are: Peak season Mid season Low season

$300,000 $400,000 $500,000

These costs could be reduced by 75% if the hotel were to close temporarily for one or more seasons of the year. There are also some costs that are incurred by the hotel and can only be avoided by its permanent closure. These are estimated to $200,000 for next year. Required: (a) Prepare, in an appropriate format, a columnar statement that will help the managers of the hotel to plan for next year. Your statement should show the hotel’s activities by season and in total. (18 marks) (b) (i) Identify, based on your statement, the actions that the managers could take to maximise the profit of the hotel for next year. (3 marks)

42 | P a g e

(ii) Explain TWO factors that the managers should consider before implementing the actions you identified in (b)(i). (4 marks) (Total = 25 marks)

43 | P a g e

Questions – Section A Part B Cost Planning and Analysis for Competitive Advantage B1 – 1 SWAL (CIMA P2 Pilot paper 2005) SW is a member of the SWAL Group of companies. SW manufactures cleaning liquid using chemicals that it buys from a number of suppliers. In the past SW has used a periodic review stock control system with maximum, minimum and re-order levels to control the purchase of the chemicals and the economic order quantity model to minimise its costs. The Managing Director of SW is considering a change by introducing a Just in Time (JIT) system. Required: As Management Accountant, prepare a report to the Managing Director that explains how a JIT system differs from the system presently being used and the extent to which its introduction would require a review of SW’s quality control procedures. (10 marks) B1 – 2 X group (CIMA P2 May 2005) The X group is a well-established manufacturing group that operates a number of companies using similar production and inventory holding policies. All of the companies are in the same country though there are considerable distances between them. The group has traditionally operated a constant production system whereby the same volume of output is produced each week, even though the demand for the group’s products is subject to seasonal fluctuations. As a result there is always finished goods inventory in the group’s warehouses waiting for customer orders. This inventory will include a safety inventory equal to two weeks’ production. Raw material inventories are ordered from suppliers using the Economic Order Quantity (EOQ) model in conjunction with a computerised inventory control system which identifies the need to place an order when the re-order level is reached. The purchasing department is centralised for the group. On receiving a notification from the computerised inventory control system that an order is to be placed, a series of quotation enquiries are issued to prospective suppliers so that the best price and delivery terms are obtained for each order. This practice has resulted in there being a large number of suppliers to the X group. Each supplier delivers directly to the company that requires the material.

44 | P a g e

The Managing Director of the X group has recently returned from a conference on World Class Manufacturing and was particularly interested in the possible use of Just In Time (JIT) within the X group. Required: Write a report, addressed to the Managing Director of the X group that explains how the adoption of JIT might affect its profitability. (10 marks) B1 – 3 ML (CIMA P2 Nov 2005) ML is an engineering company that specialises in providing engineering facilities to businesses that cannot justify operating their own facilities in-house. ML employs a number of engineers who are skilled in different engineering techniques that enable ML to provide a full range of engineering facilities to its customers. Most of the work undertaken by ML is unique to each of its customers, often requiring the manufacture of spare parts for its customers’ equipment, or the building of new equipment from customer drawings. As a result most of ML’s work is short-term, with some jobs being completed within hours while others may take a few days. To date ML has adopted a cost plus approach to setting its prices. This is based upon an absorption costing system that uses machine hours as the basis of absorbing overhead costs into individual job costs. The Managing Director is concerned that over recent months ML has been unsuccessful when quoting for work with the consequence that there has been an increase in the level of unused capacity. It has been suggested that ML should adopt an alternative approach to its pricing based on marginal costing since “any price that exceeds variable costs is better than no work”. Required: With reference to the above scenario (i) briefly explain absorption and marginal cost approaches to pricing; (ii) discuss the validity of the comment “any price that exceeds variable costs isbetter than no work”. (10 marks)

45 | P a g e

B1 – 4 PK plc (CIMA P2 Nov 2005) You are the assistant management accountant within PK plc. PK plc manufactures high quality self-assembly furniture from raw materials utilising highly skilled labour within a computer-controlled manufacturing facility. The company produces a range of furniture, and, because of the lead time to receive delivery of its raw materials, has a finished goods inventory policy of holding an average of two weeks estimated sales in inventory. Customer demand is seasonal and, as a consequence, this finished goods inventory level fluctuates throughout the year. The company also holds inventories of raw materials based upon estimates of its production requirements. An absorption costing system is used to attribute all manufacturing costs to output. Increasingly PK plc is facing competition, particularly from overseas manufacturers and its sales team have to make decisions about the extent to which it can offer price discounts in order to win customer orders. Required: Prepare a report addressed to the Management Team of PK plc that explains the changing nature of cost structures in the modern manufacturing environment and the implications for PK plc’s (i) inventory valuation (ii) short term decision making (10 marks) Note: There are 2 marks available for format and presentational style B1 – 5 Financial advisors (CIMA P2 May 2006) A firm of financial advisors has established itself by providing high quality, personalised, financial strategy advice. The firm promotes itself by sponsoring local events, advertising, client newsletters, having a flexible attitude towards the times and locations of meetings with clients and seeking new and innovative ideas to discuss with its clients. The senior manager of the firm has recently noticed that the firm’s profitability has declined, with fewer clients being interested in the firm’s new investment ideas. Indeed, many clients have admitted to not reading the firm’s newsletters. The senior manager seeks your help in restoring the firm’s profitability to its former level and believes that the techniques of Value Analysis and Functional Analysis may be appropriate.

46 | P a g e

Required: (a) Explain the meanings of, and the differences between, Value Analysis and Functional Analysis. (4 marks) (b) Briefly explain the series of steps that you would take to implement Value Analysis for this organisation. (6 marks) (Total = 10 marks) B1 – 6 Compliance v conformance (CIMA P2 May 2006) The Managing Director of a manufacturing company based in Eastern Europe has recently returned from a conference on modern manufacturing. One of the speakers at the conference presented a paper entitled “Compliance versus Conformance – the quality control issue”. The Managing Director would like you to explain to her some of the concepts that she heard about at the conference. Required: Prepare a report, addressed to the Managing Director, that discusses quality costs and theirsignificance for the company. Your report should include examples of the different quality costs and their classification within a manufacturing environment. (10 marks) Note: 2 marks are available for report format B1 – 7 AVN (CIMA P2 Nov 2006) AVN designs and assembles electronic devices to allow transmission of audio / visual communications between the original source and various other locations within the same building. Many of these devices require a wired solution but the company is currently developing a wireless alternative. The company produces a number of different devices depending on the number of input sources and the number of output locations, but the technology used within each device is identical. AVN is constantly developing new devices which improve the quality of the audio / visual communications that are received at the output locations. The Managing Director recently attended a conference on world class manufacturing entitled “The extension of the value chain to include suppliers and customers” and seeks your help. 47 | P a g e

Required: Explain (i) the components of the extended value chain; and (3 marks) (ii) how each of the components may be applied by AVN. (7 marks) (Total = 10 marks) B1 – 8 W (CIMA P2 Nov 2006) W has recently completed the development and testing of a new product which has cost $400,000. It has also bought a machine to produce the new product costing $150,000. The production machine is capable of producing 1,000 units of the product per month and is not expected to have a residual value due to its specialised nature. The company has decided that the unit selling prices it will charge will change with the cumulative numbers of units sold as follows: Cumulative sales units 0 to 2,000 2,001 to 7,000 7,001 to 14,500 14,501 to 54,500 54,501 and above

Selling price $ per unit in this band 100 80 70 60 40

Based on these selling prices, it is expected that sales demand will be as shown below: Months 1 – 10 11 – 20 21 – 30 31 – 70 71 – 80 81 – 90 91 – 100 101 – 110 Thereafter

Sales demand per month (units) 200 500 750 1,000 800 600 400 200 NIL

48 | P a g e

Unit variable costs are expected to be as follows:

First 2,000 units Next 12,500 units Next 20,000 units Next 20,000 units Thereafter

$ per unit 50 40 30 25 30

W operates a Just in Time (JIT) purchasing and production system and operates its business on a cash basis. A columnar cash flow statement showing the cumulative cash flow of the product after its “Introduction” and “Growth” stages has already been completed and this is set out below:

Months Number of units produced and sold Selling price per unit Unit variable cost Unit contribution Total contribution Cumulative cash flow

Introduction 1-10 2,000 $100 $50 $50 $100,000 ($450,000)

Growth 11-30 5,000 7,500 $80 $70 $40 $40 $40 $30 $425,000 ($25,000)

Required: (a) Complete the cash flow statement for each of the remaining two stages of the product’s life cycle. Do not copy the Introduction and Growth stages in your answer. Ignore the time value of money. (5 marks) (b) Explain, using your answer to (a) above and the data provided, the possible reasons for the changes in costs and selling prices during the life cycle of the product. (5 marks) (Total = 10 marks) B1 – 9 New product (CIMA P2 May 2007) A company is planning to launch a new product. It has already carried out market research at a cost of $50,000 and as a result has discovered that the market price for the product should be $50 per unit. The company estimates that 80,000 units of the product could be sold at this price before one of the company’s competitors enters the market with a superior product. At this time any unsold units of the company’s product would be of no value. 49 | P a g e

The company has estimated the costs of the initial batch of the product as follows:

Direct materials Direct labour ($10 per hour) Other direct costs

$000 200 250 100

Production was planned to occur in batches of 10,000 units and it was expected that an 80% learning curve would apply to the direct labour until the fourth batch was complete. Thereafter the direct labour cost per batch was expected to be constant. No changes to the direct labour rate per hour were expected. The company introduced the product at the price stated above, with production occurring in batches of 10,000 units. Direct labour was paid using the expected hourly rate of $10 and the company is now reviewing the profitability of the product. The following schedule shows the actual direct labour cost recorded: Cumulative number of batches 1 2 4 8

Actual cumulative direct labour cost $000 280 476 809 1,376

Required: (i) Calculate the revised expected cumulative direct labour costs for the four levels of output given the actual cost of $280,000 for the first batch. (ii) Calculate the actual learning rate exhibited at each level of output. (iii) Discuss the implications of your answers to (i) and (ii) for the managers of the company. (10 marks) B1 – 10 New small company (CIMA P2 Nov 2007) You are the management accountant of a new small company that has developed a new product using a labour intensive production process. You have recently completed the budgets for the company for next year and, before they are approved by the Board of Directors, you have been asked to explain your calculation of the labour time required for the budgeted output. In your calculations, you anticipated that the time taken for the first unit would be 40 minutes and that a 75% learning curve would apply for the first 30 units.

50 | P a g e

Required: (a) Explain the concept of the learning curve and why it may be relevant to the above company. (3 marks) (b) Calculate the expected time for the 6th unit of output. (3 marks) (c) Discuss the implications of the learning curve for a company adopting a penetration pricing policy. (4 marks) (Total = 10 marks) Note: The learning index for a 75% learning curve is -0.415 B1 – 11 XY (CIMA P2 May 2008) You are the Management Accountant of XY, an engineering company that assembles components into engines for sale to the automotive industry. The company is constantly under pressure from its customers to provide more efficient engines, which are also less damaging to the environment. The company uses value chain analysis as a tool in the management of its activities. The Managing Director of XY has recently been invited to a conference to give a presentation entitled “The concept of the Value Chain and the management of profits generated throughout the chain in XY.” Required: Prepare a report for the Managing Director explaining the points that should be covered in the presentation. (10 marks) B1 – 12 Inventory levels (CIMA P2 May 2008) A company experiences changing levels of demand, but produces a constant number of units during each quarter. The company allows inventory levels to rise and fall to satisfy the differing quarterly demand levels for its product.

51 | P a g e

Required: (a) Identify and explain the reasons for THREE cost changes that would result if the company changed to a Just-In-Time production method for 2009. Assume there will be no inventory at the start and end of the year. (6 marks) (b) Briefly discuss the importance of Total Quality Management to a company that operates a Just-In-Time production method. (4 marks) (Total = 10 marks) B1 – 13 Workshop (CIMA P2 May 2008) A company has developed a new product that it will manufacture in its workshop. The product is highly specialised and initially will be produced to order only. The product will be manufactured in batches. The estimated labour time required for the first batch is 40 hours, but due to the nature of the product and the manufacturing method to be used, it is expected that an 80% learning curve will apply. Required: (a) Calculate the expected time for the eighth batch. (3 marks) (b) When production commenced the first batch took 45 hours. The actual learning rates observed were as follows: Month 1 2 3 4

Total batches produced to date 1 2 4 8

Actual learning rate 75% 75% 90%

For each of months 2 and 4, state possible reasons why the actual learning rates differed from the expected rates. (3 marks) (c) The total time taken to produce the first eight batches was 182⋅25 hours. Calculate the cumulative learning rate up to the end of Month 4. (Remember that the first batch took 45 hours). (4 marks) (Total = 10 marks)

52 | P a g e

B1 – 14 Out-turn performace report (CIMA P2 May 2010) The budget for the production cost of a new product was based on the following assumptions: (i) Time for the 1st batch of output = 10 hours (ii) Learning rate = 80% (iii)Learning will cease after 40 batches, and thereafter the time per batch will be the same as the time of the final batch during the learning period, i.e. the 40th batch (iv) Standard direct labour rate per hour = $12.00 An extract from the out-turn performance report based on the above budget is as follows: Budget

Actual

Variance

Output (batches)

60

50

10 adverse

Direct labour hours

163.53

93.65

69.88 favourable

Direct labour cost

$1,962

$1,146

$816 favourable

Further analysis has shown that, due to similarities between this product and another that was developed last year, the rate of learning that should have been expected was 70% and that the learning should have ceased after 30 batches. Other budget assumptions for the new product remain valid. Required: (a) Prepare a revised out-turn performance report for the new product that: (i) shows the flexed budgeted direct labour hours and direct labour cost based on the revised learning curve data, and (ii) shows the variances that reconcile the actual results to your flexed budget in as much detail as possible. (7 marks) (b) Explain why your report is more useful to the production manager than the report shown above. (3 marks) Note: The learning index values for an 80% and a 70% learning curve are -0.3219 and 0.5146 respectively. (Total 10 marks)

53 | P a g e

B1 – 15 PQ (CIMA P2 May 2010) PQ manufactures and sells consumer electronics. It is constantly working to design the latest gadgets and “must-haves” which are unique in the market place at the time they are launched. The management of PQ are aware of the short product life cycles in this competitive market and consequently use a market skimming pricing strategy at the introduction stage. Required: Explain the changes that are likely to occur in the following items at the three later stages in the product life cycle of a typical PQ product. (i) Selling price (ii) Production costs (iii) Selling and marketing costs (Total = 10 marks) B1 – 16 Timber products (CIMA P2 May 2010) XY, a company that manufactures a range of timber products, is considering changing to a just-in-time (JIT) production system. Currently XY employs staff who are contracted to work and be paid for a total of 3,937.75 hours per month. Their labour efficiency ratio is 96% and, as a result, they are able to produce 3,780 standard hours of output each month in normal working hours. Overtime working is used to meet additional demand, though the management of XY try to avoid the need for this because it is paid at a 50% premium to the normal hourly rate of $10 per hour. Instead, XY plan production so that in months of lower demand inventory levels increase to enable sales demand to be met in other months. XY has determined that the cost of holding inventory is $6 per month for each standard hour of output that is held in inventory. XY has forecast the demand for its products for the next six months as follows: Month 1 2 3 4 5 6

Demand (Standard hours) 3,100 3,700 4,000 3,300 3,600 4,980

54 | P a g e

You may assume that all production costs (other than labour) are either fixed or are not driven by labour hours worked, and that there is zero inventory at the start of month 1 and at the end of month 6. Assume also that production and sales occur evenly during each month at present, and that the minimum contracted hours will remain the same with the JIT system. Required: (a) With the current production system: (i) Calculate for each of the six months and the period in total, the total inventory holding costs. (ii) Calculate the total production cost savings made by changing to a JIT production system. (6 marks) (b) Explain TWO other factors that should be considered by XY before changing to a JIT production system. (4 marks) (Total = 10 marks) B1 – 17 LMN (CIMA P2 May 2010) LMN comprises three trading divisions plus a Head Office. There is a director for each trading division and, in addition, there is a Managing Director who is based in Head Office. Divisional directors are empowered to make decisions concerning the day to day operations of their division and investment decisions requiring an initial investment up to $100,000. Investment decisions involving greater initial expenditure must be authorised by the Managing Director. Inter-divisional trading occurs between all of the trading divisions. The transfer prices are determined by Head Office. Head Office provides services and facilities to each of the trading divisions. At the end of each month, the actual costs of Head Office are apportioned to the trading divisions. Each Head Office cost is apportioned to the trading divisions using an appropriate basis. The bases used are: number of employees; value of sales; capital invested; and standard hours of service delivered. The Head Office costs, together with the costs and revenues generated at divisional level, are summarised in a divisional performance statement each month. The divisional directors are not happy with the present performance statement and how it is used to appraise their performance.

55 | P a g e

Required: (a) Explain, using examples from the scenario, three issues that LMN should consider when designing a new divisional performance statement. (6 marks) LMN is thinking of introducing Activity Based Costing at its Head Office to help with the apportionment of all its costs to the divisions. (b) Discuss the advantages of applying Activity Based Costing to apportion all of the Head Office costs. (4 marks) (Total = 10 marks) B1 – 18 Production manager (CIMA P2 Nov 2010) The following variances have been calculated in respect of a new product: Direct labour efficiency variance $14,700 Favourable Direct labour rate variance $ 5,250 Adverse The variances were calculated using standard cost data which showed that each unit of the product was expected to take 8 hours to produce at a cost of $15 per hour. Actual output of the product was 560 units and actual time worked in the manufacture of the product totalled 3,500 hours at a cost of $57,750. However, the production manager now realises that the standard time of 8 hours per unit was the time taken to produce the first unit and that a learning rate of 90% should have been anticipated for the first 600 units. Required: (a) Calculate planning and operating variances following the recognition of the learning curve effect. (6 marks) (b) Explain the importance of learning curves in the context of Target Costing. (4 marks) Note: The learning index for a 90% learning curve is -0.1520 (Total = 10 marks)

56 | P a g e

B1 – 19 CAL (CIMA P2 Nov 2010) CAL manufactures and sells solar panels for garden lights. Components are bought in and assembled into metal frames that are machine manufactured by CAL. There are a number of alternative suppliers of these solar panels. Some of CAL’s competitors charge a lower price, but supply lower quality panels; whereas others supply higher quality panels than CAL but for a much higher price. CAL is preparing its budgets for the coming year and has estimated that the market demand for its type of solar panels will be 100,000 units and that its share will be 20,000 units (i.e. 20% of the available market). The standard cost details of each solar panel are as follows: $ per unit Selling price 60 Bought - in components (1 set) 15 Assembly & machining cost 25 Delivery cost 5 45 Contribution 15 An analysis of CAL’s recent performance revealed that 2% of the solar panels supplied to customers were returned for free replacement, because the customer found that they were faulty. Investigation of these returned items shows that the components had been damaged when they had been assembled into the metal frame. These returned panels cannot be repaired and have no scrap value. If the supply of faulty solar panels to customers could be eliminated then, due to improved customer perception, CAL’s market share would increase to 25%. Required: (a) Explain, with reference to CAL, quality conformance costs and quality nonconformance costs and the relationship between them. (4 marks) (b) Assuming that CAL continues with its present systems and that the percentage of quality failings is as stated above: (i) Calculate, based on the budgeted figures and sales returns rate, the total relevant costs of quality for the coming year. (4 marks) (ii) Calculate the maximum saving that could be made by implementing an inspection process for the solar panels, immediately before the goods are delivered. (2 marks) (Total = 10 marks)

57 | P a g e

B1 – 20 QW (CIMA P2 Nov 2010) QW is a company that manufactures machine parts from sheet metal to specific customer order for industrial customers. QW is considering diversification into the production of metal ornaments. The ornaments would be produced at a constant rate throughout the year. It then plans to sell these ornaments from inventory through wholesalers and via direct mail to consumers. Presently, each of the machine parts is specific to a customer’s order. Consequently, the company does not hold an inventory of finished items but it does hold the equivalent of one day’s production of sheet metal so as to reduce the risk of being unable to produce goods demanded by customers at short notice. There is a one day lead time for delivery of sheet metal to QW from its main supplier though additional supplies could be obtained at less competitive prices. Demand for these industrial goods is such that delivery is required almost immediately after the receipt of the customer order. QW is aware that if it is unable to meet an order immediately the industrial customer would seek an alternative supplier, despite QW having a reputation for high quality machine parts. The management of QW is not aware of the implications of the diversification for its production and inventory policies. Required: (a) Compare and contrast QW’s present production and inventory policy and practices with a traditional production system that uses constant production levels and holds inventory to meet peaks of demand. (5 marks) (b) Discuss the importance of a Total Quality Management (TQM) system in a just-intime (JIT) environment. Use QW to illustrate your discussion. (5 marks) (Total = 10 marks) B1 – 21 Accountancy services (CIMA P2 Nov 2010) XY provides accountancy services and has three different categories of client: limited companies, self employed individuals, and employed individuals requiring taxation advice. XY currently charges its clients a fee by adding a 20% mark-up to total costs. Currently the costs are attributed to each client based on the hours spent on preparing accounts and providing advice.

58 | P a g e

XY is considering changing to an activity based costing system. The annual costs and the causes of these costs have been analysed as follows: $ 580,000 30,000 15,000 60,000 40,000

Accounts preparation and advice Requesting missing information Issuing fee payment reminders Holding client meetings Travelling to clients

The following details relate to three of XY’s clients and to XY as a whole: Client XY Hours spent on preparing accounts and providing advice Requests for missing information Payment reminders sent Client meetings held Miles travelled to meet clients

A

B

C

1,000 4 2 4 150

250 10 8 1 600

340 6 10 2 0

18,000 250 400 250 10,000

Required: Prepare calculations to show the effect on fees charged to each of these three clients of changing to the new costing system. (10 marks) B1 – 22 PT (CIMA P2 May 2011) PT manufactures and sells a number of products. All of its products have a life cycle of six months or less. PT uses a four stage life cycle model (Introduction; Growth; Maturity; and Decline) and measures the profits from its products at each stage of their life cycle. PT has recently developed an innovative product. Since the product is unique it was decided that it would be launched with a market skimming pricing policy. However PT expects that other companies will try to enter the market very soon. This product is generating significant unit profits during the Introduction stage of its life cycle. However there are concerns that the unit profits will reduce during the other stages of the product’s life cycle.

59 | P a g e

Required: For each of the (i) Growth; and (ii) Maturity stages of the new product’s life cycle explain the likely changes that will occur in the unit selling prices AND in the unit production costs, compared to the preceding stage. (Total = 10 marks) B1 – 23 TQM and JIT (CIMA P1 Pilot Paper 2005) Give FOUR reasons why the adoption of Total Quality Management (TQM) is particularly important within a Just-in-Time (JIT) production environment. (5 marks) B1 – 24 Standard costing (CIMA P1 May 2006) Briefly discuss three reasons why standard costing may not be appropriate in a modern business environment. (5 Marks) B1 – 25 Marginal v throughput (CIMA P1 May 2006) Compare and contrast marginal costing and throughput accounting. (5 Marks) B1 – 26 MRPS (CIMA P1 May 2007) Briefly explain the role of a Manufacturing Resource Planning System in supporting a standard costing system. (5 Marks)

B1 -27 JIT (CIMA P1 May 2007) Briefly explain the main differences between the traditional manufacturing environment and a just-in-time manufacturing environment. (5 marks)

60 | P a g e

B1 – 28 Key features of TQM (CIMA P1 May 2008) Describe THREE key features that are present in any organisation that is successfully focused on Total Quality Management (TQM). (5 marks)

61 | P a g e

Questions – Section B Part B Cost Planning and Analysis for Competitive Advantage B2 -1 The Q organisation (CIMA P2 May 2005) (a) The Q organisation is a large, worldwide respected manufacturer of consumer electrical and electronic goods. Q constantly develops new products that are in high demand as they represent the latest technology and are “must haves” for those consumers that want to own the latest consumer gadgets. Recently Q has developed a new handheld digital DVD recorder and seeks your advice as to the price it should charge for such a technologically advanced product. Required: Explain the relevance of the product life cycle to the consideration of alternative pricing policies that might be adopted by Q. (10 marks) (b) Market research has discovered that the price demand relationship for the item during the initial launch phase will be as follows: Price (£) 100 80 69 62

Demand (units) 10,000 20,000 30,000 40,000

Production of the DVD recorder would occur in batches of 10,000 units, and the production director believes that 50% of the variable manufacturing cost would be affected by a learning and experience curve. This would apply to each batch produced and continue at a constant rate of learning up to a production volume of 40,000 units when the learning would be complete. Thereafter, the unit variable manufacturing cost of the product would be equal to the unit cost of the fourth batch. The production director estimates that the unit variable manufacturing cost of the first batch would be £60 (£30 of which is subject to the effect of the learning and experience curve, and £30 of which is unaffected), whereas the average unit variable manufacturing cost of all four batches would be £52.71. There are no non-manufacturing variable costs associated with the DVD recorder. Required: (i) Calculate the rate of learning that is expected by the production director. (4 marks)

62 | P a g e

(ii) Calculate the optimum price at which Q should sell the DVD recorder in order to maximise its profits during the initial launch phase of the product. (8 marks) (iii) Q expects that after the initial launch phase the market price will be £57 per unit. Estimated product specific fixed costs during this phase of the product’s life are expected to be £15,000 per month. During this phase of the product life cycle Q wishes to achieve a target monthly profit from the product of £30,000. Calculate the number of units that need to be sold each month during this phase in order that Q achieves this target monthly profit. (3 marks) (Total = 25 marks)

B2 -2 F plc (CIMA P1 May 2005) F plc supplies pharmaceutical drugs to drug stores. Although the company makes a satisfactory return, the directors are concerned that some orders are profitable and others are not. The management has decided to investigate a new budgeting system using activity based costing principles to ensure that all orders they accept are making a profit. Each customer order is charged as follows. Customers are charged the list price of the drugs ordered plus a charge for selling and distribution costs (overheads). A profit margin is also added, but that does not form part of this analysis. Currently F plc uses a simple absorption rate to absorb these overheads. The rate is calculated based on the budgeted annual selling and distribution costs and the budgeted annual total list price of the drugs ordered. An analysis of customers has revealed that many customers place frequent small orders with each order requesting a variety of drugs. The management of F plc has examined more carefully the nature of its selling and distribution costs, and the following data have been prepared for the budget for next year: Total list price of drugs supplied Number of customer orders Selling and Distribution Costs Invoice processing Packing Delivery Other overheads Total overheads

£8m 8,000 £000 280 220 180 200 880

Cost driver See Note 2 Size of package – see Note 3 Number of deliveries – see Note 4 Number of orders

63 | P a g e

Notes: 1. Each order will be shipped in one package and will result in one delivery to the customer and one invoice (an order never results in more than one delivery). 2. Each invoice has a different line for each drug ordered. There are 28,000 invoice lines each year. It is estimated that 25% of invoice processing costs are related to the number of invoices, and 75% are related to the number of invoice lines. 3. Packing costs are £32 for a large package, and £25 for a small package. 4. The delivery vehicles are always filled to capacity for each journey. The delivery vehicles can carry either 6 large packages or 12 small packages (or appropriate combinations of large and small packages). It is estimated that there will be 1,000 delivery journeys each year, and the total delivery mileage that is specific to particular customers is estimated at 350,000 miles each year. £40,000 of delivery costs are related to loading the delivery vehicles and the remainder of these costs are related tospecific delivery distance to customers. The management has asked for two typical orders to be costed using next year’s budget data, using the current method, and the proposed activity-based costing approach. Details of two typical orders are shown below:

Lines on invoice Package size Specific delivery distance List price of drugs supplied

Order A 2 small 8 miles £1,200

Order B 8 large 40 miles £900

Required: (a) Calculate the charge for selling and distribution overheads for Order A and Order B using: (i) the current system; and (ii) the activity-based costing approach. (10 marks) (b) Write a report to the management of F plc in which you (i) assess the strengths and weaknesses of the proposed activity-based costing approach for F plc; and (5 marks)

64 | P a g e

(ii) recommend actions that the management of F plc might consider in the light of the data produced using the activity-based-costing approach. (5 marks) (Total = 20 marks) B2 – 3 KL (CIMA P2 Nov 2006) KL manufactures three products, W, X and Y. Each product uses the same materials and the same type of direct labour but in different quantities. The company currently uses a cost plus basis to determine the selling price of its products. This is based on full cost using an overhead absorption rate per direct labour hour. However, the Managing Director is concerned that the company may be losing sales because of its approach to setting prices. He thinks that a marginal costing approach may be more appropriate, particularly since the workforce is guaranteed a minimum weekly wage and has a three month notice period. Required: (a) Given the Managing Director’s concern about KL’s approach to setting selling prices, discuss the advantages and disadvantages of marginal cost plus pricing AND total cost plus pricing. (6 marks) The direct costs of the three products are shown below: Product Budgeted annual production (units)

W 15,000

X 24,000

Y 20,000

Direct materials Direct labour ($10 per hour)

$ per unit 35 40

$ per unit 45 30

$ per unit 30 50

In addition to the above direct costs, KL incurs annual indirect production costs of $1,044,000. Required: (b) Calculate the full cost per unit of each product using KL’s current method of absorption costing. (4 marks)

65 | P a g e

An analysis of the company’s indirect production costs shows the following: $ 220,000 100,000 400,000 324,000

Material ordering costs Machine setup costs Machine running costs General facility costs

Cost driver Number of supplier orders Number of batches Number of machine hours Number of machine hours

The following additional data relate to each product: Product Machine hours per unit Batch size (units) Supplier orders per batch

W 5 500 4

X 8 400 3

Y 7 1,000 5

Required: (c) Calculate the full cost per unit of each product using Activity Based Costing. (8 marks) (d) Explain how Activity Based Costing could provide information that would be relevant to the management team when it is making decisions about how to improve KL’s profitability. (7 marks) (Total = 25 marks) B2 – 4 Retail outlet (CIMA P2 Nov 2007) A small retail outlet sells four main groups of products: Basic Foods (milk, bread, etc); Newspapers & Magazines; Frozen Foods; and Canned Foods. A budgeted weekly profit statement is shown below:

Sales revenue Cost of sales Gross margin Power for freezers* Overheads** Net margin

Basic Foods

Newspapers and Magazines

Frozen Foods

Canned Foods

$ 800 600 200

$ 1,000 700 300

$ 2,400 1,200 1,200

100 100

100 200

$ 1,500 550 950 100 200 650

400 800

*The freezers would be emptied and switched off as necessary during redecoration. 66 | P a g e

**Overhead costs comprise general costs of heating and lighting, rent and rates, and other general overhead costs. These costs are attributed to products in proportion to the floor area occupied by each product group which is as follows: Basic Foods Floor area (m2)

50

Newspapers and Magazines 50

Frozen Foods

Canned foods

100

200

For each product group, analysis has shown that the sales revenue achieved changes in direct proportion to the floor space allocated to the product. The owner of the retail outlet has decided that the premises need to be redecorated but is undecided as to which of the following two options would be the most profitable. Option 1 Close the retail outlet completely for four weeks while the redecoration takes place. The company that is to complete the redecoration would charge $2,500 under this option. It is expected that following the re-opening of the retail outlet there would be a loss of sales for the next 12 weeks because customers would have had to find alternative suppliers for their goods. The reduction in sales due to lost customers has been estimated to be 30% of the budgeted sales during the first four weeks of reopening; 20% during the next four weeks; and 10% during the third four weeks. In addition, in order to encourage customers to return to the retail outlet, there would be a 10% price reduction on all Basic Foods and Canned Foods for the entire 12 week period. Option 2 Continue to open the retail outlet while the redecoration takes place but with a reduced amount of floor area. The useable floor area would be reduced to 40% of that originally available. After three weeks, the retail outlet would be closed for 0•5 weeks while the goods are moved to the newly redecorated area. The retail outlet would then continue to operate using 40% of its original floor area for a further three weeks before the work was fully completed. The company that is to complete the redecoration would charge $3,500 under this option, and in addition there would be product movement costs of $1,000. The owner has determined that in order to avoid losing customers there should be no reduction in the amount of floor area given to Basic Foods and Newspapers and Magazines throughout this period. The floor area to be used by Frozen Foods and Canned Foods should be determined on the basis of their profitability per unit of area. However, the Frozen Foods are presently kept in four freezers, and therefore any reductions in floor area must be determined by complete freezer units. It may be assumed that each freezer unit incurs equal amounts of power costs.

67 | P a g e

Required: (a) Advise the owner of the retail outlet which option to choose in order to minimise the losses that will occur as a result of the decision. All workings must be shown. (15 marks) (b) Explain how Activity Based Costing may be used in a retail environment to improve the decision making and profitability of the business. (10 marks) (Total = 25 marks)

68 | P a g e

Questions – Section A Part C Budgeting and Management Control C1 -1 Solicitors firm (CIMA P2 May 2010) A firm of solicitors is using budgetary control during 2010. The senior partner estimated the demand for the year for each of the firm’s four divisions: Civil, Criminal, Corporate, and Property. A separate partner is responsible for each division. Each divisional partner then prepared a cost budget based on the senior partner’s demand estimate for the division. These budgets were then submitted to the senior partner for his approval. He then amended them as he thought appropriate before issuing each divisional partner with the final budget for the division. He did not discuss these amendments with the respective divisional partners. Actual performance is then measured against the final budgets for each month and each divisional partner’s performance is appraised by asking the divisional partner to explain the reasons for any variances that occur. The Corporate partner has been asked to explain why her staff costs exceeded the budgeted costs for last month while the chargeable time was less than budgeted. Her reply is below: “My own original estimate of staff costs was higher than the final budgeted costs shown on my divisional performance report. In my own cost budget I allowed for time to be spent developing new services for the firm’s corporate clients and improving the clients’ access to their own case files. This would improve the quality of our services to clients and therefore increase client satisfaction. The trouble with our present system is that it focuses on financial performance and ignores the other performance indicators found in modern performance management systems.” Required: (a) Discuss the present budgeting system and its likely effect on divisional partner motivation. (6 marks) (b) Explain two non-financial performance indicators (other than client satisfaction and service quality) that could be used by the firm. (4 marks) (Total = 10 marks)

69 | P a g e

C1 – 2 DW (CIMA P2 Nov 2010) DW, a transport company, operates three depots. Each depot has a manager who reports directly to the Operations Director. For many years the depot managers have been asked by the Operations Director to prepare a budget for their depot as part of the company’s annual budgeting process. A new depot manager has been appointed to the Southern region and he has concerns about the validity of these annual budgets. He argues that they soon become out of date as operational circumstances change. At a recent manager’s meeting he said, “They are restrictive. They do not permit the depot managers to make decisions in response to operational changes, or change working practices for next year until that year’s budget has been approved.” Required: (a) Explain the differences between the above annual budgeting system and a rolling budget system. (4 marks) (b) Discuss how the Southern region depot manager could use a rolling budget system to address his concerns. (6 marks) (Total = 10 marks) C1 – 3 JYT (CIMA P2 May 2011) JYT manufactures and sells a range of products. It is not dominant in the market in which it operates and, as a result, it has to accept the market price for each of its products. The company is keen to ensure that it continues to compete and earn satisfactory profit at each stage throughout a product’s life cycle. Required: Explain how JYT could use Target Costing AND Kaizen Costing to improve itsfuture performance. Your answer should include an explanation of the differences between Target Costing and Kaizen Costing. (Total = 10 marks)

70 | P a g e

C1 – 4 DVD (CIMA P2 May 2011) A company produces and sells DVD players and Blu-ray players. Extracts from the budget for April are shown in the following table:

DVD Blu-ray

Sales (players) 3,000 1,000

Selling price (per player) $75 $200

Standard cost (per player) $50 $105

The Managing Director has sent you a copy of an e-mail she received from the Sales Manager. The content of the e-mail was as follows: We have had an excellent month. There was an adverse sales price variance on the DVDs of $18,000 but I compensated for that by raising the price of Blu-ray players. Unit sales of DVD players were as expected but sales of the Blu-rays were exceptional and gave a total sales volume profit variance of $19,000. I think I deserve a bonus! The Managing Director has asked for your opinion on these figures. You obtained the following information: Actual results for April were:

DVD Blu-ray

Sales (players) 3,000 1,200

Selling price (per player) $69 $215

The total market demand for DVD players was as budgeted but as a result of distributors reducing the price of Blu-ray discs the total market for Blu-ray players grew by 50% in April. The company had sufficient capacity to meet the revised market demand for 1,500 units of its Blu-ray players and therefore maintained its market share. Required: (a) Calculate the following operational variances based on the revised market details: (i) The total sales mix profit margin variance (2 marks) (ii) The total sales volume profit variance (2 marks)

71 | P a g e

(b) Explain, using the above scenario, the importance of calculating planning and operational variances for responsibility centres. (6 marks) (Total = 10 marks) C1 – 5 SFG (CIMA P2 May 2011) SFG is a national hotel group that operates more than 100 hotels. The performance of the manager of each hotel is evaluated using financial measures. Many of the hotel’s managers are not happy. They believe that there can be conflict between good performance and achieving short-term profits. They are also unhappy that their profit reports include a share of head office costs and other costs that they cannot control. Required: (a) Explain why non-financial performance measures are important in the service sector. (2 marks) (b) Recommend, with reasons, TWO non-financial performance measures that SFG could use to evaluate the performance of the hotel managers. (4 marks) (c) Explain why, and how, non-controllable costs should be shown on the profit reports. (4 marks) (Total = 10 marks) C1 – 6 Feedback and feedforward (CIMA P1 Pilot Paper 2005) Briefly outline the main features of “feedback control”, and the “feedback loop”and explain how, in practice, the procedures of feedback control can be transformed into “feed-forward control”. (5 marks) C1 – 7 Profit centre managers (CIMA P1 Pilot Paper 2005) (c) Briefly outline the advantages and disadvantages of allowing profit centre managers to participate actively in the setting of the budget for their units. (5 marks)

72 | P a g e

C1 – 8 Balance scorecard (CIMA P1 May 2005) A general insurance company is about to implement a Balanced Scorecard. You are required to: (i) State the four perspectives of a Balanced Scorecard; and (ii) Recommend one performance measure that would be appropriate for a general insurance company, for each of the four perspectives, and give a reason to support each measure. (You must recommend one measure only for each perspective.) (5 marks) C1 – 9 Participation in budgets (CIMA P1 May 2005) Briefly discuss three different circumstances where participation in setting budgets is likely to contribute to poor performance from managers. (5 marks) C1 – 10 Beyond budgeting (CIMA P1 May 2005) W Limited designs and sells computer games. There are many other firms in this industry. For the last five years the senior management has required detailed budgets to be produced for each year with slightly less detailed plans for the following two years. The managing director of W Limited has recently attended a seminar on budgeting and heard the ‘Beyond Budgeting’ arguments that have been advanced by Hope and Fraser, among others. You are required to: (i) Briefly describe the ‘Beyond Budgeting’ approach; and (2 marks) (ii) Advise the management of W Limited whether or not it should change its current budgeting system to a ‘Beyond Budgeting’ approach. (3 marks) (Toal = 10 marks)

73 | P a g e

C1 – 11 J Limited (CIMA P1 Nov 2005) J Limited has recently been taken over by a much larger company. For many years the budgets in J have been set by adding an inflation adjustment to the previous year’s budget. The new owners of J are insisting on a ‘zero-base’ approach when the next budget is set, as they believe many of the indirect costs in J are much higher than in othercompanies under their control. (i) Explain the main features of ‘zero-based budgeting’. (2 marks) (ii) Discuss the problems that might arise when implementing this approach in J Limited. (3 marks) (Total = 5 marks) C1 – 12 ST plc (CIMA P1 Nov 2005) ST plc is a medium-sized engineering company using advanced technology. It has just implemented an integrated enterprise resource planning (ERP) system in place of an old MRP (manufacturing resource planning) system. Discuss the changes that are likely to be seen after the implementation of the ERP system in: (i) the budget-setting process; and (ii) the budgetary control process (5 marks) C1 – 13 W Limited (CIMA P1 Nov 2005) W Limited has conducted a review of its budget-setting procedures. The review coordinator frequently heard the following comment from staff interviewed: “It’s impossible to make this system work because senior managers want budgets to be a challenging target whereas the finance department require an accurate forecast.” Discuss the issues raised in this comment, and advise the review coordinator on practical action that could be taken to alleviate the situation described. (5 marks)

74 | P a g e

C1 – 14 T plc (CIMA P1 May 2006) T plc is a large insurance company. The Claims Department deals with claims from policy holders who have suffered a loss that is covered by their insurance policy. Policy holders could claim, for example, for damage to property, or for household items stolen in a burglary. The Claims Department staff investigate each claim and determine what, if any, payment should be made to the claimant. The manager of the Claims Department has decided to benchmark the performance of the department and has chosen two areas to benchmark:  

the detection of false claims the speed of processing claims

For each of the above two areas: (i) state and justify a performance measure (ii) explain how relevant benchmarking data could be gathered. (5 marks) C1 – 15 Product M (CIMA P1 May 2007) A company uses variance analysis to monitor the performance of the team of workers which assembles Product M. Details of the budgeted and actual performance of the team for last period were as follows:

Output of product M Wage rate Labour hours

Budget 600 units £30 per hour 900 hours

Actual 680 units £32 per hour 1,070 hours

It has now been established that the standard wage rate should have been £31.20 per hour. Required: (i) Calculate the labour rate planning variance and calculate the operational labour efficiency variance. (ii) Explain the major benefit of analysing variances into planning and operational components. (5 marks)

75 | P a g e

C1 – 16 QBD (CIMA P1 Nov 2007) QBQ produces one type of product. Details of the budgeted sales and production are given below. Selling Price and Costs per unit

Selling price Material FX: 1.5kg @ £6 per kg Conversion costs (variable) Fixed production overheads

£ 40 9 8 15

The fixed production overhead absorption rate is based on annual production overheads of £720,000 and budgeted annual output of 48,000 units. The fixed overheads will be incurred evenly throughout the year. The company also incurs fixed costs for administration of £200,000 per year. Budgeted Sales Quarter 1 2 3 4

Units 10,000 12,000 14,000 12,000

Inventory It has been decided that inventory levels are to be reduced. Details are as follows: Finished goods: 5,500 units are currently held but it has been decided that the closing inventories for Quarters 1, 2 and 3 will be 45%, 40% and 35% of the following quarter’s sales respectively. Raw materials: 4,500 kg are currently held but it has been decided that the closing inventories for Quarters 1 and 2 will be 25% and 20% of the followingquarter’s production requirements respectively. Required: (a) Prepare a materials purchase budget for Quarter 1. (5 Marks)

76 | P a g e

C1 – 17 Budgetary planning and control (CIMA P1 Nov 2007) Briefly explain three reasons why budgetary planning and control might be inappropriate in a rapidly changing business environment. (5 Marks) C1 – 18 JIT systems (CIMA P1 Nov 2007) Briefly explain Just-in-Time (JIT) and two major requirements for the successful operation of a JIT system. (5 Marks) C1 – 19 Feedback and forward (CIMA P1 Nov 2007) Explain, giving examples, how budgets can be used for feedback control and feedforward control. (5 Marks) C1 – 20 Nursing homes (CIMA P1 Nov 2007) A nursing home uses incremental budgeting. The previous period’s budget is adjusted by reference to a set of indices. It is adjusted firstly for ‘volume changes’ and then for changes in the cost of resources. The indices are referenced to the previous period’s budget by using that budget as the base index number of 100. The index numbers to be used to prepare Period 3’s budget from that of Period 2 are as follows:

Patient days House-keeping costs Nursing costs Administration costs

Index 90 106 105 104

The budget for Period 2 was: House-keeping costs (all variable) Nursing costs (see below) Administration costs (all fixed)

£ 125,000 324,000 100,000

Nursing costs are semi-variable. The nursing costs for Period 2 were adjusted from the total nursing costs of £280,000 for Period 1 by using a Patient days index of 125 and a Nursing costs index of 108.

77 | P a g e

Required: Prepare the budget for Period 3. (5 marks) C1 – 21 Participative budgeting (CIMA P1 May 2008) Explain THREE behavioural consequences that may result after the introduction of participative budgeting. (5 marks)

C1 - 22 Rolling budgets (CIMA P1 May 2008) Discuss the advantages and disadvantages of rolling budgets. (5 marks)

78 | P a g e

Questions – Section B Part C Budgeting and Management Control C2 -1 M plc (CIMA P1 May 2006) M plc designs, manufactures and assembles furniture. The furniture is for home use and therefore varies considerably in size, complexity and value. One of the departments in the company is the Assembly Department. This department is labour intensive; the workers travel to various locations to assemble and fit the furniture using the packs of finished timbers that have been sent to them. Budgets are set centrally and they are then given to the managers of the various departments who then have the responsibility of achieving their respective targets. Actual costs are compared against the budgets and the managers are then asked to comment on the budgetary control statement. The statement for April for the Assembly Department is shown below.

Assembly labour hours Assembly labour Furniture packs Other materials Overheads Total

Budget Actual 6,400 7,140 $ $ 51,970 58,227 224,000 205,000 23,040 24,100 62,060 112,340 361,070 399,667

Variance $ 6,257 19,000 1,060 50,280 38,597

Adverse Favourable Adverse Adverse Adverse

Note: the costs shown are for assembling and fitting the furniture (they do not include time spent travelling to jobs and the related costs). The hours worked by the Manager are not included in the figure given for the assembly labour hours. The Manager of the Assembly Department is new to the job and has very little previous experience of working with budgets but he does have many years’ experience as a supervisor in assembly departments. Based on that experience he was sure that the department had performed well. He has asked for your help in replying to a memo he has just received asking him to “explain the serious overspending in his department”. He has sent you some additional information about the budget: 1. The budgeted and actual assembly labour costs include the fixed salary of $2,050 for the Manager of the Assembly Department. All of the other labour is paid for the hours they work. 2. The cost of furniture packs and other materials is assumed by the central finance office of M plc to vary in proportion to the number of assembly labour hours worked.

79 | P a g e

3. The budgeted overhead costs are made up of three elements: a fixed cost of $9,000 for services from central headquarters, a stepped fixed cost which changes when the assembly hours exceed 7,000 hours, and some variable overheads. The variable overheads are assumed to vary in proportion to the number of assembly labour hours. Working papers for the budget showed the impact on the overhead costs of differing amounts of assembly labour hours: Assembly labour hours Overhead costs

5,000 $54,500

7,500 $76,500

10,000 $90,000

The actual fixed costs for April were as budgeted. Required: (a) Prepare, using the additional information that the Manager of the Assembly Department has given you, a budgetary control statement that would be more helpful to him. (7 marks) (b) (i) Discuss the differences between the format of the statement that you have produced and that supplied by M plc. (4 marks) (ii) Discuss the assumption made by the central office of M plc that costs vary in proportion to assembly labour hours. (3 marks) © Discuss whether M plc should change to a system of participative budgeting. (6 marks) (Total = 20 marks) C2 – 2 RF Ltd (CIMA P1 May 2007) RF Ltd is a new company which plans to manufacture a specialist electrical component. The company founders will invest £16,250 on the first day of operations, that is, Month 1. They will also transfer fixed capital assets to the company. The following information is available:

80 | P a g e

Sales The forecast sales for the first four months are as follows: Month

Number of components 1 1,500 2 1,750 3 2,000 4 2,100 The selling price has been set at £10 per component in the first four months. Sales receipts Time of payment Month of sale One month later Two months later Three months later

% of customers 20* 45 25 5

The balance represents anticipated bad debts. *A 2% discount is given to customers for payment received in the month of sale. Production There will be no opening inventory of finished goods in Month 1 but after that it will be policy for the closing inventory to be equal to 20% of the following month’s forecast sales. Variable production cost The variable production cost is expected to be £6.40 per component. Time of payment Direct materials Direct wages Variable production overheads Total variable cost

£ 1.90 3.30 1.20 6.40

Notes: Direct materials: 100% of the materials required for production will be purchased in the month of production. No inventory of materials will be held. Direct materials will be paid for in the month following purchase. Direct wages will be paid in the month in which production occurs.

81 | P a g e

Variable production overheads: 60% will be paid in the month in which production occurs and the remainder will be paid one month later. Fixed overhead costs Fixed overhead costs are estimated at £75,000 per annum and are expected to be incurred in equal amounts each month. 60% of the fixed overhead costs will be paid in the month in which they are incurred and 30% in the following month. The balance represents depreciation of fixed assets. Calculations are to be made to the nearest £1. Ignore VAT and Tax. Required: (a) Prepare a cash budget for each of the first three months and in total. (15 marks) (b) There is some uncertainty about the direct material cost. It is thought that the direct material cost per component could range between £1.50 and £2.20. Calculate the budgeted total net cash flow for the three month period if the cost of the direct material is: (i) £1.50 per component; or (ii) £2.20 per component. (6 marks) (c) Using your answers to part (a) and (b) above, prepare a report to the management of RF Ltd that discusses the benefits or otherwise of performing ‘what if’ analysis when preparing cash budgets. (9 marks) (Total = 30 marks) C2 – 3 Trackit (CIMA P1 May 2008) Q, a new company, is being established to manufacture and sell an electronic tracking device: the Trackit. The owners are excited about the future profits that the business will generate. They have forecast that sales will grow to 2,600 Trackits per month within five months and will be at that level for the remainder of the first year. The owners will invest a total of $250,000 in cash on the first day of operations (that is the first day of Month 1). They will also transfer non-current assets into the company. 82 | P a g e

Extracts from the company’s business plan are shown below. Sales The forecast sales for the first five months are: Month 1 2 3 4 5

Trackits (units) 1,000 1,500 2,000 2,400 2,600

The selling price has been set at $140 per Trackit. Sales receipts Sales will be mainly through large retail outlets. The pattern for the receipt of payment is expected to be as follows: Time of payment Immediately One month later Two months later Three months later

% of sales value 15 * 25 40 15

The balance represents anticipated bad debts. * A 4% discount will be given for immediate payment. Production The budget production volumes in units are: Month 1 1,450

Month 2 1,650

Month 3 2,120

Month 4 2,460

Variable production cost The budgeted variable production cost is $90 per unit, comprising: $ Direct materials 60 Direct wages 10 Variable production overheads 20 Total variable cost 90

83 | P a g e

Direct materials: Payment for purchases will be made in the month following receipt. There will be no opening inventory of materials in Month 1. It will be company policy to hold inventory at the end of each month equal to 20% at of the following month’s production requirements. The direct materials cost includes the cost of an essential component that will be bought in from a specialist manufacturer. Direct wages will be paid in the month in which the production occurs. Variable production overheads: 65% will be paid in the month in which production occurs and the remainder will be paid one month later. Fixed overhead costs Fixed overheads are estimated at $840,000 per annum and are expected to be incurred in equal amounts each month. 60% of the fixed overhead costs will be paid in the month in which they are incurred and 15% in the following month. The balance represents depreciation of non-current assets. Ignore VAT and Tax Required: (a) Prepare a cash budget for each of the first three months and for that three-month period in total. (14 marks) (b) There is some uncertainty about the cost of the specialist component (this is included in the direct material cost). It is thought that the cost of the component could range between $32 and $50 per Trackit. It is currently included in the cost estimates at $40 per Trackit. Calculate the budgeted total net cash flow for the three-month period in total if the cost of the component was: (i) $32 (ii) $50 (6 marks) (c) Prepare a report for the owners of Q that offers advice about the profitability of their business and the situation revealed by the extracts from the business plan and your answers to (a) and (b) above. (10 marks) (Total = 30 marks)

84 | P a g e

C2 – 4 X plc (CIMA P1 Nov 2006) X Plc manufactures specialist insulating products that are used in both residential and commercial buildings. One of the products, Product W, is made using two different raw materials and two types of labour. The company operates a standard absorption costing system and is now preparing its budgets for the next four quarters. The following information has been identified for Product W: Sales Selling price is £220 per unit Sales demand Quarter 1 2,250 units Quarter 2 2,050 units Quarter 3 1,650 units Quarter 4 2,050 units Quarter 5 1,250 units Quarter 6 2,050 units Costs Material A is 5 kgs per unit @ £4 per kg Material B is 3 kgs per unit @ £7 per kg Labour skilled is 4 hours per unit @ £15 per hour Labour semi-skilled is 6 hours per unit @ £9 per hour Annual overheads £280,000 40% of these overheads are fixed and the remainder varies with total labour hours. Fixed overheads are absorbed on a unit basis. Inventory holding policy Closing inventory of finished goods is 30% of the following quarter’s sales demand. Closing inventory of materials is 45% of the following quarter’s materials usage. The management team are concerned that X Plc has recently faced increasing competition in the market place for Product W. As a consequence there have been issues concerning the availability and costs of the specialised materials and employees needed to manufacture Product W, and there is concern that these might cause problems in the current budget setting process. Required: (a) Prepare the following budgets for each quarter for X Plc: (i) Production budget in units; (ii) Raw material purchases budget in kgs and value for Material B. (5 marks)

85 | P a g e

(b) X Plc has just been informed that Material A may be in short supply during the year for which it is preparing budgets. Discuss the impact this will have on budget preparation and other areas of X Plc. (5 marks) (c) Assuming that the budgeted production of Product W was 7,700 units and that the following actual results were incurred for labour and overheads in the year: Actual production

7,250 units

Actual overheads Variable Fixed

£185,000 £105,000

Actual labour costs Skilled - £16.25 per hour Semi-skilled - £8 per hour

£568,750 £332,400

Prepare a flexible budget statement for X Plc showing the total variances that have occurred for the above four costs only. (5 marks) (d) X Plc currently uses incremental budgeting. Explain how Zero Based Budgeting could overcome the problems that might be faced as a result of the continued use of the current system. (5 marks) (e) Explain how rolling budgets are used and why they would be suitable for X Plc. (5 marks) (Total = 25 marks)

86 | P a g e

Questions – Section A Part D Control and Performance Measurement of Responsibilty Centres D1 -1 EVA and RI (CIMA P2 Pilot paper 2005) Explain and discuss the similarities and differences between Residual Income and Economic Value Added as methods for assessing the performance of divisions. (5 marks) D1 – 2 Controllability principle (CIMA P1 Pilot Paper 2005) Define the “controllability principle” and give arguments for and against its implementation in determining performance measures. (5 marks) D1 – 3 Transfer pricing (CIMA P1 Pilot Paper 2005) Discuss the problems that arise specifically when determining transfer prices where divisions are located in different countries. (5 marks) D1 – 4 EVA (CIMA P1 May 2005) (i) Briefly explain the main features of Economic Value Added (EVA®) as it would be used to assess the performance of divisions. (2 marks) (ii) Briefly explain how the use of EVA® to assess divisional performance might affect the behaviour of divisional senior executives. (3 marks) D1 – 5 WD, PD & TD (CIMA P1 May 2005) C plc is a large company that manufactures and sells wooden garden furniture. It has three divisions: The Wood Division (WD) purchases logs and produces finished timber as planks or beams. Approximately two-thirds of its output is sold to the Products Division, with the remainder sold on the open market.

87 | P a g e

The Products Division (PD) manufactures wooden garden furniture. The policy of C plc is that the PD must buy all its timber from the WD and sell all its output to the Trading Division. The Trading Division (TD) sells wooden garden furniture to garden centres, large supermarkets, and similar outlets. It only sells items purchased from PD. The current position is that all three divisions are profit centres and C plc uses Return on Investment (ROI) measures as the primary means to assess divisional performance. Each division adopts a cost-plus pricing policy for external sales and for internal transfers between divisions. The senior management of C plc has stated that the divisions should consider themselves to be independent businesses as far as possible. (i) For each division suggest, with reasons, the behavioural consequences that might arise as a result of the current policy for the structure and performance evaluation of the divisions. (5 marks) (ii) The senior management of C plc has requested a review of the cost-plus transfer pricing policy that is currently used. Suggest with reasons, an appropriate transfer pricing policy that could be used for transfers from PD to TD, indicating any problems that may arise as a consequence of the policy you suggest. (5 marks) D1 – 6 G group (CIMA P1 May 2007) G Group consists of several autonomous divisions. Two of the divisions supply components and services to other divisions within the group as well as to external clients. The management of G Group is considering the introduction of a bonus scheme for managers that will be based on the profit generated by each division. Required: Briefly explain the factors that should be considered by the management of G Groupwhen designing the bonus scheme for divisional managers. (5 marks)

88 | P a g e

D1 – 7 Digital equipment (CIMA P1 May 2008) A multi-national company manufactures and sells a wide range of digital equipment. The company is structured into three Divisions: Computers, Audio-visual and Photographic. The Divisions operate as investment centres and the performance of the Divisional Managers is evaluated by using Return on Investment (ROI). The Manager of the Photographic Division was concerned that the Division was falling behind its competitors in terms of financial returns and market share, and has implemented strategies to improve the situation. An external benchmarking exercise was undertaken to try to establish the position of the Division in relation to its competitors in a number of key areas. It has now been suggested that the Division should also carry out an internal benchmarking exercise. The manager of the Photographic Division is considering introducing a Balanced Scorecard to measure the success of the strategies. He has identified two perspectives and two associated goals. They are: Perspective Innovation Customer

Goal Technology Leadership Support

Required: (a) For the “Innovation Perspective” of the Division, recommend a performance measure and briefly explain how the measure will reflect the achievement of the stated goal. (3 marks) (b) For the “Customer Perspective” of the Division, state which data should be collected and explain how this could be used to ensure the goal of “support’’ is met. (2 marks) (c) Explain THREE reasons why internal benchmarking may provide information that is more useful to the Manager of the Photographic Division, in terms of monitoring and improving performance, than that provided by external benchmarking. (5 marks) (d) Explain THREE reasons why ROI may not be a good performance measure. (5 marks) (Total = 15 marks)

89 | P a g e

Questions – Section B Part D Control and Performance Measurement of Responsibilty Centres D2 -1 Y and Z (CIMA P1 Nov 2005) Y and Z are two divisions of a large company that operate in similar markets. The divisions are treated as investment centres and every month they each prepare an operating statement to be submitted to the parent company. Operating statements for these two divisions for October are shown below: Operating Statements for October

Sales revenue Less variable costs Contribution Less controllable fixed costs (includes depreciation on divisional assets) Controllable income Less apportioned central costs Net income before tax Total divisional net assets

Y £000 900 345 555 95

Z £000 555 312 243 42

460 338 122

201 180 21

£9·76m

£1·26m

The company currently has a target return on capital of 12% per annum. However, the company believes its cost of capital is likely to rise and is considering increasing the target return on capital. At present the performance of each division and the divisional management are assessed primarily on the basis of Return on Investment (ROI). Required: (a) Calculate the annualised Return on Investment (ROI) for divisions Y and Z, and discuss the relative performance of the two divisions using the ROI data and other information given above. (9 marks) (b) Calculate the annualised Residual Income (RI) for divisions Y and Z, and explain the implications of this information for the evaluation of the divisions’ performance. (6 marks)

90 | P a g e

(c) Briefly discuss the strengths and weaknesses of ROI and RI as methods of assessing the performance of divisions. Explain two further methods of assessment of divisional performance that could be used in addition to ROI or RI. (5 marks) (Total = 20 marks) D2 – 2 FP (CIMA P1 May 2006) FP sells and repairs photocopiers. The company has operated for many years with two departments, the Sales Department and the Service Department, but the departments had no autonomy. The company is now thinking of restructuring so that the two departments will become profit centres. The Sales Department This department sells new photocopiers. The department sells 2,000 copiers per year. Included in the selling price is £60 for a one year guarantee. All customers pay this fee. This means that during the first year of ownership if the photocopier needs to be repaired then the repair costs are not charged to the customer. On average 500 photocopiers per year need to be repaired under the guarantee. The repair work is carried out by the Service Department who, under the proposed changes, would charge the Sales Department for doing the repairs. It is estimated that on average the repairs will take 3 hours each and that the charge by the Service Department will be £136,500 for the 500 repairs. The Service Department This department has two sources of work: the work needed to satisfy the guarantees for the Sales Department and repair work for external customers. Customers are charged at full cost plus 40%. The details of the budget for the next year for the Service Department revealed standard costs of: Parts Labour Variable overheads Fixed overheads

at cost £15 per hour £10 per labour hour £22 per labour hour

The calculation of these standards is based on the estimated maximum market demand and includes the expected 500 repairs for the Sales Department. The average cost of the parts needed for a repair is £54. This means that the charge to the Sales Department for the repair work, including the 40% mark-up, will be £136,500.

91 | P a g e

Proposed Change It has now been suggested that FP should be structured so that the two departments become profit centres and that the managers of the Departments are given autonomy. The individual salaries of the managers would be linked to the profits of their respective departments. Budgets have been produced for each department on the assumption that the Service Department will repair 500 photocopiers for the Sales Department and that the transfer price for this work will be calculated in the same way as the price charged to external customers. However the manager of the Sales Department has now stated that he intends to have the repairs done by another company, RS, because they have offered to carry out the work for a fixed fee of £180 per repair and this is less than the price that the Sales Department would charge. Required: (a) Calculate the individual profits of the Sales Department and the Service Department, and of FP as a whole from the guarantee scheme if: (i) The repairs are carried out by the Service Department and are charged at full cost plus 40%; (ii) The repairs are carried out by the Service department and are charged at marginal cost; (iii)The repairs are carried out by RS. (8 marks) (b) (i) Explain, with reasons, why a ‘full cost plus’ transfer pricing model may not be appropriate for FP. (3 marks) (ii) Comment on other issues that the managers of FP should consider if they decide to allow RS to carry out the repairs. (4 marks) (c) Briefly explain the advantages and disadvantages of structuring the departments as profit centres. (5 marks) (Total = 20 marks)

92 | P a g e

D2 – 3 ZZ group (CIMA P1 Nov 2006) The ZZ Group has two divisions, X and Y. Each division produces only one type of product: X produces a component I and Y produces a finished product (FP). Each FP needs one C. It is the current policy of the group for C to be transferred to Division Y at the marginal cost of £10 per component and that Y must buy all the components it needs from X. The markets for the component and the finished product are competitive and price sensitive. Component C is produced by many other companies but it is thought that the external demand for the next year could increase to 1,000 units more than the sales volume shown in the current budget for Division X. Budgeted data, taken from the ZZ Group Internal Information System, for the divisions for the next year is as follows: Division X Income statement Sales Cost of sales Variable costs Contribution Fixed costs (controllable) Profit

£70,000 £50,000 £20,000 £15,000 £ 5,000

Production/Sales (units) 5,000 (3,000 of which are transferred to Division Y) External demand (units) 3,000 (Only 2,000 of which can be currently satisfied) Capacity (units) 5,000 External market price per unit £20 Balance sheet extract Capital employed

£60,000

Other information Cost of capital charge

10%

Division Y Income statement Sales Cost of sales Variable costs Contribution Fixed costs (controllable) Profit Production/Sales (units) Capacity (units) Market price per unit

£270,000 £114,000 £156,000 £100,000 £ 56,000 3,000 7,000 £90

93 | P a g e

Division Y Balance sheet extract Capital employed

£110,000

Other information Cost of capital charge

10%

Four measures are used to evaluate the performance of the Divisional Managers. Based on the data above, the budgeted performance measures for the two divisions are as follows:

Residual income Return on capital employed Operating profit margin Asset turnover

Division X (£1,000) 8·33% 7·14% 1·17

Division Y £45,000 50·91% 20·74% 2·46

Current policy It is the current policy of the group for C to be transferred to Division Y at the marginal cost of £10 per component and that Y must buy all the components that it needs from X. Proposed policy ZZ Group is thinking of giving the Divisional Managers the freedom to set their own transfer price and to buy the components from external suppliers but there are concerns about problems that could arise by granting such autonomy. Required: (a) If the transfer price of the component is set by the Manager of Division X at the current market price (£20 per component), recalculate the budgeted performance measures for each division. (8 marks) (b) Discuss the changes to the performance measures of the divisions that would arise as a result of altering the transfer price to £20 per component. (6 marks) (c) (i) Explain the problems that could arise for each of the Divisional Managers and for ZZ Group as a whole as a result of giving full autonomy to the Divisional Managers.

94 | P a g e

(ii) Discuss how the problems you have explained could be resolved without resorting to a policy of imposed transfer prices. (6 marks) (Total = 20 marks) D2 – 4 Computer manufacturer (CIMA P1 Nov 2007) A multinational computer manufacturer has a number of autonomous subsidiaries throughout the world. Two of the group’s subsidiaries are in America and Europe. The American subsidiary assembles computers using chips that it purchases from local companies. The European subsidiary manufactures exactly the same chips that are used by the American subsidiary but currently only sells them to numerous external companies throughout Europe. Details of the two subsidiaries are given below. America The American subsidiary buys the chips that it needs from a local supplier. It has negotiated a price of $90 per chip. The production budget shows that 300,000 chips will be needed next year. Europe The chip production subsidiary in Europe has a capacity of 800,000 chips per year. Details of the budget for the forthcoming year are as follows: Sales

600,000 chips $ per chip Selling price 105 Variable costs 60 The fixed costs of the subsidiary at the budgeted output of 600,000 chips are $20 million per year but they would rise to $26 million if output exceeds 625,000 chips. Note: The maximum external demand is 600,000 chips per year and the subsidiary has no other uses for the current spare capacity. Group Directive The Managing Director of the group has reviewed the budgets of the subsidiaries and has decided that in order to improve the profitability of the group the European subsidiary should supply chips to the American subsidiary. She is also thinking of linking the salaries of the subsidiary managers to the performance of their subsidiaries but is unsure which performance measure to use.

95 | P a g e

Two measures that she is considering are “profit” and the “return on assets consumed” (where the annual fixed costs would be used as the “assets consumed”). The Manager of the European subsidiary has offered to supply the chips at a price of $95 each. He has offered this price because it would earn the same contribution per chip that would be earned on external sales (this is after adjusting for increased distribution costs and reduced customer servicing costs). Required: (a) Assume that the 300,000 chips are supplied by the European subsidiary at a transfer price of $95 per chip. Calculate the impact of the profits on each of the subsidiaries and the group. (5 marks) (b) Calculate the minimum unit price at which the European subsidiary would be willing to transfer the 300,000 chips to the American subsidiary if the performance and salary of the Manager of the subsidiary is to be based on (i) the profit of the subsidiary (currently $7 million) (ii) the return on assets consumed by the subsidiary (currently 35%). (9 marks) (c) Write a report to the Managing Director of the group that discusses issues raised by the directive and the introduction of performance measures. (You should use your answers to parts (a) and (b), where appropriate, to illustrate points in your report). (10 marks) (d) Briefly explain how multi-national companies can use transfer pricing to reduce their overall tax charge and the steps that national tax authorities have taken to discourage the manipulation of transfer prices. (6 marks) (Total = 30 marks) D2 – 5 Perfumes and cosmetics (CIMA P2 May 2010) H manufactures perfumes and cosmetics by mixing various ingredients in different processes, before the items are packaged and sold to wholesalers. H uses a divisional structure with each process being regarded as a separate division with its own manager who is set performance targets at the start of each financial year which begins on 1 January. Performance is measured using Return on Investment (ROI) based on net book value of capital equipment at the start of the year. The company depreciates its capital equipment at the rate of 20% per annum on a reducing balance basis.

96 | P a g e

The annual depreciation is calculated at the start of the financial year and one-twelfth of this annual amount is included as monthly depreciation in the fixed overhead costs of each process. Output transferred from one process to another is valued using transfer prices based on the total budgeted costs of the process plus a mark-up of 15%. Process B This is the first process. Raw materials are blended to produce three different outputs, two of which are transferred to Processes C and D respectively. The third output is accounted for as a by-product and sold in the external market without further processing. The equipment used to operate this process originally cost $800,000 on 1 January 2005. The Process B account for April 2010 was as follows: Litres Opening WIP NIL Material W 10,000 Material X 5,000 Material Y 12,000 Direct labour Overhead Profit & Loss Totals 27,000

$ NIL 25,000 10,000 24,000 30,000 75,000 18,800 182,800

Litres Normal Loss 3,000 By-product 5,000 Output to C 9,000 Output to D 10,000 Closing WIP NIL

$ 3,000 5,000 82,800 92,000 NIL

Totals

182,800

27,000

The material costs are variable per unit of input and direct labour costs are fixed in the short term because employees’ contracts provide them with a six month notice period. Overhead costs include a share of Head Office costs, and of the remaining overhead costs some vary with the input volume of the process. The level of activity in April 2010 was typical of the monthly volumes processed by the company. Process C This process receives input from Process B to which is added further materials to produce a finished product that is sold in the external market at the budgeted selling price of $20 per litre. The equipment used to operate this process originally cost $500,000 on 1 January 2008. The Process C account for April 2010 was as follows: Litres Opening WIP 1,000 Input from B 9,000 Material Z 3,000 Direct labour Overhead

$ 11,200 82,800 15,000 20,000 50,000

Totals

179,000

13,000

Normal Loss Abnormal Loss Output Closing WIP

Litres 3,000 1,500 7,500 1,000

$ 1,500 750 150,000 11,200

Profit & Loss Totals

13,000

15,550 179,000

97 | P a g e

The material costs are variable per unit of input and direct labour costs are fixed in the short term because employees’ contracts provide them with a six month notice period. Overhead costs include a share of Head Office costs, and of the remaining overhead costs some vary with the input volume of the process. The level of activity that occurred in April 2010 was typical of the monthly volumes processed by the company, and the opening and closing work in process are identical in every respect. The process is regarded as an investment centre and completed output and losses are valued at their selling prices. The manager of Process C is concerned at the level of output achieved from the input volume and is considering investing in new equipment that should eliminate the abnormal loss. This would involve investing $1,000,000 in new processing equipment on 1 January 2011; the existing equipment would be sold on the same date at a price equal to its net book value. Process D This process receives input from Process B which is further processed to produce a finished product that is sold in the external market at the budgeted selling price of $16 per litre. The equipment used to operate this process originally cost $300,000 on 1 January 2000. The Process D account for April 2010 was as follows: Litres Opening WIP 1,000 Input from B 10,000 Direct labour Overhead Totals 11,000

$ 5,500 92,000 30,000 30,000 157,500

Normal Loss Output Closing WIP Profit & Loss Totals

Litres 1,000 9,000 1,000 11,000

$ 3,000 144,000 5,500 5,000 157,500

Direct labour costs are fixed in the short term because employees’ contracts provide them with a six month notice period. Overhead costs include a share of Head Office costs, and of the remaining overhead costs some vary with the input volume of the process. The level of activity in April 2010 was typical of the monthly volumes processed by the company, and the opening and closing work in process are identical in every respect. The process is regarded as an investment centre and completed output and losses are valued at their selling prices. The manager of Process D believes that the transfer price from Process B is unfair because the equivalent material could be purchased in the open market at a cost of $7.50 per litre. Required: (a) (i) Calculate the annualised Return on Investment (ROI) achieved by each of the process divisions during April 2010. (4 marks)

98 | P a g e

(ii) Discuss the suitability of this performance measure in the context of the data provided for each process division. (4 marks) (b) (i) Calculate the effect on the annualised Return on Investment in 2011 of Process Division C investing in new capital equipment. (4 marks) (ii) Discuss the conflict that may arise between the use of NPV and ROI in this investment decision. (4 marks) (c) Discuss the transfer pricing policy being used by H from the viewpoints of the managers of Process Division B and Process Division D. (9 marks) (Total = 25 marks) D2 – 6 SWZ (CIMA P2 Nov 2010) SWZ is a manufacturing company that has many trading divisions. Return on Investment (ROI) is the main measure of each division’s performance. Each divisional manager’s salary is linked only to their division’s ROI. The following information summarises the financial performance of the S division of SWZ over the last three years: Year ending 31 October Turnover Cost of sales Gross profit Other operating costs Pre-tax operating profit

2008 $000 400 240 160 120 40

2009 $000 400 240 160 104 56

2010 $000 400 240 160 98 62

Capital invested as at the end of the year

400

320

256

Other operating costs include asset depreciation calculated at the rate of 20% per annum on a reducing balance basis. The figures shown in the above table for the capital invested as at the end of the year is the net book value of the division’s fixed assets.

99 | P a g e

All of the above values have been adjusted to remove the effects of inflation. There have been no additions or disposals of fixed assets within the S division during this period. Required: (a) Discuss the performance of the S division over the three year period. (9 marks) The manager of the S division is now considering investing in a replacement machine. The machine that would be replaced would be sold for its net book value which was $40,000 at 31 October 2010 and the new machine would cost $100,000. The new machine would have an expected life of five years and would be depreciated using the same depreciation rates as the existing machinery. The new machine would reduce the division’s cost of sales by 10%. At the end of five years it would be sold for its net book value. The divisional cost of capital is 8% per annum. The company has evaluated the investment and correctly determined that it has a positive Net Present Value (NPV) of $24,536. Required: (b) Prepare calculations to show why the manager of the S division is unlikely to go ahead with the investment. Ignore taxation. (11 marks) (c) Prepare calculations to show how the use of Residual Income (RI) as the performance measure would have led to a goal congruent decision by the manager of the S division in relation to the purchase of the replacement machine. Ignore taxation. (5 marks) (Total = 25 marks) D2 – 7 DE company (CIMA P2 May 2011) The DE Company has two divisions. The following statement shows the performance of each division for the year ended 30 April 2011:

Sales Variable cost Contribution Fixed costs Operating profit

D $000 500,200 380,400 119,800 30,000 89,800

E $000 201,600 140,000 61,600 20,000 41,600 100 | P a g e

Division E manufactures just one type of component. It sells the components to external customers and also to Division D. During the year to 30 April 2011, Division E operated at its full capacity of 140,000 units. The transfer of 70,000 units to Division D satisfied that division’s total demand for that type of component. However the external demand was not satisfied. A further 42,000 components could have been sold to external customers by Division E at the current price of $1,550. The current policy of the DE Company is that internal sales should be transferred at their opportunity cost. Consequently during the year, some components were transferred to Division D at the market price and some were transferred at variable cost. Required: (a) Prepare an analysis of the sales made by Division E that shows clearly, in units and in $, the internal and external sales made during the year. (3 marks) (b) Discuss the effect of possible changes in external demand on the profits of Division E, assuming the current transfer pricing policy continues. (6 marks) Division E is considering investing in new equipment which would reduce its unit variable costs by 20% and increase its capacity by 10% for each of the next five years. The capital cost of the investment is $120m and the equipment would have no value after five years. The DE company and its divisional managers evaluate investments using net present value (NPV) with an 8% cost of capital. External annual demand for the next five years will continue to be 112,000 components at $1,550 each but the DE Company will insist that the internal annual demand for 70,000 components must be satisfied. Required: (c) Assuming that the current transfer pricing policy continues: (i) Evaluate the investment from the perspective of the manager of Division E. (6 marks) (ii) Evaluate the investment from the perspective of the DE Company. (4 marks) Note: Ignore inflation and taxation.

101 | P a g e

(d) Explain TWO factors that should be considered when designing divisional performance measures. (6 marks) (Total = 25 marks)

102 | P a g e

Solutions – Section A Part A Pricing and Product Decisions A1 – 1 EXE (CIMA P2 Pilot Paper 2005) Relevant costs

Steel Brass fittings Skilled labour Semi skilled labour Overheads Estimating time Administration O/H Profit Selling price

Workings W1 W2 W3 W4 W5 W6 W7 W8

$ 55.00 20.00 300.00 Nil 7.50 Nil 382.50 Nil Nil 382.50

Therefore the lowest quote is $382.50 Workings W1 – Steel It is a direct material which is regularly used and therefore we would need to use the replacement cost to value it for the quote. 10m² x $5.50 = $55.00 W2 – Brass fittings We need the brass fittings specifically for the job and therefore should be included. W3 – Skilled labour Skilled labour can either be obtained by asking staff to do overtime or stop production of another product, because there is no skilled labour available. Cost of overtime = 25hrs x $8p/h x 1.5 = $300 Stop existing production = labour costs + lost contribution = (25hrs x $8p/h) + (25hrs x $13p/h) = $525 Choose the cheapest option which would be paying staff overtime, therefore $300.

103 | P a g e

W4 – Semi-skilled labour The cost for semi-skilled labour should be zero as we currently have surplus staff that we can use on this job. W5 – Overheads The overheads are fixed and therefore ignored, but the machine power costs will be incurred as a result of this job, and so therefore we should include these costs. $0.75 x 10hrs = $7.50. W6 – Estimating time The cost of the estimating has already been spent and so should not be included. It is a sunk cost. W7 – Administration overheads These costs are ignored as they are not incurred as result of this job. W8 – Profit The profit mark up should be ignored as we are working out the lowest possible quote for this job. A1 – 2 MNP (CIMA P2 May 2005) Part (a) Joint products are when two or more products are produced from the same process. Shared costs at the split-off point are called common costs. The basis used to apportion the common costs between M, N and P is litres produced. This can be seen by the fact that we are left with the same value per litre of $5.68 when we divide the respective values and litres of products M, N and P. It is fair to use this method as outputs are in litres and the results will be truly reflective of costs apportioned. Costs of the complete process are apportioned between the joint products only (never byproducts) for stock valuation, pricing or profitability purposes. Part (b) Viability means whether or not it is profitable, if it is viable it means that it is profitable and if it is not viable it means that it is not profitable. Sales from M, N and P = ($6.25 x 25,000) + ($5.20 x 15,000) + ($6.80 x 45,000) = $540,250

104 | P a g e

Common costs for M, N and P = $141,875 + $85,125 + $255,375 = $482,375 Profit from M, N and P = £540,250 - $482,375 = $57,875 The common process is viable. To work out the optimal processing plan we need to also consider the extra benefit compared to the extra cost involved in processing M, N and P further. Products M N P

Extra revenue $8.40 - $6.25 = $2.15 $6.45 – $5.20 = $1.25 $7.45 - $6.80 = $$0.65

Extra cost $1.75 $0.95 $0.85

Net benefit $0.40 $0.30 ($0.20)

The optimal processing plan is to make products M and N further as they yield further profits, but not product P as it yields a further loss. Product P should be sold just after the common process. A1 – 3 VBJ (CIMA P2 May 2005) Relevant costs

Coach Fuel Driver Hotel Cost of quote General overheads Total

Workings W1 W2 W3 W4 W5 W6

$ 360 1,500 4,000 or 800 5,000 Nil Nil 10,860 or 7,660

Workings W1 – Coach If we were to go ahead with this contract we would need to obtain a replacement coach to cover existing obligations. If this is ignored then we lose contribution and incur significant penalties on existing obligations. Replacement coach costs = $180 x 2 days = $360 or if we don’t honour our current obligations then we would lose contribution of $250 x 2 days = $500. It is cheaper to hire replacement coach for $360.

105 | P a g e

W2 – Fuel We need to spend $1,500 on fuel costs for this new contract. W3 – Driver We need to hire a replacement driver for the 10 days of the new contract to cover current obligations. Therefore $400 x 10 days = $4,000. An alternativie way of looking at this is that the coach in any event will not be used for 8 days and so therefore ther is no need to hire a driver for 10 days as he will not be used. Instead hire a driver for 2 days when he will be used for the coach. Therefore $400 x 2 days = $800. W4 – Hotel This a specific cost for the new contract of $5,000. W5 – Cost of quote The cost for preparing the quote of $250 has already been spent and so should not be included. W6 – General overheads These are to be ignored as they are not relevant to the quote. A1 – 4 QXY plc (CIMA P2 Nov 2006) Explanation of sales revenue line The sales revenue line shows the amount of sales earned throughout the different level of activities. It can be seen that between zero and somewhere between activity B and C there is a constant rise or straight line increase in sales revenue. This means that the unit price charged is the same and volume sold has been increasing at a constant rate. Beyond the point between B and C sales revenue increases at much slower rate, this indicates that selling price per unit is too high and in order to achieve previous rates of growth there has to be a reduction in unit price. Explanation of the fixed cost line A fixed cost is a cost which cannot be easily identified or related to a cost per unit or activity of any kind e.g. a cost which remains constant when the production of a good or service within the organisation rises or falls. Fixed cost over the long-term will normally display the characteristics of ‘stepped’ cost behaviour. That is the cost remains constant but only within a certain range of production. Once this range of production is exceeded the fixed cost will rise.

106 | P a g e

In the diagram we can see that the fixed cost line is stepped. Between the zero activity and up to activity level B fixed costs are constant. At zero activity fixed costs need to be spent such as machinery and buildings in order to manufacture products. If we were to increase our level of activity beyond level B there needs to be an increase in fixed costs and then the costs are constant up to activity level D. There is another increase in fixed costs at activity D when looking beyond this point and then the costs are constant again. These sudden stepped increases in fixed costs could be due to the factory reaching full capacity and then extra leasehold expenses will need to be incurred in order to obtain more buildings, if production is to increase or expand further. Another example is supervisor’s salaries, they could be paid fixed salaries, but supervision is limited to how many workers that can be supervised. Once the size of the workforce exceeds a certain range another supervisor will need to be employed. Explanation of total cost line Total costs include both fixed costs and variable costs. Variable costs are costs that can be easily identified or related to a cost per unit or activity level of some kind e.g. a cost which rises or falls directly with the production/provision of a good or service within an organisation. Examples could include labour piece work schemes e.g. a factory worker that gets paid for each unit they make or the cost of material/components for the production or assembly of a product. All variable cost starts from the origin of the graph indicating the cost is nil if the activity level is zero. Variable cost does not necessarily behave in a linear manner e.g. a constant amount incurred for each unit of activity. It can behave in a curvilinear (non-linear) manner as well, in which case the variable cost line would be curved not straight. In the diagram it can be seen that at activity zero total cost is equal to fixed costs. At this point there are no variable costs as there is no activity only fixed costs. Between activity levels zero and B we can see that total cost line is increasing at constant level. The constant increase is due to variable costs being incurred as a result of increasing activity. Total costs increases in a stepped fashion at activity levels B and D because of the costs behaviour of fixed costs as mentioned previously. It can be seen that variable costs are increasing at the same constant rate within total costs up to activity level D, after this point it can be seen that the total costs line increases more steeply. This is due to increased variable costs per unit.

107 | P a g e

Issues to consider when making decisions At activity level A it can be seen from the diagram the sales revenue line intersects the total cost line indicating that this is the point when the company makes no loss or profit i.e. breakeven. Any activity beyond this point sales revenue will exceed total costs causing the company to make a profit, and anything below this activity, total costs will exceed sales revenue causing the company to make losses. After activity level B fixed costs will increases sharply because of perhaps new investment required in the manufacturing process and profits will be reduced compared to just before activity level B. The operational mangers needs to consider whether the sales revenue forecast is likely to hold true, if not then profits can be reduced significantly as a result of this investment. Between activity levels B and C the sales revenue line has a much higher gradient line than total costs and the company is earning greater profits as it increases its activity. Profits are maximised just before point C when beyond this point the sales revenue line is increasing at a slower rate when compared to total costs. At activity level D there is another sharp increase in fixed costs and also variable costs are rising at steeper gradient to sales revenue. The operational manger should recommend to the company to continue to produce activity as long as the extra revenue is greater than the extra cost or variable cost. A1 – 5 RST (CIMA P2 May 2007) Part (a) We need to work out the extra benefit of making RZ, SZ and TZ and then compare these to the extra cost involved. We should make those which give a net extra benefit. Product

Extra benefit

RZ SZ TZ

$6.00 - $3.00 = $3.00 $5.75 - $5.00 = $0.75 $6.75 - $3.50 = $3.25

Extra variable cost $1.40 $0.90 £1.00

Extra fixed cost

Net

Nil Nil $600 / 1200 kg = $0.50

$1.60 ($0.15) $1.75

Products R and T should be further processed to produce products RZ and TZ respectively as they provide and extra net benefit of $1.60 and $1.75 per kg respectively. We have assumed that future production levels of product TZ would be based on current levels and so therefore the extra fixed costs per unit is based on 1,200 kg. Product S should not be further processed to make product SZ as there is net cost of $0.15 per kg every time an SZ is produced. 108 | P a g e

These recommendations are based purely on financial grounds and the company should also look at qualitative factors as well before making their final decisions. Part (b) (i) Product R S T

Selling price $3 $5 $3.50

Kg 800 2,000 1,200

Total sales value $2,400 $10,000 $4,200 $16,600

The common costs = $17,500. Therefore there would be a net loss of $900 if the products R, S and T were sold to the external market. It is not financially viable. Part (b) (ii) If there is no external market for R, S and T then we must further process all these products to produce RZ, SZ and TZ which can be sold. Product RZ SZ TZ

Net benefit per kg $1.60 ($0.15) $1.75

Kg 800 2,000 1,200

Loss from the common process Net benefit

Total ($) 1,280 (300) 2,100 3,080 (900) 2,180

After further processing the products it now is financially viable as there is a net benefit of $2,180. A1 – 6 HS (CIMA P2 Nov 2007) Part (a) 1. Determine the price function or demand function The price or demand function formula is: P = a - bq P = Price a = Price at which demand would be zero (i.e. the “p” when q=0) b = The gradient of the demand curve q = Quantity sold at that price (P) 109 | P a g e

P = 1,350 q = 8,000 b = 50 /100 = 0.05 a=? In order to determine the price function we need to first find the value of “a”. Substitute all known values into the price function formula to determine “a”. 1,350 = a – 0.05 (8,000) 1,350 = a – 400 1,350 + 400 = a 1,750 = a Now we can construct the price function: P = 1,750 – 0.05q 2. Determine the marginal revenue function (MR) The MR function is the price function itself, but it will have twice the value of whatever the ‘b’ value is, within the price function. Therefore: MR = 1,750 – 2(0.05)q MR = 1,750 – 0.1q 3. Determine the marginal cost function (MC) Marginal cost is $270 being the direct material cost; however this does not include any variable costs from labour and conversion costs. We need to use the high low method to work this out.

High Low Difference

Units 9,400 7,300 2,100

$’000 7,000 5,446 1,554

Variable cost = 1,554,000 / 2,100 = $740 per unit Therefore total marginal cost = $270 + $740 = $1,010

110 | P a g e

4. Equate MR = MC to obtain the units sold to maximise profits MR = MC 1,750 – 0.1q = 1,010 -740 = -0.1q 740 = 0.1q 7,400 = q Therefore we sell 7,400 units when we maximise our profits. 5. Use the price function to determine the selling price that would maximise profits We can substitute in 7,400 units into the price function we created before to determine the selling price that will maximise profits. P = 1,750 – 0.05q P = 1,750 – 0.05 (7,400) P = 1,750 -370 P = 1,380 The selling price that will maximise profits is $1,380. Part (b) Exam tip: Please note that the question only asks for two reasons however we have provided a selection of possible reasons. Any two would suffice. The pricing model assumes that there is a linear or straight line relationship between price and demand in reality this may not be true. It is difficult to determine the price at which demand would be zero in other the maximum price that can be charged. The theory assumes that price is the only factor which will influence or change demand but in reality there are other factors such fashion, the availability of substitutes and the cost of complimentary products. It is difficult to ascertain the variable costs accurately without detailed knowledge of cost curves this may not be available as HS is a new company.

111 | P a g e

A1 – 7 Bank charges (CIMA P2 May 2008) Part (a) (i) Existing bank account structure The amount of business customers will fall by 20% if no action is taken. Therefore the number of customers going forward = 1,000,000 x 80% = 800,000 Number of transactions per customer per year = 800,000 x 1,000 = 800,000,000 Transaction charges = £0.60 x 800,000,000 = £480,000,000 Number of customers with a positive balance = 45% of 800,000 = 360,000 Interest paid to positive balance customers = 0.1% of £2,000 x 360,000 = £720,000 Number of customers with a negative balance = 55% of 800,000 = 440,000 Interest received from negative balance customers = 20% of £4,000 x 440,000 = £352,000,000 Investment income is 3% of net position of customer balances = 3% of ((440,000 x £4,000) – (360,000 x £2,000)) = £31,200,000 Net income for the bank = £480,000,000 - £720,000 + £352,000,000 + £31,200,000 = £862,480,000 Part (a) (ii) Option 1 The amount of business customers will increase by 5%. Therefore the number of customers going forward = 1,000,000 x 1.05 = 1,050,000 Monthly fixed charges = £10 x 12mths x 1,050,000 = £126,000,000 Number of customers with a positive balance = 45% of 1,050,000 = 472,500 Interest paid to positive balance customers = 0.5% of £2,000 x 472,500 = £4,725,000 Number of customers with a negative balance = 55% of 1,050,000 = 577,500 Interest received from negative balance customers = 20% of £4,000 x 577,500 = £462,000,000 Investment income is 3% of net position of customer balances = 3% of ((577,500 x £4,000) – (472,500 x £2,000)) = £40,950,000 Net income for the bank = £126,000,000 - £4,725,000 + £462,000,000 + £40,950,000 = £624,225,000 112 | P a g e

Option 2 The amount of business customers will increase by 1%. Therefore the number of customers going forward = 1,000,000 x 1.10 = 1,100,000 Number of customers with a positive balance = 45% of 1,100,000 = 495,000 Interest paid to positive balance customers is nil Number of customers with a negative balance = 55% of 1,100,000 = 605,000 Interest received from negative balance customers = 20% of £4,000 x 605,000 = £484,000,000 Investment income is 3% of net position of customer balances = 3% of ((605,000 x £4,000) – (495,000 x £2,000)) = £40,900,000 Net income for the bank = £484,000,000 + £40,900,000 = £524,900,000 Conclusion The recommended course of action should be to continue with the excising bank account structure. A1 – 8 WX (CIMA P2 May 2011) Part (a) (i) The direct material and labour costs are completely variable. This can be determined by dividing the combined costs of labour and material costs by the respective activity level in the forecast, which will result in the same cost per unit at all activity levels. For example at activity level 100,000 units material and labour costs added together are $800,000, and therefore cost per unit = $8 per unit. This is the same rate at activity levels 160,000 and 200,000 units if you compare them with their respective combined material and labour costs. Overhead costs however are not completely variable and must be analysed between fixed overheads and variable overheads. We need to use the “high-low method” to find the variable overheads. Units 200,000 100,000 100,000

Overhead cost ($) 1,460,000 800,000 580,000 113 | P a g e

$580,000 / 100,000 = $5.80. Total variable cost per unit = $8 + $5.80 = $13.80. Part (a) (ii) 1. Determine the price function or demand function The price or demand function formula is: P = a - bx P = Price a = Price at which demand would be zero (i.e. the “p” when x=0) b = The gradient of the demand curve x = Quantity sold at that price (P) P = 25 b = 1 /25,000 = 0.00004 x = 150,000 a=? In order to determine the price function we need to first find the value of “a”. Substitute all known values into the price function formula to determine “a”. 25 = a – 0.00004 (150,000) 25 = a – 6 25 + 6 = a 31 = a Now we can construct the price function: P = 31 – 0.00004x 2. Determine the marginal revenue function (MR) The MR function is the price function itself, but it will have twice the value of whatever the ‘b’ value is, within the price function. Therefore: MR = 31 – 2(0.00004)x MR = 31 – 0.00008x 3. Determine the marginal cost function (MC) Marginal cost is given in the question as $13.80.

114 | P a g e

4. Equate MR = MC to obtain the units sold to maximise profits MR = MC 31 – 0.00008x =13.80 -0.00008x = 13.80 - 31 -0.00008x = -17.20 x = -17.20 / -0.00008 x = 215,000 Therefore we sell 215,000 units when we maximise our profits. 5. Use the price function to determine the selling price that would maximise profits We can substitute in 215,000 units into the price function we created before to determine the selling price that will maximise profits. P = 31 – 0.00004x P = 31 – 0.00004 (215,000) P = 31 – 8.6 P = 22.4 The selling price that will maximise profits is $22.40. Part (b) Exam tip: Please note that the question only asks for two reasons however we have provided a selection of possible reasons. Any two would suffice. The optimum selling process is derived from the price function or demand function. The main problem with using the price function is that the quality of the market research to determine the demand function has to be very good, for it to have any real value; otherwise it may give unrealistic predictions. It also assumes that price and quantity are the only factors in determining demand, but we must be mindful of other factors which are just as important such as quality, advertising, the availability of substitutes, brand loyalty, fashion, and the cost of complimentary products. It is difficult to estimate the demand curve. The price function assumes that there is a linear or straight line relationship between price and demand in reality this may not be true. It is difficult to determine the price at which demand would be zero in other words the maximum price that can be charged.

115 | P a g e

It is difficult to ascertain the variable costs accurately without detailed knowledge of cost curves this may not be available.

116 | P a g e

Solutions – Section B Part A Pricing and Product Decisions A2 -1 TQ (CIMA P2 Pilot paper 2005) Part (a) (i) In order to work out the price when profits are maximised we need to construct the demand function, from this we can derive the marginal revenue (MR) curve and equate this to the marginal cost curve (MC) to obtain the profit max position. MR=MC. We are given in the question that when x = 0 then P = $100 which is the maximum price. If this is the case then we can work out “a” by using the demand function formula. Therefore: P = a – bx 100 = a – b(0) 100 = a – 0 100 = a We need to obtain another set of price and quantity to obtain the gradient or “b” value of P = a – bx, as the above set does not enable us to find it. We are told that the selling price is $60 but not the quantity sold, but we are given fixed overhead volume variances at this price. We can use these to obtain a quantity for this price. Fixed overhead volume variance = (Actual production – Budgeted production) x F/OH per unit For each period we shall work out the actual units produced using the above formula. This is to determine whether we have different actual units for each period and therefore we would have a different demand function for each period as well. Period 1 -1200 = (Actual production – 520) x 10 -1,200 / 10 = Actual production – 520 -120 + 520 = Actual production 400 = Actual production

117 | P a g e

Period 2 -1900 = (Actual production – 590) x 10 -1900 / 10 = Actual production – 590 -190 + 590 = Actual production 400 = Actual production Period 3 -2600 = (Actual production – 660) x 10 -2600 / 10 = Actual production – 660 -260 + 660 = Actual production 400 = Actual production We have the same actual units in each period of 400 and at price $60, this is quantity we will sell. We can now work out “b”. P = a – bx 60 = 100 – b (400) 60 - 100 = -b (400) -40 / 400 = -b -0.1 = -b 0.1 = b Therefore the demand function is P = 100 – 0.1x MR = 100 – 2(0.1)x MR = 100 – 0.2x MC = budgeted variable costs = $25 Therefore profit max is when MR = MC 100 – 0.2x = 25 -0.2x = 25 – 100 -0.2x = -75 x = -75 / -0.2 x = 375 The quantity we would sell to maximize profits is 375 units. The price for this quantity is P = 100 – 0.1 (375) = $62.50

118 | P a g e

Part (a) (ii) Actual contribution in period 3 400 units x ($60 per unit - $25 per unit) = $14,000 We would have earned if the optimal price was used in period 3 375 units x ($62.50 per unit - $25 per unit) = $14,062.50 Difference = $62.50 Part (b) This question had at least one mark for writing your answer in a report format!

REPORT To: From: Subject: Date:

Board of Directors of TQ Management Accountant Alternative pricing strategies 19th September 2003

1. Introduction This report is designed to discuss the alternative pricing strategies available to TQ. 2. Market skimming TQ can initially charge a high price, achieve low volume but earn a larger profit per unit sold. This is known as market skimming. It is a strategy which exploits a price insensitive market or an inelastic demand for a product. TQ is selling a third generation mobile phone, it is a high technology product and as a result there maybe an expectation by customers to pay a premium for these products. TQ’s third generation phone is the first of its kind and customers will want to be the first to enjoy it. TQ could also use this approach as it is good for innovative high quality products where little competition exists initially. Therefore a premium can be charged without reducing demand for it too much. There is always a choice of lowering the price later on when competition comes it to the market with cheaper substitutes. The market skimming approach also allows TQ to recover the significant levels of development costs it had to spend on developing this phone and the fact that they have a very short life cycle. 119 | P a g e

3. Market penetration TQ can charge a very low price in order to capture a larger market share quicker, this is known as market penetration. This approach will aim to achieve a wide customer base as quickly as possible by setting the price very low. It means that because if its affordability a lot of people will be able to purchase it, as a result the first 2 stages are quickly achieved in the product life cycle and also achieve economies of scale far quicker. It will also be more difficult for other companies to enter the market as they would have to compete on this low price and may struggle to make a profit. 4. Demand based approach TQ could create a demand function much like that created from the second generation phones in part (a). This would represent a straight line relationship between price charged and quantity sold (P = a - bx) of the third generation phones, and allow us to find that price that we can charge that would maximise profits (MR = MC). The main problem with this is that the quality of the market research to determine the demand function has to be very good, for it to have any real value. It also assumes that price and quantity are the only factors in determining demand, but TQ must be mindful of other factors which are just as important such as quality, advertising, substitutes and brand loyalty. It is difficult to estimate the demand curve. The demand function approach does give us a useful insight into the relationship between price and quantity and a basis to predict what strategy to take. It makes an attempt to incorporate demand into its calculations and is concerned with the marginal costs of making decision ignoring irrelevant costs. The demand function also allows us to see at what price and quantity we would maximise profits and we can predict quantity sold given any selling price and vice versa. I hope you have found this report useful but should you require any further assistance or have any questions please do not hesitate to contact me. Signed : Management Accountant.

120 | P a g e

A2 – 2 QP plc (CIMA P2 Nov 2005) Part (a) The theory of constraints is where a bottleneck exists (or limiting factor in production); this therefore is a constraint on throughput. Throughput is the rate of production. A manager’s aim in this situation would be to operate the bottleneck resource at 100% capacity, whilst running non-bottleneck resources at a speed that matches this (which may not be at 100%). This would be efficient use of resources, but also would avoid the pile up of unnecessary work-in-progress in a JIT environment. Throughput accounting aims to maximise contribution whilst minimising conversion (labour and overhead) cost. Throughput contribution = Sales - material cost It is assumed only materials and components are the variable cost. Conversion cost = all other fixed cost Return per factory hour is similar to the concept of contribution maximisation; you should notice the following calculation is similar to the contribution per unit of a limiting factor used in short-term decision-making. Return per factory hour =

TA ratio =

Sales less material cost Usage (in hours) of the bottleneck resource

Contribution (sales less material) per hour Conversion cost per hour (or cost per factory hour)

The above TA ratio can be used to assess the manager in terms of how well they have done maximising contribution, whilst eliminating the build up of stock. Such a technique is similar to limiting factor analysis, as managers try to improve their TA Ratio they must concentrate on producing those products that generate the highest contribution per bottleneck resource (or limiting factor) in order to optimise contribution and therefore profit. Traditionally when absorption costing was used, managers could improve results by setting production levels higher than sales (therefore the carry forward of fixed overhead this period to the next in the valuation of closing stock), however stock piling is not a good thing and such stock piling will not improve performance if you use the TA Ratio.

121 | P a g e

Part (b) We need to first identify the limiting factor and from the question it would seem it is only ingredient L. Ingredient M may be limited but it has a substitute being V which has unlimited supply. Having identified the limiting factor we must now work out the contribution we earn from each of the meals, then the contribution per kg of L, and then rank in order of production.

Selling price Ingredient K Ingredient L Ingredient M Throughput contribution Kg of L used Contribution per kg of L Rank in order of production

TR £ 340 (150) (70) (30) 90

PN £ 450 (120) (90) (75) 165

BE £ 270 (90) (40) (45) 95

7kg

9kg

4kg

£90/7kg = £12.86

£165/9kg = £18.33

£95/4kg = £23.75

3

2

1

Total ingredient L available is 7,000kg, however we must make a minimum order first of 50 batches for each type of meal. The ingredients used for this must be deducted before working out the optimum production plan.

TR PN BE Total

Minimum contract 50 50 50

Kg of L per batch 7 9 4

Ingredient L used 350kg 450kg 200kg 1,000kg

Amount of ingredient L available after the minimum contract is 6,000kg.

BE PN TR

300 batches x 4kg 350 batches x 9kg 6,000kg – 1,200kg – 3,150kg = 1,650kg

1,650kg / 7kg = 236 batches

L used 1,200 3,150 1,650 6,000

Optimum production plan

PN BE TR

Minimum contract 50 50 50

Production 350 300 236

Total 400 350 286 122 | P a g e

Part (c) We need to interpret the linear programming solution that the computer has produced. Objective function This is £110,714 and is the maximum contribution that can be earned given the constraints. TR, PN and BE values These are the amounts we should produce in order to maximise contribution. Therefore we should make 500 batches of TR, 357 batches of PN and 71 batches of BE. TR slack value This is zero and represents the fact that we have produced up to the maximum demand of TR, this being 500 batches. There is no shortfall in demand. PN and BE slack values These values represent the fact that we have not met our maximum demands for PN and BE by 43 batches and 279 batches respectively. We are only producing 357 PN’s and 71 BE’s where our maximum demand is 400 and 350 batches respectively. L and M values These values represent shadow prices for ingredients L and M as they are both scarce resources. Ingredient M is now also scarce because it says in the question that substitute ingredient V is poisonous and therefore cannot be used. The values show that for each kg of scarce resource we obtain at normal cost we can expect to earn £3 of contribution for each kg of L and £28 contribution for each kg of M. Ingredient K does not have a shadow price because it is not a scarce resource; we have enough of K to produce our batches.

123 | P a g e

A2 – 3 ZP plc (CIMA P2 Nov 2005) Part (a)

Consultants salary Travel Accommodation

Workings W1 W2 W3

D 40,000 4,000 nil 44,000

E 140,000 7,000 8,000 155,000

F 60,000 4,000 3,000 67,000

Total 240,000 15,000 11,000 266,000

Workings W1 - Consultants salary are direct costs for each client group, driven by chargeable hours. Total cost = 4 x £60,000 = £240,000 Chargeable hours = (100 x 10) + (700 x 5) + (300 x 5) = 6,000 hours Rate per hour = £240,000 / 6,000 hours = £40 per hour Cost for D = £40 x 100 x 10 = £40,000 Cost for E = £40 x 700 x 5 = £140,000 Cost for F = £40 x 300 x 5 = £60,000 W2 – Travel cost driven by number of miles. Number of total miles = (50 x 3 x 10) + (70 x 8 x 5) + (100 x 3 x 5) = 5,800 miles Cost per mile = £15,000 / 5,800 miles = £2.59 per mile Cost for D = £2.59 x 50 x 3 x 10 = £ 3,885 Cost for E = £2.59 x 70 x 8 x 5 = £7,252 Cost for F = £2.59 x 100 x 3 x 5 = £3,885 W3 – Accommodation costs are driven by visits to clients in excess of 50 miles. Therefore not applicable to client D as distance to their clients is less than 50 miles. Number of qualifying visits = (8 x 5) + (3 x 5) = 55 Cost per visit = £11,000 / 55 = £200 Cost for E = £200 x 8 x 5 = £8,000 Cost for F = £200 x 3 x 5 = £3,000

124 | P a g e

Other costs Office premises Advertising Telephone Support staff

£ 50,000 5,000 10,000 120,000 185,000

There is no obvious activity which directly drives these costs and therefore we must leave them out of our calculations as they would not add any further benefit if they were included. Part (b) Comparing the current system with the proposed ABC system

Chargeable hours Current system (£75p/hr) ABC Difference

D 1,000 £ 75,000 44,000 31,000

E 3,500 £ 263,000 155,000 108,000

F 1,500 £ 113,000 67,000 46,000

Total 6,000 £ 451,000 266,000 185,000

The difference is due to the unattributable costs of £185,000 as shown in part (a). We need to obtain a cost driver for these other costs to make the above comparison more meaningful. We need to look at whether the same or similar figures can be arrived at by doing a rate per chargeable hour on the total costs for ABC. Blanket rate on chargeable hours = £266,000 / 6,000 hrs = £44.33p/hr

Chargeable hours Blanket (£44.33p/hr) ABC Difference

D 1,000 £ 44,333 44,000 333

E 3,500 £ 155,155 155,000 155

F 1,500 £ 66,500 67,000 (500)

Total 6,000 £ 265,988 266,000 (12)

It seems that ABC does not add much more value than the current system, as the current system gives similar figures. In conclusion ABC is theoretically superior, but in this case perhaps not appropriate unless further costs drivers reveal different figures.

125 | P a g e

Part (c) Unit level costs These are driven by the quantity of items produced. The more produced the lower the unit cost. Example: Consultants produce chargeable hours in exchange for their salary at work. We can work out a unit cost per chargeable hour. Cost per hour = £240,000 / 6,000 hours = £40 per hour Batch level costs These are costs incurred for every time a batch is processed for example administration costs maybe incurred when a batch of mugs are made in a factory. Examples: The cost incurred in visiting every client once. The cost incurred in billing every client. The cost incurred in chasing clients for outstanding monies. The cost incurred in negotiating the fees on a contract with a client. Product sustaining These are costs needed to continue the product in the future, for example client care costs. Examples: The cost to employ administrative staff to organise, plan for clients service requirements. The cost to entertain clients such as buying them lunch, evening meals and drinks or even tickets to see sports events. Facility sustaining These are fixed costs spent for the use of the company as a whole, for example warehouse or machinery costs. Examples: Telephone systems, fax machines, photocopiers and the canteen.

126 | P a g e

A2 – 4 AVX plc (CIMA P2 May 2006) Part (a) (i) (ii) November 1 batch made which took 50 hours which took as long as the standard and therefore no learning effect as the labour efficiency variance was nil. December 1 batch was made which cost £500 – £170 = £330 Actual spend of standard cost is £330 / £500 x 100% = 66% Actual hours taken = 66% x 50hrs = 33hrs Average hours per batch = (50hrs +33hrs) / 2 = 41.5hrs Learning curve effect = 41.5 hrs / 50hrs = 83% January 2 batches were made which cost £1,000 – £452.20 = £547.80 Actual spend of standard cost is £547.80 / £1,000 x 100% = 54.78% Actual hours taken = 54.78% x 100hrs = 54.78hrs Average hours per batch taken to date = (50hrs + 33hrs + 54.78hrs) / 4 = 34.45hrs Learning curve effect = 34.45hrs / 41.5hrs = 83% February 4 batches were made which cost £2,000 – £1,089.30 = £910.70 Actual spend of standard cost is £910.70 / £2,000 x 100% = 45.53% Actual hours taken = 45.53% x 200hrs = 91.06hrs Average hours per batch taken to date = (50hrs + 33hrs + 54.78hrs + 91.06hrs) / 8 = 28.61hrs Learning curve effect = 28.61hrs / 34.45hrs = 83%

127 | P a g e

March 8 batches were made which cost £4,000 – £1,711.50 = £2,288.50 Actual spend of standard cost is £2,288.50 / £4,000 x 100% = 57.21% Actual hours taken = 57.21% x 400hrs = 228.84hrs Average hours per batch taken to date = (50hrs + 33hrs + 54.78hrs + 91.06hrs + 228.84hrs) / 16 = 28.61hrs Learning curve effect = 28.61hrs / 28.61hrs = 100% April 16 batches were made which cost £8,000 – £3,423 = £4,577 Actual spend of standard cost is (£4,577 / £8,000) x 100% = 57.21% Actual hours taken = 57.21% x 800hrs = 457.68hrs Average hours per batch taken to date = (50hrs + 33hrs + 54.78hrs + 91.06hrs + 228.84hrs + 457.68hrs) / 32 = 28.61hrs Learning curve effect = 28.61hrs / 28.61hrs = 100% The learning curve is 83% and ceases in March, when batches take 28.61 hours to make on average. The implications are that there are no further labour efficiencies to be had after the curve ceases and selling price and costs should be based on a batch taking 28.61 hrs. Part (b) Note: If Price (P) = a – bx then Marginal Revenue (MR) = a – 2bx 1. Determine the price function or demand function The price or demand function formula is: P = a - bQ P = Price a = Price at which demand would be zero (i.e. the “p” when Q=0) b = The gradient of the demand curve Q = Quantity sold at that price (P)

128 | P a g e

P = 1,200 b = 20 Q = 16 a=? In order to determine the price function we need to first find the value of “a”. Substitute all known values into the price function formula to determine “a”. 1,200 = a – 20 (16) 1,200 = a – 320 1,200 + 320 = a 1,520 = a Now we can construct the price function: P = 1,520 – 20Q 2. Determine the marginal revenue function (MR) The MR function is the price function itself, but it will have twice the value of whatever the ‘b’ value itself is, within the price function. Therefore: MR = 1,520 – 2 (20) Q MR = 1,520 - 40Q 3. Determine the marginal cost function (MC) Marginal cost is given in the question as £672.72. 4. Equate MR = MC to obtain the units sold to maximise profits MR = MC 1,520 – 40Q = 672.72 -40Q = 672.72 – 1,520 -40Q = -847.28 Q = 21.182 Therefore we sell 21.182 units when we maximise our profits. 5. Use the price function to determine the selling price that would maximise profits We can substitute in 21.182 units into the price function we created before to determine the selling price that will maximise profits.

129 | P a g e

P = 1,520 – 20Q P = 1,520 – 20 (21.182) P = 1,520 – 423.64 P = 1,096.36 The selling price that will maximise profits is £1,096.36. Part (c) (i) Standard costing A standard cost is a planned or forecast unit cost for a product or service, which is assumed to hold good given expected efficiency and cost levels within an organisation. It represents a target cost and is useful for planning, controlling and motivating within an organisation. Under a standard costing system an organisation can value stock at standard cost, incorporating this within the ledger or cost accounts of the organisation, the budget or forecasts being a memorandum kept outside the ledger accounts. Target costing Market price to achieve desired market share

XX

TARGET COST (Balance)

(XX)

Desired profit

XX

Used by Nissan, Sony and Toyota and many other Japanese companies, who sought not what a product ‘does’ cost (which is what most UK companies used as the method of pricing) but rather what it ‘should’ cost. The idea is that the product price is determined by the market place, a desired level profit is decided upon and the balance is the costs to manufacture the products through improved processes to be able to reduce costs to sell at the market price. Target costing seeks to continually improve the manufacturing process by the use of JIT, TQM, cost reduction, value analysis and benchmarking. Standard costing develops a product, determines its cost and adds a mark up to determine a price. This was very much an internal process ignoring competition or demand. Target costing starts with the external market price and works backwards to deicide on levels of cost and profit.

130 | P a g e

Part (c) (ii) AVX Plc sells the CB45 circuit board which is a high technology product and has been able to charge a premium price for these products. AVX Plc has used this approach and has enjoyed good product volume as little competition exists initially, however it seems that competitors have now managed to create a close substitute to this circuit board and so customers are choosing the cheaper alternative. This indicates that AVX Plc must look to reducing the price of the circuit board in order to sustain the volume of sales they expect, and use economies of scale to reduce costs. If a price reduction is not made then we could find continuation of falling volumes sold and the CB45 falling out of favour with customers who have bought the cheaper alternative. It also shows that these circuit boards have a short lifecycle as in six months it seems that the CB45 is already into its maturity stage and perhaps heading towards decline. AVX Plc can no longer set its own price and must accept the market price. If AVX Plc wanted to sustain the same price for these circuit boards then they to be modified to provide additional functions that customers value which the competitor’s circuit boards do not provide. A2 – 5 GHK (CIMA P2 May 2006) Part (a) Workings W1 – Direct Material A We know that they used $5 per kg in the budgets. This is in correct as they should have used $7 per kg as the material is in regular use and is not scarce. Material A cost percentage change = ($7 per kg / $5 per kg) x 100% = 140% Therefore increase the budgets by 140% to obtain a basis on $7 per kg, and then divide by the number of units budgeted for to obtain a cost per unit. Product G H J K

Relevant cost $9,000 x 140% = $12,600 $12,000 x 140% = $16,800 $4,500 x 140% = $6,300 $18,000 x 140% = $25,200

Budgeted units 3,000 3,000 3,000 3,000

Cost per unit $12,600 / 3,000 = $4.20 $16,800 / 3,000 = $5.60 $6,300 / 3,000 = $2.10 $25,200 / 3,000 = $8.40

131 | P a g e

W2 – Direct Material B This is used when required and the cost of $10 per kg is used in the budgets. Therefore on a per unit basis: G: $6,000 / 3,000 units = $2 H: $6,000 / 3,000 units = $2 J: $13,500 / 3,000 units = $4.50 K: $36,000 / 3,000 units = $12 W3 – Direct Labour This is used when required and the cost of $10 per hour is used in the budgets. Therefore on a per unit basis: G: $6,000 / 3,000 units = $2 H: $24,000 / 3,000 units = $8 J: $22,500 / 3,000 units = $7.50 K: $9,000 / 3,000 units = $3 W4 – Overhead We need to look at the change in overhead costs between different levels of units. This will be our relevant cost for overheads. 3,000 units Product G Product H Product J Product K

$6,000 $13,000 $11,000 $11,000

5,000 units $8,000 $19,000 $17,000 $17,000

Product Selling price Direct Material A (W1) Direct Material B (W2) Direct Labour (W3) Overhead (W4) Contribution Contribution per $ of Material B

Variable overheads $2,000 $6,000 $6,000 $6,000

G $ 10 (4.20) (2) (2) (1) 0.80 0.80 / 2 = 0.40

Variable overheads per unit $2,000 / 2,000 units = $1 $6,000 / 2,000 units = $3 $6,000 / 2,000 units = $3 $6,000 / 2,000 units = $3

H $ 20 (5.60) (2) (8) (3) 1.40 1.40 / 2 = 0.70

J $ 15 (2.10) (4.50) (7.50) (3) (2.10) (2.10) / 4.50 = 0.47

K $ 30 (8.40) (12) (3) (3) 3.60 3.60 / 12 = 0.30

132 | P a g e

Part (b) We need to work out how much extra contribution we can make if we did not supply the major customer. All resources are available except material B. Let’s find out how much of the scarce resource material B will be released if the major customer was not supplied. Product

Units in contract

G H J K

500 1,600 800 400

Material B cost per unit $2 $2 $4.50 $12

Material B cost per kg $10 $10 $10 $10

Material B per unit in kg $2 / $10 = 0.2 $2 / $10 = 0.2 $4.50 / $10 = 0.45 $12 / $10 = 1.2

Total kg 500 x 0.2 = 100 1,600 x 0.2 = 320 800 x 0.45 = 360 400 x 1.2 = 480

Total material B released = 100 + 320 + 360 + 480 = 1,260 kg Our optimum production plan as described in the requirement is greater than normal customer demand for products G and H, but for product J and K they are lower. We should not use the released material B to produce any more of G or H as there is no further demand for them nor should we look to produce J as it has a negative contribution. Therefore we should make product K with the released material B. We should only produce another 4,000 – 2,017 = 1,983 units of K as demand would be satisfied then. With 1,260 kg of material B we can make another 1,260 kg / 1.20 kg per unit = 1,050 units of K. The extra contribution from K = 1,050 units x $3.60 per unit = $3,780. As a result of stopping the contract to the major customer it means that we will be stopping the production of J which is loss making, and therefore we will be saving on it’s the specific fixed costs of $1,000. Therefore the extra contribution will be from K = £4,780. Product G H J K Total

Units 500 1,600 800 400

Contribution per unit $0.80 $1.40 ($2.10) $3.60

Total contribution $400 $2,240 ($1,680) $1,440 $2,400

More contribution is earned if the contract is ignored. The extra amount of contribution that is earned is $4,780 - $2,400 = $2,380. GHK would be indifferent meeting the contract or paying the penalty if the penalty was $2,380.

133 | P a g e

Part (c) Product G H J K

Selling Price per unit $10 $20 $15 $30

Contribution per unit $0.80 $1.40 ($2.10) $3.60

C/S ratio 0.80/10 = 0.08 1.40/20 = 0.07 (2.10)/15 = 0.14 3.60/30 = 0.21

Part (d) We need to see how fixed costs are covered by the sales of each of these products first, if we are to sketch a profit volume chart. Specific fixed costs can be avoided by not undertaking a product but non-specific or general fixed costs are unavoidable. We need to work out how much non-specific fixed costs we have. Workings Product G – for 5,000 units Overhead costs = $8,000 We know that the variable element of this is $1 per unit x 5,000 units = $5,000 We also know that product specific fixed cost is $1,000 Therefore the non-specific fixed cost = $8,000 - $5,000 - $1,000 = $2,000 Product H – for 5,000 units Overhead costs = $19,000 We know that the variable element of this is $3 per unit x 5,000 units = $15,000 We also know that product specific fixed cost is $1,000 Therefore the non-specific fixed cost = $19,000 - $15,000 - $1,000 = $3,000 Product J – for 5,000 units Overhead costs = $17,000 We know that the variable element of this is $3 per unit x 5,000 units = $15,000 We also know that product specific fixed cost is $1,000 Therefore the non-specific fixed cost = $17,000 - $15,000 - $1,000 = $1,000 Product K – for 5,000 units Overhead costs = $17,000 We know that the variable element of this is $3 per unit x 5,000 units = $15,000 We also know that product specific fixed cost is $1,000 Therefore the non-specific fixed cost = $17,000 - $15,000 - $1,000 = $1,000

134 | P a g e

Product G H J K Specific fixed costs for all four products Total

$ 2,000 3,000 1,000 1,000 7,000 4,000 11,000

We need to now introduce each of the products and noting the cumulative profit or loss figure. We would obviously make the product which gives the most contribution first. Remember the requirement says that we have no longer any restrictions on resources and make as many as we desire, and we should make up to the demand we have which would of course include the one off contact to a major customer. Cumulative Profit

Cumulative Sales

-$7,000

Nil

-$7,000 - $1,000 = -$8,000 -$8,000 + (4,400 x $30 x 0.12) = $7,840 -$7,840 - $1,000 = $6,840

4,400 x $30 = $132,000

Make G

$6,840 + (4,100 x $10 x 0.08) = $10,120

$132,000 + (4,100 x $10) = $173,000

Specific fixed costs of H

$10,120 - $1,000 = $9,120

Make H

$9,120 + (4,600 x $20 x 0.07) = $15,560

Specific fixed costs of J

$15,560 - $1,000 = $14,560

Make J

$14,560 + (3,800 x $15 x -0.14) = $6,580

No products sold (only non-specific costs) Specific fixed costs of K Make K Specific fixed costs of G

$173,000 + (4,600 x $20) =$265,000

$265,000 + (3,800 x $15) =$322,000

135 | P a g e

Profit volume chart for products G, H, J, and K 20,000

15,000

H Profit/Loss ($’000)

J

G

10,000

5,000

K 0 50 5,000

100

150

200

250

300

350

Sales ($’000)

10,000

Part (e) The chart illustrates the profit that could be earned by the 4 products. It assumes that we will earn profits in line with contribution and assumes that the products are manufactured in the order of highest contribution earners first. It also shows which products are earning more contribution as the more steep the line is the more contribution is earned, clearly here we can see that K is the most steep and earns the most contribution then G and then H. J slopes downwards and shows that it is making a negative contribution per unit. J should not be manufactured on these grounds. We can also see the specific fixed costs for each product on the chart by the initial reduction of profit by £1,000 at the start of each separate product line. The specific fixed would not be spent if the product was not made.

136 | P a g e

A2 – 6 H (CIMA P2 May 2007) Part (a) Relevant costs

Technical report Material A Material B Direct labour Supervision Machine A Machine B Despatch Fixed overhead costs Profit mark-up Total costs

Workings W1 W2 W3 W4 W5 W6 W7 W8 W9 W10

$ 0 15,000 2,000 500 0 240 100 400 0 0 18,240

Workings W1 – Technical report The cost of the technical report has already been spent and so should not be included. It is a sunk cost. W2 – Material A It is a direct material which is regularly used and therefore we would need to use the replacement cost to value it. 10,000 sheets x $1.50 = $15,000. W3 – Material B We need 200 litres of ink specifically for the job but we can only buy in order sizes of 250 litres. There is also no certainty of any value for the remaining ink therefore the full amount should be included. $8 x 250 litres = $2,000. W4 – Direct labour The cost for direct labour is the overtime = 50hrs x $10p/h = $500. W5 – Supervision The cost for the supervisor should be zero as currently she is able to include the additional duties within her current hours. W6 – Machine A Lost contribution of machine A hours = 20 hrs x $12per hr = $240.

137 | P a g e

W7 – Machine B These are the extra running costs = 25 machine hrs x $4 per hr = $100. W8 – Despatch Delivery cost necessary for the catalogues of $400. W9 – Fixed overhead costs These costs are ignored as they are not incurred as result of this job. W10 – Profit The profit mark up should be ignored as we are working out the lowest possible quote for this job. Part (b) In the case of short-term pricing it is appropriate to look at relevant costing. This is because relevant costing takes into account the true costs and benefits that would occur as a result of selling at a price today. It ignores those costs that have already been spent such as fixed overheads as these would not be incurred now if the product was sold. In the case of long-term pricing it is appropriate to include these fixed overheads as they have not being occurred yet and would be as result of the manufacture of the product in question. As a result of this the long-term price maybe higher than the short-term price, and may mean that the price is no longer competitive. Traditional absorption costing systems tries to include an element of fixed overhead costs in the unit cost of products; this would be based on an activity basis that best represents how the fixed overheads will be used. However fixed overheads do not increase or decrease on a per unit basis they change when capacity is reached for a facility in the organisation. This makes the inclusion of fixed overheads arbitrary and the true profitability of the different products distorted. Relevant costing would ignore the fixed overheads when trying to understand the profitability of the different products because it would show the true contribution to fixed overheads without any arbitrary distortions.

138 | P a g e

A2 – 7 DFG (CIMA P2 Nov 2007) Part (a) Define the key variables - this is the assigning of letters to the products and services that are needed to be made and then using these letters to represent the amount that should be made at the optimum point. Let:

D = number of D units produced G = number of G units produced

Construct the objective function – this is looking at identifying the main objective that is trying to be achieved. Here we are trying to maximise the contribution for D and G, so we need to work this out first and then construct the objective function.

Selling price Direct material A Direct material B Skilled labour Variable overhead Contribution

D per unit ($) 115 20 12 28 14 41

G per unit ($) 120 10 24 21 18 47

Objective function: C = 41D + 47G Set up the constraints – these show the limits of resources available to us to try and meet the conditions of the objective function and they are usually described as linear equations. D per unit G per unit Usage of material A 20 / 5 = 4kg 10 / 5 = 2kg Usage of material B 12 / 3 = 4kg 24 / 3 = 8kg Usage of skilled labour 28 / 7 = 4hrs 21 / 7 = 3hrs Usage of machine 14 / 2 = 7hrs 18 / 2 = 9hrs 4D + 2G ≤ 1,800 4D + 8G ≤ 3,500 4D + 3G ≤ 2,500 7D + 9G ≤ 6,500 D ≤ 400, G ≤ 400

(Material A constraint) (Material B constraint) (Skilled labour hours constraint) (Machine hours constraint) (Maximum demand constraints)

139 | P a g e

Logic or non-negativity constraints – these are constraints which will ensure that the answer obtained in the solution is sensible in that only zero or positive values are in the answer. D ≥ 0, G ≥ 0

(Non-negativity constraints)

All constraints are plotted on to a graph and then moving away from the origin a solution is sought where all constraint conditions are met and maximises the objective function. Material A (4D + 2G = 1,800) If D = 0 then: 4(0) + 2G = 1,800 2G = 1,800 G = 1,800 / 2 G = 900 If G = 0 then: 4D + 2(0) = 1,800 4D = 1,800 D = 1,800 / 4 D = 450 Material B (4D + 8G = 3,500) If D = 0 then: 4(0) + 8G = 3,500 8G = 3,500 G = 3,500 / 8 G = 438 If G = 0 then: 4D + 8(0) = 3,500 4D = 3,500 D = 3,500 / 4 D = 875

140 | P a g e

Skilled labour (4D + 3G = 2,500) If D = 0 then: 4(0) + 3G = 2,500 3G = 2,500 G = 2,500 / 3 G = 833 If G = 0 then: 4D + 3(0) = 2,500 4D = 2,500 D = 2,500 / 4 D = 625 Machine hours (7D + 9G = 6,500) If D = 0 then: 7(0) + 9G = 6,500 9G = 6,500 G = 6,500 / 9 G = 722 If G = 0 then: 7D + 9(0) = 6,500 7D = 6,500 D = 6,500 / 7 D = 929

141 | P a g e

G 900

Material A (4D + 2G = 1,800)

800 Skilled labour (4D + 3G = 2,500) 700 Machine hours (7D + 9G = 6,500) 600 Max sales D = 400 500 Max sales G = 450

400 300

Material B (4D + 8G = 3,500) Feasible Region

200 100

100

200

300

400

500

600

700

800

900

D

Through observing the graph the solution appears to be to make 290 units of G and 315 units of D. This however is only as accurate as the graph drawn. The solution can also be derived through simultaneous equations which is far more accurate than using the graph or graphical method. We know that our solution is where “material A constraint” intersects with the “material B constraint”.

Material A Material B

4D + 2G = 1,800 4D + 8G = 3,500

Equation 1 Equation 2

We can use the subtraction method or substitution method to solve. We will use the subtraction method. Therefore subtract equation 2 from equation 1 4D + 2G = 1,800 4D + 8G = 3,500 -6G = -1,700

Equation 1 Equation 2

142 | P a g e

6G = 1,700 G = 1,700 / 6 G = 283 Substitute G =283 into equation 1 4D + 2(283) = 1,800 4D + 566 = 1,800 4D = 1,800 – 566 4D = 1,234 D = 1,234 / 4 D = 308.5 Therefore to maximise contribution we should make: 308 units of D and 283 units of G. Part (b) The shadow price is the extra contribution earned if one more unit of the scarce resource was made available. A shadow price only exists for scarce resource. Skilled labour has a nil shadow price because at the optimal solution it is not fully utilised as there are some skilled labour hours left. This can be seen in the graph in that the optimal solution lies below the skilled labour constraint indicating that there are excess skilled labour hours available. Direct material A is a scarce resource because at the optimal solution in the graph we are on the constraint itself and so we have fully utilised all this resource. This is further indicated by the value of £5.82 above. If we were to obtain another unit of direct material A then we would earn a contribution of £5.82. Also as a result of this our optimal solution would change. Part (c) If the selling price of product D increased then this would have direct impact on the objective function or iso-contribution line. It would change how the objective function sloped. In order to calculate how much the selling price would have to rise before the optimal solution would change we would have to identify the extreme points of the feasible region and the relative unit contributions of D and G that would cause a change in choice of optimal solution in the objective function.

143 | P a g e

A2 – 8 Highly skilled workers (CIMA P2 May 2008) Part (a) The limiting factor is material M1 and the most profitable course action would be to make those products that will maximise the contribution earned per square metre of M1. P4 $

P6 $

C3 $

C5 $

Selling price or opportunity cost

125

175

75

95

Variable cost Contribution

(55) 70

(75) 100

(44) 31

(45) 50

M1 used at $20 per square metre

0.75

0.50

0.25

0.50

70/0.75 = $93.33

100/0.50 = $200

31/0.25 = $124

50/0.50 = $100

4

1

2

3

Contribution per square metre Rank in order of production

Amount of M1 available is 1,000 square metres.

P6 C3 C5

1,500 units x 0.50 500 units x 0.25 1,000 – 750 – 125 = 125 square metres left

125 / 0.50 = 250 units

M1 used 750 125 125 1,000

Optimum production plan

P6 C3 C5

Make (units) 1,500 500 250

Other factors: Exam tip: The examiner is only after three other factors, however we have given you other factors that could be equally valid to discuss. 

The non-production of P4 may have a detrimental impact on customers. They may go to other suppliers who provide both P6 and P4 and not just P6.

144 | P a g e



P6 and P4 maybe complimentary products and so the non- production of P4 may reduce the sales of P6.



The fixed overheads may have a variable element varying with production; if this is the case then contribution will change for some of the products and therefore the optimum production plan.



The stability of the cost of material M1 as if this increases then it may not be beneficial to make C3 and C5 but to buy them in.



The future market of the products if they are going to continue to be popular with customers or whether they are starting to fall out of favour because of better products coming on to the markets to replace them.



The elasticity of demand of the products and in particular how sensitive demand would be to changes in prices.

Part (b) If we had more material M1 available then we would make a further 750 units of C5 and then 2,000 units of P4. The amount of M1 needed to make 750 units of C5: 750 units x 0.50 = 375 square metres This would save $100 per square metre of M1 and so therefore the maximum price would be $120 including the cost of M1. The amount of M1 needed to make 2,000 units of P4: 2,000 units x 0.75 = 1,500 square metres This would give contribution of $93.33 per square metres of M1 and so therefore the maximum price would be $113.33 including the cost of M1. After satisfying these requirements there is no further need for M1 and so therefore the maximum price to be paid for M1 is zero. Part (c) If we were to supply this contract and we have no more M1 then we have to stop the production of some or all of C5, C3 and P6. We should stop the production of those products which give us the least amount of contribution per square metre of M1 first. 500 units of P4 would need 500 x 0.75 = 375 square metres of M1. 145 | P a g e

Therefore take material M1 as follows to fulfil the new order: 125 square metres from C5 125 square metres from C3 125 square metres from P6 Lost contribution: C5 125 x $100 C3 125 x $124 P6 125 x $200 Total

$12,500 $15,500 $25,000 $53,000

Contribution from the 500 units = 500 x $70 = $35,000 Net benefit from doing the 500 units = $53,000 - $35,000 = $18,000 Therefore the minimum financial penalty must be $18,000. A2 – 9 RT (CIMA P2 May 2010) Part (a) In order to find the optimum production plan we must first establish what the scarce resource is that is restricting production to meet all demand. We will work out the total amount of resources needed to meet maximum demand and then compare this to the resources that we have available to us to determine any scarce resources.

Total demand Direct labour (hours) Material A (kg) Material B (litre) Machine hours

R 750 2,250 3,750 1,500 2,250

T 1,150 5,750 4,600 1,150 4,600

Total 8,000 8,350 2,650 6,850

Direct labour hours is the scarce resource or limiting factor as we only have available to us 7,500 hours and we need 8,000 hours.

146 | P a g e

Product

R $ per unit 130 (24) (15) (14) (30) 47

T $ per unit 160 (40) (12) (7) (40) 61

Contribution per labour hour

47/3 = 15.67

61/5 = 12.20

Rank in order of production

1

2

Selling price Direct labour ($8 per hour) Material A ($3 per kg) Material B ($7 per litre) Machine hours ($10 per hour) Contribution

Total labour hours available is 7,500 hrs, however we must fulfil a commercial customer order first of 250 R’s and 350 T’s before working out the optimum production plan. Commercial contract 250 350

R T Total

Labour hours per unit 3 5

Labour hours used 750 1,750 2,500

Amount of labour hours available after commercial contract is 5,000 hrs.

R T

500 units x 3 hrs 5,000 hrs – 1,500 hrs = 3,500 hrs

3,500 hrs / 5 hrs = 700 units

Labour hrs used 1,500 3,500 5,000

Optimum production plan

R T

Commercial contract 250 350

Production 500 700

Total 750 1,050

Contribution per unit from commercial contract Product Selling price Direct labour ($8 per hour) Material A ($3 per kg) Material B ($7 per litre) Machine hours ($10 per hour) Contribution

R $ per unit 100 (24) (15) (14) (30) 17

T $ per unit 135 (40) (12) (7) (40) 36 147 | P a g e

Contribution from production plan

R T Contribution

Contract 250 x $17 = $4,250 350 x $36 = $12,600

Market 500 x $47 = $23,500 700 x $61 = $42,700

$27,750 $55,300 $83,050

Part (b) By carrying out the commercial contract it means that we fall short of meeting market demand of T by 100 units. The lost contribution of this is 100 units x $61 per unit = $6,100. This is less than the financial penalty of $10,000 if the commercial contract was not met, so therefore it is better at least from a financial perspective to fulfil the commercial contract. Part (c)

Labour (hrs) Material A (kg) Material B (kg) Machine hours

Revised resource availability 7,500 x 0.9 = 6,750 8,500 x 0.9 = 7,650 3,000 x 0.9 = 2,700 7,500 x 0.9 = 6,750

Commercial contract 2,500 2,650 850 2,150

Resources remaining 4,250 5,000 1,850 4,600

Objective function = 47R + 61T Constraints: 3R + 5T ≤ 5R + 4T ≤ 2R + T ≤ 3R + 4T ≤ R ≤ 500, T ≤ 800

4,250 5,000 1,850 4,600

(Labour) (Material A) (Material B) (Machine hours) (Maximum demand)

Logic or non-negativity constraints – these are constraints which will ensure that the answer obtained in the solution is sensible in that only zero or positive values are in the answer. R ≥ 0, T ≥ 0

(Non-negativity or logic)

All constraints are plotted on to a graph and then moving away from the origin a solution is sought where all constraint conditions are met and maximises the objective function.

148 | P a g e

(Labour) 3R + 5T = 4,250 If R = 0 then: 3(0) + 5T = 4,250 5T = 4,250 T = 4,250 / 5 T = 850 If T = 0 then: 3R + 5(0) = 4,250 3R = 4,250 R = 4,250 / 3 R = 1,417 (Material A) 5R + 4T = 5,000 If R = 0 then: 5(0) + 4T = 5,000 4T = 5,000 T = 5,000 / 4 T = 1,250 If T = 0 then: 5R + 4(0) = 5,000 5R = 5,000 R = 5,000 / 5 R = 1,000 (Material B) 2R + T = 1,850 If R = 0 then: 2(0) + T = 1,850 T = 1,850 If T = 0 then: 2R + 0 = 1,850 2R = 1,850 R = 1,850 / 2 R = 925

149 | P a g e

(Machine hours) 3R + 4T = 4,600 If R = 0 then: 3(0) + 4T = 4,600 4T = 4,600 T = 4,600 / 4 T = 1,150 If T = 0 then: 3R + 4(0) = 4,600 3R = 4,600 R = 4,600 / 3 R = 1,533 T 2,000 Material B (2R + T = 1,850) 1,600 Skilled labour (3R + 5T = 4,250) 1,400 Machine hours (3R + 4T = 4,600)

1,200

Max sales R = 500 1,000 Max sales T = 800

800 600 400

Material A (5R + 4T = 5,000) Feasible Region

200

500

1,000

1,500

The optimal solution is the furthest point away from the origin within the feasible region; therefore the optimal production plan is make 500 units of R and 550 units of T in addition to the contract.

150 | P a g e

R

Part (d) If there is an increase in optimism by mangers then the constraints would all move to the right illustrating the availability of more resources and production would increase. Currently the most constraining lines are labour hours and maximum demand for R. If labour hours were increased then there would be further production of T as the labour constraint would move to the right. This would continue up until the labour constraint intersects with the material A constraint and maximum demand for R, it is at this point material A also becomes a constraining resource and would need to be increased if production were to further increase. Clearly it can be said that in any event labour hours needs to increase if production were to be expanded, other resources if increased would have no immediate effect as there are resources in excess available. A2 – 10 LM (CIMA P2 Nov 2010) Part (a) In order to find the optimum production plan we must first establish what the scarce resource is that is restricting production to meet all demand. We will work out the total amount of resources needed to meet maximum demand and then compare this to the resources that we have available to us to determine any scarce resources.

Total demand Direct labour (hours) Direct material (kg) Machine hours

L 400 1,600 800 400

M 700 1,400 6,300 1,400

Total 3,000 7,100 1,800

Direct material is the scarce resource or limiting factor as we only have available to us 6,000 kg and we need 7,100 kg. Product Selling price Direct labour ($7 per hour) Direct material ($5 per kg) Machine hours ($10 per hour) Contribution Contribution per kg Rank in order of production

L $ per unit 70 (28) (10) (10) 22

M $ per unit 90 (14) (45) (20) 11

22/2 = 11

11/9 = 1.22

1

2 151 | P a g e

Amount of material available is 6,000 kg.

L M

400 units x 2 kg 6,000 kg – 800 kg = 5,200 kg

5,200 kg / 9 kg = 577 units

Kg used 800 5,193 5,993

Optimum production plan

L M

Production 400 577

Contribution from production plan L M Contribution

400 x $22 = $8,800 577 x $11 = $6,347 $15,147

Part (b) The agreed order of 250 units of product M should be worked out separately for the resources needed to complete it first as this is a requirement, then we can see what resources are left over to formulate our new resource constraints. 250 units of product M would need: Direct labour = 2 hrs per unit x 250 units = 500 hrs Direct material = 9kg per unit x 250 units = 2,250 kg Machine hours = 2 hrs per unit x 250 units = 500 hrs Resources have also been overestimated by 20% and need to be reduced before deduction of resource usage by the agreed order of 250 units of M, therefore: Direct labour = (3,500 hrs x 80%) - 500 hrs = 2,300 hrs Direct material = (6,000 kg x 80%) - 2,250 kg = 2,550 kg Machine hours = (2,000 hrs x 80%) – 500 hrs = 1,100 hrs Revised resource constrains: Direct labour Direct material Machine hours

4L + 2M ≤ 2,300 2L + 9M ≤ 2,550 1L + 2M ≤ 1,100

Objective function: C = 22L + 11M 152 | P a g e

Part (c) Product L 400 and product L other value 0 The value 400 represents the amount of L we should produce in order to maximise contribution given the resource constraints. The other value 0 means that we have no further units of L to make as we have reached the maximum market demand for L. units. There is no shortfall in demand. Product M 194 and product M other value 506 The value 194 represents the amount of M we should produce in order to maximise contribution given the resource constraints. The other value 506 means that we have not met our maximum market demand of M by 506. We are only producing 194 units of M where as the maximum market demand is 700. There is a shortfall in demand of 700 units – 194 units = 506 units. Machine hours 312 This value represents the number of unused machine hours left at the optimal production point where contribution is maximised given current resource constraints. We can proof this amount of unused machine hours by comparing what has been used to the total amount of machine hours available. L units produced = 400 x 1 hr per unit = 400 hrs M units produced = 194 x 2 hrs per unit = 388 hrs Total hours used = 400 + 388 = 788 hrs Hours available = 1,100 hrs Hours unused = 1,100 hrs – 788 hrs = 312 hrs Direct material $1.22 This is the shadow price for direct materials as it is a scarce resource at the optimal production point where contribution is maximised given current resource constrains. The shadow price is maximum price you should pay above the original cost for one more extra unit of the scarce resource, in this case being one more kg of direct material. The proof is that if we were given 1 more kg of direct material we would use it to increase output of product M, because still has unfulfilled demand. Each unit of M requires 9 kg, therefore 0.11 units of M could be produced from 1 kg of material. Each unit of M yields a contribution of $11 and therefore 0.11 units yield $1.22 contribution.

153 | P a g e

Labour hours 312 This value represents the number of unused labour hours left at the optimal production point where contribution is maximised given current resource constraints. Contribution $10,934 This is the maximum contribution that can be earned given the constraints at the optimal production point. Proof: Contribution from L units = 400 x $22 = $8,800 Contribution from M units = 194 x $11 = $2,134 Total contribution = $8,800 + $2,134 = $10,934 A2 – 11 Hotel (CIMA P2 May 2011) Part (a) Peak $ 855,000 (184,680) (68,400) (225,000) 376,920

$ Room revenue Guest related costs (W1) Room costs (W2) Avoidable general costs (W3) Room / Guest contribution Snacks Gross contribution (W4) Cook costs

4,617 (5,000)

$

12,150 (6,667) (383)

Restaurant Gross contribution (W5) Staff costs

17,313.75 (13,500)

$

Low $ 412,500 (108,000) (82,500) (375,000) (153,000)

8,150 (8,133) 5,483

33,750 (18,000) 3,813.75 380,350.75 (75,000) 305,350.75

Total contribution Non-avoidable general costs Net contribution Hotel annual fixed costs Hotel annual profit

Mid $ 720,000 (162,000) (81,000) (300,000) 177,000

(233)

4,867 98,313.75 (54,000)

24,750 (128,483) (125,000) (253,483)

Workings W1 – Guest related costs Season Peak Mid Low

Days (D) 90 120 150

Rooms (R) 95 75 50

Occupants (O) 1.8 1.5 1.2

Guests (D x R x O) 15,390 13,500 9,000

Total $ 1,987,500 (454,680) (231,900) (900,000) 400,920

24,867 (20,000)

47,250 (22,500) 15,750 198,233 (100,000) 98,233

$

Guest related costs ($12 per guest per night) 184,680 162,000 108,000 154 | P a g e

44,313.75 450,100.75 (300,000) 150,100.75 (200,000) (49,899,25)

W2 – Room costs Peak = ($5 + $3) x 90 days x 95 rooms = $68,400 Mid = ($5 + $4) x 120 days x 75 rooms = $81,000 Low = ($5 + $6) x 150 days x 50 rooms = $82,500 W3 – Avoidable general costs Peak = $300,000 x 75% = $225,000 Mid = $400,000 x 75% = $300,000 Low = $500,000 x 75% = $375,000 W4 – Snacks gross contribution Peak = 15,390 guests x 10% x $10 x 30% = $4,617 Mid = 13,500 guests x 30% x $10 x 30% = $12,150 Low = 9,000 guests x 30% x $10 x 30% = $8,100 W5 – Restaurant gross contribution Peak = 15,390 guests x 30% x $15 x 25% = $17,313.75 Mid = 13,500 guests x 50% x $20 x 25% = $33,750 Low = 9,000 guests x 70% x $30 x 25% = $47,250 Part (b) (i) Overall from our statement it is obvious that the hotel is making losses of nearly $50,000. Actions management could take to maximise profits: 

During the low season they could shut down the hotel as it is making the main losses in this part of the year.



Close down the snack service during the peak and the low season as it makes losses in these seasons but makes profits during the mid season when it should remain open.

Part (b) (ii) Other factors that the managers should consider: It should be investigated as to how easily the snack service can be shut down and reopened only for the mid season. If the snack service was closed entirely whether or not this would encourage guests to use the restaurant.

155 | P a g e

If the hotel was closed during the low season whether or not this would still retain the regular guests to come in the peak and mid season or whether guests will view this as being an obvious move to focus on profits rather than customer service.

156 | P a g e

Solutions – Section A Part B Cost Planning and Analysis for Competitive Advantage B1 – 1 SWAL (CIMA P2 Pilot Paper 2005) This question had at least one mark for writing your answer in a report format!

REPORT To: From: Subject: Date:

Managing Director of the SWAL group Management Accountant JIT differences and review of current quality control systems 19th September 2001

1. Introduction This report is designed to explain how a just-in-time (JIT) system differs from the system presently being used and the extent to which its introduction would require a review of SW’s quality control procedures. 2. JIT purchasing and production The JIT philosophy states that products should only be produced if there is an internal or external customer waiting for them. Traditionally manufacturers stockpiled. JIT aims ideally for zero stock e.g. raw materials delivered immediately at the time they are needed, no build up of work-in-progress, finished goods only produced if there is a customer waiting for them. 1. Closer relationships with suppliers required, fewer and more frequent items, in return for this the supplier would get long term steady purchase orders. 2. Smaller more frequent deliveries need to be managed in order to produce. 3. Higher quality machines perhaps multi-purpose with regular maintenance. 4. Involvement and training of staff to maintain flexibility. 3. The current system compared to JIT The current system aims to procure chemicals based on holding a minimum quantity this is very different to JIT. JIT would aim to always hold a zero or near zero stock level as chemicals should be bought in when there is a demand for them not to hold a minimum level. To secure regular small amounts of chemicals bought in under JIT, SWAL group would have to enter into long term contracts, and therefore order costs may increase using such as stock control system. 157 | P a g e

However holding costs such as damage, deterioration, administration, security and interest costs will all be significantly reduced under JIT. The EOQ will not be needed in this situation because holding costs will be near zero and there would not be any significant amounts of chemicals held. 4. JIT and quality control procedures If SWAL adopts JIT in managing its stock levels then there maybe a lot of reliance on the one supplier delivering on time and ensuring the chemicals are of satisfactory quality. There is no room for error and it would mean production would have to stop as SWAL would not carry any emergency stocks. SWAL would need to build good working relationships with suppliers. One way of achieving this is to have one supplier with whom SWAL has a long-term contract guaranteeing many purchases or exclusivity to that supplier. Quality control checks should be carried out on materials by supplier before it arrives at SWAL to ensure quality of product. SWAL will not have the time to carry out detailed checks as the chemicals would be needed straight away in production. If the chemicals were not satisfactory then the supplier could face penalties, cancellation fees and perhaps the termination of long-term contract. Quality controls should be put into place in the production process to eliminate scrap, reworks and defects. There should be proper and regular maintenance of the machines used in production by the staff using them, this will help to ensure an unbroken production line. Staff should receive the necessary training in how to use the machines and maintain them. Quality circles, where a group of managers, normally working in the same area, volunteer to meet on a regular basis, can be used to:    

Identify areas for improvement. Analyse work related problems in order to find alternative solutions. Brainstorm for new ideas or alternatives. Investigate report and discuss findings.

There is much that is needed to be changed an improved if we are to adopt a JIT approach and should be planned carefully, and the benefits of higher quality and cheaper production will only be seen in the long term. I would suggest many consultations with myself and other experts in this field before embarking on such a stock control system.

158 | P a g e

I hope you have found this report useful but should you require any further assistance or have any questions please do not hesitate to contact me. Signed : Management Accountant. B1 – 2 X group (CIMA P2 May 2005) This question had two marks for explaining and defining JIT and the rest of the eight marks were for profitability would be affected.

REPORT To: From: Subject: Date:

Managing Director of the X group Assistant Management Accountant Adoption of JIT and its impact on profitability 19th September 2005

1. Introduction This report is designed to explain how the adoption of JIT might affect X group’s profitability. 2. JIT purchasing and production The JIT philosophy states that products should only be produced if there is an internal or external customer waiting for them. Traditionally manufacturers stockpiled. JIT aims ideally for zero stock e.g. raw materials delivered immediately at the time they are needed, no build up of work-in-progress, finished goods only produced if there is a customer waiting for them.    

Closer relationships with suppliers required, fewer and more frequent items, in return for this the supplier would get long term steady purchase orders. Smaller more frequent deliveries need to be managed in order to produce. Higher quality machines perhaps multi-purpose with regular maintenance. Involvement and training of staff to maintain flexibility.

3. The impact on profitability By the introduction of JIT, X group should see the following effects on profits: 1. Reduction in stock holding costs for raw materials, finished goods and work in progress. 2. Reduction in number of suppliers used to try to ensure quality and as a result may pay slight premium. 159 | P a g e

3. Increase in price of raw materials from suppliers for regular small amounts ordered at short notice, but maybe reduced through the introduction of long-term contracts with suppliers. 4. Additional costs incurred in improved quality control procedures put in the production process at X group. 5. Staff training costs to ensure that they are able to carry out control procedures and use the material efficiently. 6. Additional planning is required by management to ensure that there are no stock outs, and therefore addition administration costs. I hope you have found this report useful but should you require any further assistance or have any questions please do not hesitate to contact me. Signed : Assistant Management Accountant. B1 – 3 ML (CIMA P2 Nov 2005) Part (i) Traditional absorption costing takes the total budgeted fixed overhead for the period and divides by a budgeted (or normal) activity level in order to find the overhead absorption rate The overhead absorption rate is then used to absorb fixed overheads into the range of products we provide. Once all costs have been allocated then a price is obtained by adding a % mark up on the costs. The mark up would represent the profit earned on the product. This approach is used by companies that have a single or small range of products, fixed overhead are a small percentage of total cost, and mass production techniques are used. ML engineering uses an absorption costing approach based on machine hours. This is particularly useful in this circumstance as it means that every product will always recover a portion of the production fixed costs. However it may not be a fair method as not every product will consume the fixed overheads based on the single activity being machine hours. Some products may not use machine hours but still consume fixed production overheads. The main consequence of this is that prices charged on the various products may not be competitive, due to the fact the “cost plus” approach depends on the total costs allocated to that product. As a result of this some products make losses because fixed production costs being unfairly allocated to it and therefore discontinued when in fact it maybe making profit.

160 | P a g e

The alternative being a marginal cost approach would seek to ignore the fixed production overheads. The difference between absorption costing and marginal costing organisations is that the marginal costing organisation makes no attempt to ‘absorb’ production overhead into a cost unit. It treats production overhead as a period cost only and does not absorb overhead, but rather charges it entirely to the profit and loss account for each period. It is equally important to remember that marginal costing organisations would also value stock at variable production cost only not full production cost, as does an absorption costing company. This means that pricing is more competitive as costs would be lower and it means that unused capacity in ML could be used as more contracts would be secured. The main problem with this is that ML still may not be competitive as the mark up maybe too high. Part (ii) Quite clearly the Managing Director is suggesting a marginal costing approach to price by making this comment. A marginal costing approach is very good approach in but only in the short term, this is because fixed costs have already been spent or committed to and by ignoring these and concentrating on those costs that do change in the short term it means that we are covering all the costs that we would incur, but only in the short term. Therefore any price that exceeds variable costs would be making some contribution to fixed costs, but this may not be sustainable over the long term and fixed costs need to be recovered at some point and enough profit needs to be made for the products to be continued to be made. By ignoring fixed production costs there is no attempt to understand and control them to ensure that they are costs that should be spent and used effectively. Using a marginal costing approach there is danger that these costs could spiral out of control over the long term. In addition by charging a lower price initially it maybe difficult to justify to customers price increases especially if the market is competitive. B1 – 4 PK plc (CIMA P2 Nov 2005)

REPORT To: From: Subject: Date:

Management Team of PK plc Assistant Management Accountant Changing nature of cost structures in the modern environment 12th December 2005 161 | P a g e

1. Introduction This report is designed to explain the changing nature of cost structures in the modern manufacturing environment and the implications to PK plc’s inventory valuation and short term decision making. 2. Cost structures in the modern manufacturing environment Absorption costing system used by PK plc takes the total budgeted fixed overhead for the period and divides by a budgeted (or normal) activity level in order to find the overhead absorption rate. This approach is used by companies that have a single or small range of products, fixed overhead are a small percentage of total cost, and mass production techniques are used. It is a system that has been used for over a hundred years now and is perhaps seen as inappropriate today because of increased competition from overseas companies, economies of scale enjoyed by many companies, trade barriers and barriers to entry into many markets being reduced significantly or abolished, improvements in technology and access that customers have around the world to other manufacturers has made the global economy into a local economy. PK plc is in an industry producing high quality pieces of furniture and where labour skills are very important. High quality furniture manufacture tends to have a lot of different machines which have specific in built programs. This means there are a range of different machines and different types of fixed production overheads not just one type these days. Therefore we need to understand how these are driven by relating these to activities which cause their expenditure and as a result control the costs far better. So unlike traditional absorption costing where you have one or two costs drivers we are now seeking several cost drivers relating to different segments of the total fixed production costs and relating this to how products consume them. This is a modern technique known as activity based costing (ABC). 3. Inventory valuation We can obtain an inventory valuation using the absorption costing method which includes a portion of fixed production costs in the valuation. Inventory valuation is important because it is a reporting requirement but also gives you an understanding as to how much money is being tied up in these products that are being manufactured and how much profit is being made once the items have been sold. Financial reporting requirements state that stock valuation should include “all costs bringing the item to its present location”. This shows that stock valuation is not based on change of activity but on a blanket absorption rate and therefore not giving a true reflection of cost of the product and therefore profitability. 162 | P a g e

4. Short-term decision making When making decision in the short term then we must include those cost and revenues which change as a result of our decision, and ignore those that don’t. This normally means that we are interested in variable costs only as fixed costs are normally historical or committed. Fixed production costs cannot be changed in the short term. If this is the case indeed PK plc should focus on making decisions looking at how much contribution (sales less variable costs) they can earn rather than profit (sales less all costs). This will make them more competitive against their rivals, being able to charge a lower price and still earning contribution to fixed costs. But of course over the long term they cannot take a marginal costing approach because they would need to recover the fixed production costs as well. ABC has many uses and there are very good benefits to an organisation when formulating their long term strategic decisions, such as product pricing, mix of products, discontinuance, launch or promotion of existing products and the launch of new products. ABC’s strength lies in the fact it allows accuracy over costs and drivers for products and as a result a sensible pricing strategy is achieved. It more specifically gives a good long term understanding of the variable costs being very relevant for decision making. However ABC information must be put into perspective as these are historic costs and cannot be used alone to predict future costs. They should be used as a starting point and other internal and external information should be used to determine future costs. All costs are variable in the long term and subject to change. I hope you have found this report useful but should you require any further assistance or have any questions please do not hesitate to contact me. Signed : Assistant Management Accountant. B1 – 5 Financial advisors (CIMA P2 May 2006) Part (a) Value Analysis Value analysis focuses on removing those costs during production which do not add value to the product because the customer does not actually want them as they are not viewed by the customer as adding any value.

163 | P a g e

Value engineering is the activity which looks at how to achieve the same quality product for customers at the lowest costs possible. It focuses before on costs before production has begun whereas value analysis focuses at costs during production. Although value analysis is another form of cost reduction it is different in that it seeks not just to reduce costs as far as possible but takes into account what is important to customers. Four aspects of value:  Cost value – cost of production.  Exchange value – sales price.  Utility value – what its functionality is.  Esteem value – prestige that it is perceived hold. Value analysis seeks to reduce “cost value” while maintaining the “esteem value” of a product. It normally is easier reduce the “cost value” on items which have a “utility value” as opposed to an “esteem value”. Functional analysis This looks at the functions that a product has and their perceived value to customers versus the cost of providing these. There are nine basic steps involved in functional analysis: 1. Choosing the object of analysis – product, service or overhead area where there is high cost, complex, low demand. 2. Select the members of the team – different departments. 3. Gather information – internal and external. 4. Define the functions of the object – list all the functions of product (e.g. functions of a pen?). 5. Draw a functional family tree – showing relationship of function to part product and its cost. 6. Evaluate the function – arrive at a total target cost and compare to customer’s expectations. 7. Suggest alternatives and compare these with the current or target (expected) cost – new methods and materials. 8. Choose the alternative operations. 9. Review the actual results.

164 | P a g e

Part (b) Value analysis method 1. Obtain the precise requirements of the customer by considering utility, esteem and exchange value. 2. Obtain alternative ways of manufacturing the requirements of the customer’s product. For example, the newsletter that the firm produces is not read by most clients, perhaps looking at alternative ways to convey this information to clients such as email, meetings, free one off business advice days. 3. Authorise the suggested alternative methods in achieving the customer desires by senior management, before production begins. 4. Produce and manufacture product. 5. Evaluate changes and response from customers. Benefits of this maybe positive feedback from customers, improved sales and profitability, eliminate wastage, attract more staff and better morale within the company. B1 – 6 Compliance v conformance (CIMA P2 May 2006)

REPORT To: From: Subject: Date:

Managing Director Management Accountant Quality costs 12th February 2006

1. Introduction This report is designed to discuss quality costs and their significance for the company, and in particular costs of conformance and costs of non-conformance. 2. Costs of conformance Total quality management (TQM) is the process of adopting a quality conscious philosophy within staff culture, as well as adopting standards and procedures e.g. such as ISO 9000 certification to monitor and control quality. Benchmarking, quality circles and investment in training and development of employees often achieve this. Conformance costs are those costs which are spent to try to achieve a standard or target, such regulations or functional specifications of a product for a customer. 165 | P a g e

There are two main types of conformance costs:  

Prevention costs – these are spent to try and ensure the product is made to detailed specifications and regulations. For example training staff, customer surveys, supplier reviews and investment in machines. Appraisal costs – these are spent to understand how well a process has performed and corrective action is taken if needed subsequently. For example measuring equipment, inspections and tests, product quality audits.

3. Costs of non-conformance Non-conformance costs are quality failure costs that are needed to correct products, as they did not meet expectations or target. There are two main types of non-conformance costs: 



Internal failure costs – these are quality failure costs before the products or services have been transferred to the customer. For example re-inspection of goods, losses or scrapping of materials and finished goods, additional administrative costs. External failure costs - these are quality failure costs before the products or services have been transferred to the customer. External costs should be avoided as they expose poor manufacturing abilities to customers. Examples are administration of customer complaints, administration of customer services, product liability claims, repairs and replacements, lost good will and reputation.

4. Importance of quality costs In the modern manufacturing environment customers are demanding better quality products at cheaper prices and on demand. This has been due to increased competition through global networks, deregulation and privatization of industries, increased innovation and use of automation, shorter product lifecycles and more efficient manufacturers. This has meant manufacturers these days must consider spending more and more of their money on producing better quality items, through improved processes and work with high quality materials. Companies these days must consider what their target audience is and what is important to them be it quality, cost, service or speed of delivery. Then consider what they need to spend on conformance costs and non-conformance costs. Companies must decide whether to undertake a check of all products or inspection by random sample of the products before they leave for customers. They should consider which is appropriate for their customers needs.

166 | P a g e

Although conformance costs are high costs in the short term, they would yield greater benefits in the long term in the form of greater profitability and longevity for the business. Manufacturing companies could use Kaizen costing as way of improving the long-term quality of their products. It is the process of continuous improvement by small incremental rather than transformational changes. Its also believes strongly in empowerment of employees to enable them to improve operations. Kaizen costing focuses on reducing variable costs of future periods below that of prior periods. Similar in concept to total quality management philosophy, Japanese idea and again works hand in hand with quality circles, benchmarking or quality assurance.      

Human resources are your most valued asset and they should be involved in the search for perfection. They have superior knowledge as they are in the operation. This is the opposite of traditional systems where the managers develop standards. Incremental or gradual improvement. Perfection should be sought all the time and so there is always room for improvement. Cost reduction targets more frequent rather than traditional annual standard costs. Variance analysis used to help with Kaizen costing. Investigations carried out if targets not met even if improvements have been made.

I hope you have found this report useful but should you require any further assistance or have any questions please do not hesitate to contact me. Signed : Assistant Management Accountant. B1 – 7 AVN (CIMA P2 Nov 2006) Question 2 Part (i) Porter grouped the various activities of an organisation into what he called the value chain; he divided the organizations activities into nine types, classified as either primary or secondary activities. These activities incur costs, but in combination with other activities provide customer satisfaction and therefore add value. The main components are as follows: 

Technology - R & D, product design and testing, process design and testing.



Inbound Logistics - Receiving, handling and storing inputs (warehousing, stock control, inbound transport). 167 | P a g e



Operations - Conversion of inputs (people, materials, machines etc) into the final product or service.



Marketing & Sales - Informing the customer (Marketing mix).



Outbound logistics - Delivery of the actual product or service (storage, outbound transport to customer).



Service - Customer service, repairs, installations, supply of parts/consumables.

The extended value chain is including external factors into the chain such as suppliers and customers creating a value system, an example of additional components are as below:

Supplier value chain Distributor value chains Supplier value chain Wholesaler value chain Supplier value chain

Retailer value chains

Part (ii) AVN manufactures electronic devices which appear to be quite sophisticated. This is a fast moving, volatile and competitive market and therefore AVN needs to ensure that they are supporting each of the main components of the value chain in order to be successful. Each of the main components may be applied to AVN as follows: Technology – AVN must make sure that their R&D is directed towards the needs of the customer as there is no point developing a product which consumers do not want to purchase. Functional analysis would be a useful way of achieving this. Once the prototype has been developed AVN must make sure that it is designed and tested to make it easy for the consumer to operate and be durable and robust enough for its purpose.

168 | P a g e

The product must also be cost effective and so target costing should be used here to determine the market price and profit desired and then the balancing figure being the budget for costs, which must be realistic. Inbound logistics – The material required for this product must be sourced from reputable and reliable suppliers. They must be checked for quality assurance and this would reduce level of assurance needed in AVN. Re-works and customers returns are in the long run more costly to rectify than making sure it was correct the first time it was manufactured. Cost should also be considered to make sure that the service is delivered at rate where AVN can make profits. Storage should also be suitable for these materials and be secure with additional capacity if needed. Operations – Commercial production must be cost effective and efficient. There should be coordination between inbound logistics and production determining the level of materials required. There should also be an understanding of what types of products should be made and the mix of labour required to achieve this. There should also be investment in quality machinery and quality controls to prevent internal and external failure costs. Marketing and sales – It is important that AVN informs the right target consumers of the new product that is being launched and its main benefits over older models or other substitutes supplied by other competitors. Outbound logistics – This is the delivery of the products to consumers. It is important that AVN deliver these items in perfect condition, on time and the right products. AVN must employ suitably trained sales team and distribution team to meet consumer needs, and also consider whether this will be cost effective. Service – It is vital for AVN to receive continual feedback on their products from the consumers to ensure that their needs are being fulfilled exactly, and if needs are changing these are also met by creating revised models. AVN should be looking to deliver a high quality after sales service to consumers in the way of dedicated support lines, replacement of parts and installations if required, all of which should be cost effective.

169 | P a g e

B1 – 8 W (CIMA P2 Nov 2006) Part (a)

Months No. of units produced and sold SP per unit VC per unit Contribution per unit Total contribution Cumulative cash flow

Maturity Decline 31 - 70 71 - 110 20,000 20,000 20,000 $60 $60 $40 $30 $25 $30 $30 $35 $10 $600,000 $700,000 $200,000 $1,275,000 $1,475,000

Part (b) Changes in costs There has been a continued reduction in unit variable costs of this product from its introduction stage to its maturity stage. It can be seen from the data given that as we produce more of the product unit variable cost falls from $50 to $25 per unit. This can be attributable to economies of scale being enjoyed by the company through mass production techniques, such as below: 

Learning curve effects enjoyed by the workforce if the production is labour intensive, repetitive and most of the staff is retained over the long term.



Technical efficiencies learned by all departments in the manufacture of the product, especially the production department through improved understanding of how material and machines can be used more efficiently to maximise production.



Stock control systems maybe more efficient as the data states the company employs a JIT approach to stock management. This reduces stock holding costs to nil and material is only used when and bought if there is a demand.



Discounts would have been received as the company would buy more and more bigger batches of the raw material throughout the product’s life cycle.



Total quality management techniques maybe used which means that the product is made to a very high standard reducing internal failure costs such as the inspection and scrap material and also reduction in external failure costs such as repairs and replacement.

170 | P a g e

The unit variable cost has also increased at the end of the product life cycle from $25 to $30 in its decline stage. This is expected as the demand for this product would be reduced significantly as it would have fallen out of favour with customers, and therefore less is being made and as a result fewer materials needed. The lower prices on raw materials cannot be enjoyed because of the reduced quantities bought by the company and hence the increase in unit variable costs. Changes in selling prices W has managed to initially charge a high price, achieve low volume but earned a larger profit per unit sold. This is known as market skimming. It is a strategy which exploits a price insensitive market or an inelastic demand for a product. It would seem that W’s product has been a product that customers have been willing to pay a premium for but this not sustainable as competitors enter the market. This can be seen during the growth stage as competition begins to enter the market W has had to reduce its selling price from $100 per unit to $80 and then $70 per unit. W will still continue to increase contribution through economies of scale. During the maturity and decline phase W has had to continue to reducing the selling price of their product in order to stay competitive as cheaper substitutes have entered the market produced by other manufactures. This approach will aim to sustain its wide customer base and W should continue to produce the product as long the selling price is greater than the marginal cost of manufacturing the product. We can see from our answer in (a) that this is still the case for W in the decline phase. Production should cease when this is not the case as the there would be no extra benefit for the company, at this point the product has fallen out of favour with customers. B1 – 9 New product (CIMA P2 May 2007) Part (i) The company did originally estimate the labour costs for the first batch to be $250,000, however the actual cost for the first batch was $280,000. There is a learning curve rate of 80% which we will apply to the actual cost to obtain the revised expected cumulative labour costs. Number of cumulative batches 1 2 4 8

Average labour cost (‘000s) $280 $280 x 0.8 = $224 $224 x 0.8 = $179.20 $179.20 x 0.8 = $143.36

Total labour costs (‘000s) $280 224 x 2 = $448 179.20 x 4 = $716.80 143.36 x 8 = $1,146.88 171 | P a g e

Part (ii) Number of cumulative batches 1 2 4 8

Average labour cost (‘000s) $280 $476 / 2 = $238 $809 / 4 = $202.25 $1,376 / 8 = $172

Learning curve rate

$238 / $280 = 0.85 $202.25 / $238 = 0.85 $172 / $202.25 = 0.85

Learning curve rate at each output is 85%. Part (iii) The labour costs estimated were different to the actual labour costs. The actual initial batch labour cost was more than the estimated cost by $30 but the actual learning rate was 5% better than the estimated. This will have an impact on the labour costs involved and may have an overall reduction in costs. This would lead to a more competitive pricing strategy, increased sales and profits, if the price could be reduced. B1 – 10 New small company (CIMA P2 Nov 2007) Part (a) Exam tip: You need only mention the first 2 paragraphs below to obtain the 3 marks for this part. The further paragraphs subsequent are to show other relevant things that are just as valid. Learning curve theory is the theory that as output doubles the average time per unit (total time for all units divided by the units produced) drops by a fixed percentage each time this happens. The same theory can be applied to find the average cost per unit as well as the average time. Labour intensive production processes like the one used in the company would mean that the humans used in this process will learn to do their job more efficiently and so over time the labour time needed would fall. The learning curve theory can calculate this fall in labour time required. Being a new small company it therefore has new products and so would be in the early stages of the product life cycle and production, therefore there would be room for improvement in terms of labour needs. If the industry that the new small company is in is very competitive then pricing on long run average costs can consider the extent to which costs will fall because of the learning curve effect. 172 | P a g e

Realistic standard costs could be achieved if learning curve theory is used to judge labour performance and would reflect anticipated learning benefits. Budgeting for manpower needs would be far more accurate if the learning curve concept is adopted and so therefore would safeguard against over manning the production process. Part (b) Y = aXb Y = average time for that (X) number of units or the average cost per unit a = time for the first unit or the cost for the first unit X = the number of units you want to calculate an average time or cost for b = the index of learning (log r/log 2) a = 40 mins, b = -0.415 To work out the time taken for the sixth unit of output: Time for the first 6 units = 40 x (6 to the power of –0.415) x 6 = 114.1 mins Time for the first 5 units = 40 x (5 to the power of –0.415) x 5 = 102.6 mins Time for the 6th unit = 114.1 mins – 102.6 mins = 11.5 mins Part (c) Market penetration or a penetration pricing policy is the decision to charge a very low price in order to capture a larger market share quicker. This strategy is used when introducing a new product into an established and competitive market. This policy also has the following implications:   

The economies of scale for the product should be achieved quicker as market share is obtained rapidly. Competition may be discouraged and leave the market due to the low price being charged. This kind of pricing policy is good for inferior products as they are cheap to manufacture and so costs are low and therefore the price charged can be kept low.

The company should also seek to ensure that all costs are recovered in the long run or over the life of the product. There are two ways of achieving this: 

Lifecycle costing – recording all costs and revenues separately and so being able to monitor if costs are being recovered.

173 | P a g e



Experience curves – show how costs decline over the life of the product because of the learning curve effect.

B1 – 11 XY (CIMA P2 May 2008) Exam tip: The examiner very often gives between 1 and 3 marks for presenting your answer in the report format as requested in the question. So you must do this and not lose these easy marks!

REPORT To: From: Subject: Date:

Managing Director of XY Management Accountant of XY The concept of the Value Chain 21st May 2008

1. Introduction This report is designed to explain the points that should be covered in the presentation on Value Chain and the management of profits generated throughout the chain in XY. 2. The concept of Value Chain Porter grouped the various activities of an organisation into what he called the value chain; he divided the organizations activities into nine types, classified as either primary or secondary activities. These activities incur costs, but in combination with other activities provide customer satisfaction and therefore add value. The main components are as follows: 

Technology - R & D, product design and testing, process design and testing.



Inbound Logistics - Receiving, handling and storing inputs (warehousing, stock control, inbound transport).



Operations - Conversion of inputs (people, materials, machines etc) into the final product or service.



Marketing & Sales - Informing the customer (Marketing mix).



Outbound logistics - Delivery of the actual product or service (storage, outbound transport to customer).



Service - Customer service, repairs, installations, supply of parts/consumables.

174 | P a g e

Value chain analysis is underpinned by the concept of quality which involves the following:    

Commitment to developing processes that achieve high product quality and customer satisfaction Commitment to continuous improvement Involvement of the entire workforce Quality assurance through statistical method a key component

The meaning of quality can be subjective but generally it can mean non-inferiority, superiority or usefulness of the product or service the customer is buying. A common interpretation of quality is "fitness for its use or purpose". TQM is the process of embracing a quality conscious philosophy or culture, as well as adopting quality standards and procedures within an organisation, aiming towards perfection and continuous improvement. Kaizen, the Japanese equivalent to TQM means continuous improvement by small incremental steps. I hope you have found this report useful but should you require any further assistance or have any questions please do not hesitate to contact me. Signed : Management Accountant. B1 – 12 Inventory levels (CIMA P2 May 2008) Part (a) Exam tip: The examiner is only after three cost changes being explained, however we have given you other cost changes that could be equally valid to discuss. If Just-In-Time (JIT) were introduced the following cost changes would result: 

Reduction in stock holding costs to nil as material is only used and bought if there is a demand being the philosophy of JIT.



Reduction in stock holding costs for finished goods and work in progress as all items will be sold to waiting customers.



Reduction in the number of suppliers used to ensure quality and consistency as material would go straight into production and there would be no time for quality control. This would increase the cost of materials however there would equally be a reduction to nil of internal quality control procedure costs.

175 | P a g e



Increase in price of raw materials from suppliers for regular small amounts ordered at short notice, but maybe reduced through the introduction of long-term exclusive contracts with suppliers.



Quality control costs will have to be put into place in the production process to eliminate scrap, re-works and defects which would delay despatch of goods to customers.



Staff training costs to ensure that they are able to use the new machinery efficiently and effectively and be able to maintain them.



Additional planning is required by management to ensure that there are no stock outs as JIT is based on the idea of holding zero stock, and therefore addition planning costs are needed.

Part (b) Total quality management (TQM) is important to a company that operates a JIT production method because TQM means that the product is made to a very high standard reducing internal failure costs such as the inspection and scrap material and also reduction in external failure costs such as repairs and replacement. JIT requires a TQM approach in order to operate holding zero stock levels and only producing if there is demand, being the philosophy of JIT. This would mean whatever is made must be to a high level of quality the first time and every time as there is no spare material to re-make a product that is defectively made. If defective products were made then production would have to stop while more material is sourced and the product remade, this may cause a loss of sales and customer goodwill as we would not be able to deliver the product on time. If the defective product were to be sold on to the customer this would cause the product to be sent back for repairs and customer confidence and goodwill being lost. If TQM were adopted then it would encourage a quality conscious philosophy within staff culture, as well as adopting standards and procedures.

176 | P a g e

B1 – 13 Workshop (CIMA P2 May 2008) Part (a) Y = aXb Y = average time for that (X) number of units or the average cost per unit a = time for the first unit or the cost for the first unit X = the number of units you want to calculate an average time or cost for b = the index of learning (log r/log 2) a = 40 hours, b = log 0.8 / log 2 = -0.3219 To work out the time taken for the 8th batch: Time for the first 8 batches 40 x (8 to the power of –0.3219) x 8 = 163.85 hours Time for the first 7 batches 40 x (7 to the power of –0.3219) x 7 = 149.67 hours Time for the 8th batch = 163.85 hours – 149.67 hours = 14.18 hours Part (b) 

The learning rate has reduced between months 2 and 4 from 75% to 90%. This maybe due to a number of reasons:



An increase in staff turnover and so new staff have to learn how to make these products efficiently and effectively and so require more time than the experienced staff who have left.



A gradual reduction in how much more that can be learned by the existing work force in the manufacturing of the product.



Inferior materials being introduced and so have led to more re-works and longer production times.



Production is no longer in its early stages and so great improvements have already been had.



Motivation and enthusiasm of the existing staff has fallen due to lack of incentives in perhaps skills or pay, and so therefore taking longer to produce the items.

177 | P a g e

Part (c) a = 45 hrs, y = 182.25 hrs / 8 batches = 22.78 hrs on average for 8 batches If we assume that “r” represents the learning rate, then: Batches 1 2 4 8

Average hours per unit 45 45 x r 45 x r x r 45 x r x r x r

Therefore for 8 batches: 45r³ = 22.78 r³ = 22.78 / 45 r³ = 0.506 r = (0.506) to the power of 1/3 r = 0.797 Therefore learning rate is 80% B1 – 14 Out-turn performace report (CIMA P2 May 2010) Part (a) (i) (ii)

Output (batches) Direct labour hours Direct labour cost ($)

Flexed budget 50 68.91 hours (W1) $826.92 (W2)

Actual 50 93.65 hours $1,146

Variance 24.74 hours (A) $319.08 (A)

Direct labour efficiency variance = 24.74 hrs x $12 per labour hr = $296.88 (A) Direct labour rate variance = $319.08 - $296.88 = $22.20 (A) Workings (W1) – Direct labour hours The learning curve ceases once we reach 30 batches, meaning that labour will not get any faster in the production of any more units. Therefore we need to work out the total time for 30 batches and compare with the total time for 29 batches to obtain the time taken to make the 30th batch and use that as the time needed to make any further batches.

178 | P a g e

Using the learning curve formula: Y = aXb Y = average time for that (X) number of units or the average cost per unit a = time for the first unit or the cost for the first unit X = the number of units you want to calculate an average time or cost for b = the index of learning (log r/log 2) a = 10 hours, b = -0.5146 Work out the average time for 30 batches: Y = 10 x (30 to the power of –0.5146) = 1.737 hours Total time for 30 batches = 1.737 x 30 = 52.11 hours Work out the average time for 29 batches: Y = 10 x (29 to the power of –0.5146) = 1.768 hours Total time for 29 batches = 1.768 x 29 = 51.27 hours Time for the 30th batch = 52.11 hours – 51.27 hours = 0.84 hours Total time for 50 batches = 52.11 hours + (20 batches x 0.84 hours) = 68.91 hours (W2) – Direct labour cost 68.91 labour hours x $12 per labour hour = $826.92 Part (b) 

The original budget did not take into account the revised expectations of the learning curve for direct labour and therefore making comparisons is meaningless.



The original budget was not adjusted or flexed for the actual level of output to obtain a fair basis of what should be expected to be used in terms of resources and costs.



The revised out-turn performance report analyses the labour cost variance further into the efficiency variance and labour rate variance, thus allowing improved understanding of the real cause of the variance and then being able to assign responsibilities to the appropriate managers.

179 | P a g e

B1 – 15 PQ (CIMA P2 May 2010) (i) Selling price PQ’s product being a consumer electronic product will have been a product that customers are willing to pay a premium for but this will not be sustainable as competitors will eventually enter the market. This will be seen during the growth stage as competition begins to enter the market, PQ will have to reduce its selling price; however PQ will still continue to increase contribution through economies of scale. During the maturity and decline phase PQ will have to continue to reduce the selling price of their product in order to stay competitive, as closer more competitive substitutes will enter the market produced by other manufactures, these substitutes would have been created by manufactures purchasing PQ’s product and reverse engineering the product. This approach will aim to sustain demand and contribution from PQ’s product as long the selling price is greater than the marginal cost of manufacturing the product. The cash flow will be used to develop other products in development. In the decline phase PQ will further reduce the selling price of the product and it will eventually cease as there will be no extra benefit from this product as the product will have fallen out of favour with customers, and any of the product left will be sold at a vastly discounted selling price to clear it out of PQ’s inventory. The new more advanced replacement product will have been fully introduced. (ii) Production costs There will be a continued reduction in unit production costs of this product during its growth stage but unlikely to see any further reductions beyond this stage. This will be attributable to economies of scale being enjoyed by the company through mass production techniques, such as below: 

Learning curve effects enjoyed by the workforce because the production is labour intensive, repetitive and most of the staff is retained over the long term.



Technical efficiencies learned by all departments in the manufacture of the product, especially the production department through improved understanding of how material and machines can be used more efficiently to maximise production.



Stock control systems will be more efficient if the company employs a JIT approach to stock management. This will reduce stock holding costs to nil and material is only used or bought if there is a demand.



Discounts will have been received as the company would buy more and more bigger batches of the raw material throughout the product’s life cycle.

180 | P a g e



Total quality management techniques will be used which means that the product will be made to a very high standard reducing internal failure costs such as the inspection and scrap material and also reduction in external failure costs such as repairs and replacement.

During the maturity phase there is unlikely to be any further significant cuts in unit variable costs, and in the decline phase the unit production cost will begin to increase. This is expected as the demand for this product will be reduced significantly as it will have fallen out of favour with customers, and therefore less is being made and as a result fewer materials needed. The lower prices on raw materials cannot be enjoyed because of the reduced quantities bought by the company and hence the increase in unit production costs. In addition production costs will increase due to machine breakdowns and inefficiencies. (iii) Selling and marketing costs During the growth stage there will be much reduced expenditure on selling and marketing costs as there will be wide customer awareness of the product already and a minimal amount of expenditure will be required to reinforce the continued customer awareness of the product. During the maturity stage such costs will be reduced further as the product is being more sold on reputation and word of mouth. Selling and marketing costs are not bringing any further benefit. During the decline stage selling and marketing costs will cease as the product will fall out of favour with customers and will mainly become obsolete and production of which will also cease. B1 – 16 Timber products (CIMA P2 May 2010) Part (a) (i) (ii) Month Demand Basic Inc/(Dec) Closing Ave Std Hrs Production Inventory Inventory Inventory Std Hrs Std Hrs Std Hrs Std Hrs 1 3,100 3,780 680 680 340 2 3,700 3,780 80 760 720 3 4,000 3,780 (220) 540 650 4 3,300 3,780 480 1,020 780 5 3,600 3,780 180 1,200 1,110 6 4,980 3,780 (1,200) 0 600 Total

Inventory Holding Costs ($6 per month) 2,040 4,320 3,900 4,680 6,660 3,600 25,200

Overtime Costs if using JIT ($15 per hour) (W1)

3,437.55

18,750.00 22,187.55

The total production cost savings are $25,200 - $22,187.55 = $3,012.45. 181 | P a g e

Workings (W1) – Overtime costs Month 3 = 220 std hours / 0.96 = 229.17 hours x $15 per hour = $3,437.55 Month 6 = 1,200 std hours / 0.96 = 1,250 hours s $15 per hour = $18,750 Part (b) Only 2 factors are needed to be explained from the following: 1. There needs to be close relationships and contractual agreements with the suppliers which would need to be maintained throughout, since no inventory will be kept at XY for urgent requests such sales or defective items. 2. Smaller and more frequent deliveries will need to be planned and co-ordinated to coincide with production needs. The supplier may not have the logistics to support XY. 3. Higher quality machines will be needed with regular maintenance to avoid delays. 4. There will need to be involvement and training of staff to maintain flexibility of working hours and skills. 5. Staff need to take responsibility of their quality and so they need to be encouraged and motivated to do so. B1 – 17 LMN (CIMA P2 May 2010) Part (a) Performance within an organisation should be focused on assessing what can be controlled by divisions or individuals and omitting any items which are uncontrollable. However this is clearly not the case here as there are issues the divisional directors are responsible for but are not within their control. Three issues are as follows: 

The investment decisions that divisional directors are responsible for maybe limited due to the $100,000 threshold. It is not clear as to the size of the divisions and therefore it cannot be ascertained with certainty if divisional directors can freely make meaningful independent decisions.



Head office costs are apportioned on an arbitrary basis with no consideration for the activities or costs expended in each of the decisions; furthermore the divisions do not have control over their own efficiency of resources that they use.

182 | P a g e



The transfer prices are enforced by head office and the divisional directors have no input into their calculations. It is not clear whether head office is imposing on divisions to transfer internally or whether they can decide themselves to purchase internally or externally. There is also no information on the extent of internal demand for goods and services.

Part (b) Activity based costing (ABC) will look in more detail about what caused the head office costs to be incurred and will seek to work out many ‘cost drivers’ (activities). A cost driver is any factor that causes a change in the cost activity, so it is important to identify a causal relationship between the cost driver and the cost. So for example there maybe head office costs which do not relate to any activities of the divisions but are administrative expenses to support the organisation as a whole. These should be ignored as they cannot not be affected by decisions undertaken by the divisions. LMN must only look at costs which would occur due to the activities of the divisions, this may mean including other costs and removing others. This would then result in a fairer way to assess the divisions. This would allow more efficient management of resources by understanding what drives costs incurred by divisions. There would be better costing information for planning and control for example and how different products, customers or distribution channels consume different resources. Furthermore, more realistic and competitive pricing to cover overheads and better profitability analysis because of improved accuracy over costs. B1 – 18 Production manager (CIMA P2 Nov 2010) Part (a)

Output Direct labour hours Direct labour cost

Flexed budget 560 4,480 $67,200

Actual output 560 3,500 $57,750

Revised flexed budget 560 1,712 (W1) $25,680

Planning variance = flexed budget – revised flexed budget Planning variance = $67,200 - $25,680 = $41,520 (F) Labour efficiency variance = (actual hours – revised flexed hours) x std cost per hr Labour efficiency variance = (3,500 -1,712) x $15 = $26,820 (A) Labour rate variance = actual hours at std rate – actual cost Labour rate variance = (3,500 x $15) - $57,750 = $5,250 (A)

183 | P a g e

Workings (W1) – Direct labour hours Using the learning curve formula: Y = aXb Y = average time for that (X) number of units or the average cost per unit a = time for the first unit or the cost for the first unit X = the number of units you want to calculate an average time or cost for b = the index of learning (log r/log 2) a = 8 hours, b = -0.1520 Work out the average time for the first 560 units: Y = 8 x (560 to the power of –0.5146) = 3.057 hours Total time for 560 units = 3.057 x 560 = 1,712 hours Part (b) Target costing is a strategy which seeks to the selling price of a product at the market price which consumers are willing to pay, being the price that the product should be sold for in the market. Then deducting a desired level of benefit or profit for the organisation in order for the manufacture to be commercially viable, and then the product be manufactured within the value left over thereby becoming the budgeted costs or target costs. Market price to achieve desired market share TARGET COST (balancing figure) Desired profit

XX (XX) XX

Learning curves is an important part of a target costing strategy as it will help in reducing costs within the business. It is only applicable to those businesses that have a labour intensive operation where savings can be made through experience and efficiencies. In a machine intensive operation these savings are limited as machines tend produce at the same rate. Businesses can achieve target costs once a certain level of activity has been achieved and so therefore for lessons can be learned and applied to standard costs once it is known the learning capacity of the labour force.

184 | P a g e

B1 – 19 CAL (CIMA P2 Nov 2010) Part (a) Quality conformance costs are those costs which are spent to try to achieve a standard or target, such regulations or functional specifications of a product for a customer. There are two main types of quality conformance costs: 

Prevention costs – these are spent to try and ensure the product is made to detailed specifications and regulations. For example training staff, customer surveys, supplier reviews and investment in machines.



Appraisal costs – these are spent to understand how well a process has performed and corrective action is taken if needed subsequently. For example measuring equipment, inspections and tests, product quality audits.

Quality non-conformance costs are quality failure costs that are needed to correct products, as they did not meet expectations or target. There are two main types of non-conformance costs: 

Internal failure costs – these are quality failure costs before the products or services have been transferred to the customer. For example re-inspection of goods, losses or scrapping of materials and finished goods, additional administrative costs.



External failure costs - these are quality failure costs before the products or services have been transferred to the customer. External costs should be avoided as they expose poor manufacturing abilities to customers. Examples are administration of customer complaints, administration of customer services, product liability claims, repairs and replacements, lost goodwill and reputation.

It is clear from the scenario that CAL from a quality perspective provides middle range quality of solar panels, as they have competitors who sell at a cheaper price but offer an inferior range of solar panels and others who sell at a higher price but offer a high quality range of solar panels. CAL is losing out on an increase of 25% in its market share due to external failure costs of poor assembly skills by staff.

185 | P a g e

Part (b) (i) Non-conformance cost calculations Cost of replacing faulty goods for free is that we would have to supply not 20,000 items but a further 2% in addition to this demand. Therefore, supply 20,000 x 100/98 = 20,408. 408 items represent the free replacement which would cost CAL $45 per unit, therefore a total of $18,360. Other non-conformance costs include the lost increase of market share by 25%. This would result in 20,000 x 25% = 5,000 extra lost sales. This would give a lost contribution of 5,000 x $15 per unit = $75,000. Total non-conformance costs = $18,360 + $75,000 = $93,360 Part (b) (ii) If through an inspection it was found that some solar panels were faulty and not fit for customer consumption then the lost sale could be avoided as well as the delivery charge. The item would still be faulty and the only cost incurred would be internal failure costs. The maximum savings will therefore be $93,360 less the internal costs before delivery being 408 units x $40 per unit = $16,320, giving a net saving of $77,040. B1 – 20 QW (CIMA P2 Nov 2010) Part (a) QW currently employs a form of just-in-time (JIT) whereby products should only be produced if there is an internal or external customer waiting for them. QW manufacture to specific customer order and so there is no finished goods stockpiled in the company but they do however hold 1 day’s stock of raw materials to meet demand if suppliers are not able to deliver. The system also encourages efficient work of staff as delays may result in lost sales. The proposed system aims to procure items to meet constant rates of production. This would mean holding minimum levels of raw materials as well as finished goods to meet fluctuations in demand. This is very different to the current system being JIT which aims to always hold a zero or near zero stock level.

186 | P a g e

This system will ensure demand is met but would lead to inefficient production and obsolete finished goods. This is because managers would tend to produce more goods than is necessary to meet demand. This system would also lead to other holding costs such as damage, deterioration, administration, security and interest costs. Part (b) It is essential that a JIT system is underpinned by TQM. This is because under a JIT system if an item is discovered by the customer as faulty then the company will not be in a position to replace immediately as it does not hold any stock. It would have to manufacture the item again and so there will be a delay to customers who may not be happy about this. The delay may slightly less with QW as they hold 1 day’s worth of raw materials but nevertheless there will be waiting time for customers. In a constant rate production system TQM is not as important because there would be an inventory of both raw materials and finished goods that can be used to replace the faulty item. This system has the advantage of customers not having to wait for their replacement item, however it has the disadvantage of greater costs for holding greater amounts of inventory and more importantly the company moving away from a focus on quality of product resulting in the long term loss of customer goodwill and difficulties in convincing employees that quality is important. B1 – 21 Accountancy services (CIMA P2 Nov 2010) Cost driver rates:

Accounts preparation and advice

$580,000 / 18,000 hrs = $32.22 per hr

Requesting missing information

$30,000 / 250 times = $120 per request

Issuing fee payment reminders

$15,000 / 400 times = $37.50 per reminder

Holding client meetings

$60,000 / 250 meetings = $240 per meeting

Travelling to clients

$40,000 / 10,000 miles = $4 per mile

187 | P a g e

Client Accounts preparation and advice Requesting missing information Issuing fee payment reminders Holding client meetings Travelling to clients Total costs

A $ 32,220 480 75 960 600 34,335

B $ 8,055 1,200 300 240 2,400 12,195

C $ 10,955 720 375 480 0 12,530

Total costs on old basis (W1)

40,280

10,070

13,695

Client fees – new basis Client fees – old basis Difference

41,202 48,336 (7,134)

14,634 12,084 2,550

15,036 16,434 (1,398)

Workings (W1) – Total cost on old basis $725,000 / 18,000 hrs = $40.28 per hr B1 – 22 PT (CIMA P2 May 2011) (i) Growth stage Unit selling prices Due to the very short expected life cycle, during the growth stage as competition begins to enter the market, PT will have to reduce its selling price; in order to stay competitive as substitutes will be being produced by competitors. These substitutes would have been created by manufacturers purchasing PT’s product and reverse engineering the product. This approach will aim to sustain demand and contribution from PQ’s product as long the selling price is greater than the marginal cost of manufacturing the product. The cash flow will be used to develop other products in development. Unit production costs There will be a continued reduction in unit production costs of this product during its growth stage but unlikely to see any further reductions beyond this stage. This will be attributable to economies of scale being enjoyed by the company through mass production techniques, such as below:

188 | P a g e



Learning curve effects enjoyed by the workforce because the production is labour intensive, repetitive and most of the staff is retained over the long term.



Technical efficiencies learned by all departments in the manufacture of the product, especially the production department through improved understanding of how material and machines can be used more efficiently to maximise production.



Stock control systems will be more efficient if the company employs a JIT approach to stock management. This will reduce stock holding costs to nil and material is only used or bought if there is a demand.



Discounts will have been received as the company would buy more and more bigger batches of the raw material throughout the product’s life cycle.



Total quality management techniques will be used which means that the product will be made to a very high standard reducing internal failure costs such as the inspection and scrap material and also reduction in external failure costs such as repairs and replacement.

(ii) Maturity stage Unit selling price The selling price would be unlikely to be reducing any further because the product would have now established itself in the market and reached a price that could be maintained until the end of the life of the product. Unit production costs During the maturity phase there is unlikely to be any further significant cuts in unit variable costs, and costs may begin to increase. This is expected as the demand for this product will be reduced significantly as it will have fallen out of favour with customers, and therefore less is being made and as a result fewer materials needed. The lower prices on raw materials cannot be enjoyed because of the reduced quantities bought by the company and hence the increase in unit production costs. In addition production costs will increase due to machine breakdowns and inefficiencies. B1 – 23 TQM and JIT (CIMA P1 Pilot Paper 2005) Note: Only FOUR reasons are required – SIX possible reasons are given below. The TQM philosophy is cultural as well as systems led. It emphasises the importance of continuous improvement and staff working closer together.

189 | P a g e

 

Get things right first time e.g. aim for zero defects. Eliminate waste and inefficiency.

The JIT philosophy requires that products should only be produced if there is an internal or external customer waiting for them; it aims ideally for zero stock. The following reasons explain why TQM is important within a JIT environment. 1. Less equipment downtime and major stoppages in production give greater efficiency of production flow to help meet customer orders quicker. 2. Less reworks, scrap and wastage will ensure faster lead times to meet customer demand. 3. Avoids sub-standard material from suppliers or sub-standard finished goods holding up production. 4. More effective teamwork and training of staff will help to improve flexibility to meet customer orders quicker. 5. Helps staff diagnose and rectify problems quicker. 6. Improved service to customers. In conclusion within a JIT environment lead time is paramount due to the absence of stock e.g. a chase demand strategy of meeting customer demand. TQM will help improve response times by a more efficient and flexible organisation. B1 – 24 Standard costing (CIMA P1 May 2006) Tip: The question requires only three reasons why standard costing may not be appropriate in a modern business environment. The solution below gives four possible answers. Sometimes hard to define an ‘attainable standard’ especially with the complexity and diversity of modern manufacturing. Traditionally manufacturers produced a small range of products using mass production techniques. Nowadays products have shorter lifecycles and different batches of many different products can be produced, therefore standard costing is more complex as well as less useful due to the shorter lifespan of products. With more automation within operations and less human intervention, standards are becoming less valuable as information. Automation produces greater uniformity and consistency of products produced. This places less value on the use of variances e.g. labour variances, as well as less frequency and materiality of variances occurring. Standard costing is an internal not external control measure. Improvement within organisations needs to consider competition, customers and other global environmental factors, due to greater intensity of competition. Nowadays external benchmarking and value analysis is considered more useful to improve an organisation and its products. Dynamic environments changing frequently do not support the use of standard costing due to greater flexibility required. 190 | P a g e

Modern manufacturing techniques such as TQM and quality circles, mean as manufacturers aim for near perfection, the frequency and materiality of variances should not occur so often. Today focus is more on quality and customer satisfaction not the minimisation of cost. B1 – 25 Marginal v throughput (CIMA P1 May 2006) Both marginal costing and throughput accounting (TA) concentrate on the maximisation of contribution; however each approach has a different definition of variable cost and therefore contribution. Marginal costing defines variable production cost as material, labour and variable production overhead and values inventory using this definition. The throughput accounting approach defines variable cost as material cost only and values inventory using this definition. TA assumes the only true variable cost when calculating throughput contribution is the material and component cost only of making a product. The TA method is perhaps more in line with the modern thought. Traditionally when marginal costing was invented, staff were paid using piece work schemes and more flexible hiring and firing of staff existed at this time. Nowadays labour could be assumed a fixed not variable cost for a manufacturer. The TA method, according to the theory of constraints, identifies a systems bottleneck (most limiting factor) that is restricting the flow of throughput contribution and focuses attention on exploiting or alleviating it. This is similar to the limiting factor approach using marginal costing. Both marginal costing and throughput accounting make no attempt to ‘absorb’ fixed production overhead into a standard cost unit or the income statement. They both treat production overhead as a period cost only and charge this entirely to the income statement for each period. B1 – 26 MRPS (CIMA P1 May 2007) A manufacturing resource planning system is an information system which provides a list of parts and materials required for the type and number of products entered and also the planning of labour types or skills or grade required, machine type/hours, priority of other production and bottlenecks present for the quantity required. It can produce the budgeted cost for each batch or product to be produced as well as aid effective production planning and control.

191 | P a g e

In order to effectively support a standard costing system there must be master files set up containing information about the standard costs, quantities and time needed to manufacture products. Separate master files for production, materials in stock, lead times and labour and machine time are needed and then performance can be measured through variance analysis. B1 – 27 JIT (CIMA P1 May 2007) The JIT philosophy requires that products should only be produced if there is an internal or external customer waiting for them. Traditionally manufacturers stockpiled and were not concerned whether or not there was any demand for the products. The main aim was to have enough stock for each stage of production. This would be able to sustain any sudden surges in demand but as a result of this there would be an excess of raw materials, work in progress and finished goods being carried. JIT aims ideally for zero stock e.g. raw materials delivered immediately at the time they are needed, no build up of work-inprogress, finished goods only produced if there is a customer waiting for them. It aims to try to achieve this by doing the following: 1. 2. 3. 4. 5. 6. 7.

Closer relationships with suppliers required Smaller more frequent deliveries need to be managed in order to produce Cell manufacturing Reduction in set up time (eliminating non value added activities) Higher quality machines perhaps multi-purpose with regular maintenance Involvement and training of staff to maintain flexibility Elimination of defects

B1 – 28 Key features of TQM (CIMA P1 May 2008) There are several key features that are present in an organisation that uses TQM and the list below is not exhaustive. Any three of these would be adequate in answering the question: 

The company will establish senior management commitment and support for TQM.



Processes are put in place to facilitate the adoption of standards and procedures (such as ISO 9000 certification) to monitor and control quality. This is often achieved by benchmarking of competition and investment in employees.



Production to achieve zero defects and eliminate waste. This would be achieved by ensuring that adequate time is spent in manufacturing the products correctly the first time.

192 | P a g e



Employees are to look for continuous improvements in all the processes, resulting in improved productivity, efficiency and less idle time from work processes undertaken.



There is greater investment in quality assurance such as training of staff to ensure high standards, suppliers guarantee high quality, assurance standards and procedures adopted (feed forward control).



The company manufactures products which are more customer focussed achieved through the use of added value activities such as continuous feedback from customers and the reduction of customer complaints.



Stock being held by the company will be zero and there will be no work in progress as products are being made only if there is an external or internal demand.

193 | P a g e

Solutions – Section B Part B Cost Planning and Analysis for Competitive Advantage B2 -1 The Q organisation (CIMA P2 May 2005) Part (a) Stages in the life of a product will include: Introduction e.g. customers unaware, high initial research and development and marketing cost to create and market the new product Growth e.g. awareness increases and industry size increases as more and more customers make purchases of the product, new features to improve or differentiate the product as more competition enters Maturity e.g. product development, modifications and improvements to extend the products life. Mature and a more saturated market as industry growth slows down and demand reaches a limit, greater production efficiency/economies of scale obtained by this stage. Decline e.g. obsolesce of product, usually replaced by a better or newer versions or other broader substitutes

Sales

Time Introduction High unit costs. Extensive sales promotion

Growth Competitors enter market. Unit costs fall

Maturity Profits good. High sales promotion

Decline Overcapacity in industry. Some players leave market

Senility Product possibly retained to enhance product portfolio

194 | P a g e

Product life spans can be from 1 week to over 100 years. During an introduction of a new product, the decision is either to charge a very low price in order to capture a larger market share quicker (market penetration) or to initially charge a high price, achieve low volume but earn a larger profit per unit sold (market skimming). Market skimming It may extend the life of the product life-cycle. Q sells high technology products and is also a respected manufactured worldwide as a result there maybe an expectation by customers to pay a premium for these products. Q could also use this approach as it is good for innovative high quality products where little competition exists initially. Therefore a premium can be charged without reducing demand for it too much. There is always a choice of lowering the price later on when competition comes it to the market with cheaper substitutes. Market penetration This approach will aim to achieve a wide customer base as quickly as possible by setting the price very low. It means that because if its affordability a lot of people will be able to purchase it, as a result of the first 2 stages are quickly achieved in the product life cycle and also achieve economies of scale far quicker. It will also be more difficult for other companies to enter the market as they would have to compete on this low price and may struggle to make a profit. Recommendation It would be advisable for Q to start with a market skimming policy because the products they are selling are high technology items in high demand, and also manufactured from a well respected company. Once competition has entered their market with close substitutes then Q should adopt a market penetration policy to maintain its market position. Part (b) (i) We know that at 10,000 units the VC affected by the learning curve rate is £30, and at 40,000 units the VC affected by the learning curve rate is £22.71. We know that to get from 10,000 units to 40,000 units we have to apply the learning curve rate twice.

195 | P a g e

Therefore if the learning rate is “r” then: £22.71 = £30 x r x r 22.71 = 30r² 22.71 / 30 = r² 0.757 = r² √0.757 = r 0.87 = r Therefore the learning rate is 87%. Part (b) (ii) Work out the variable costs which are affected by the learning curve rate: Number of cumulative units 10,000 20,000 30,000 40,000

Total VC per unit £60.00 £56.10 £54.06 £52.71

VC affected by learning curve £30.00 £26.10 (W1) £24.06 (W1) £22.71

VC not affected by learning curve £30.00 £30.00 £30.00 £30.00

W1 – We can use the formula to work these out Y = aXb Where: a = 30, b = log 0.87 / log 2 = -0.201 Therefore the average variable cost for 2 batches or 20,000 units: Y = 30 (2) (to the power of -0.201) Y = 26.10 The same principle for 3 batches or 30,000 units: Y = 30 (3) (to the power of -0.201) Y = 24.06 Now work out contribution by comparing selling price to variable costs for every demand level and then working out the optimum price.

196 | P a g e

Demand units 10,000 20,000 30,000 40,000

Selling price per unit £100 £80 £69 £62

VC per unit £60.00 £56.10 £54.06 £52.71

Contribution per unit £40.00 £23.90 £14.94 £9.29

Total Contribution £400,000 £478,000 £448,200 £371,600

The price that optimises contribution is £80 in the initial launch phase of 20,000 units. Part (b) (iii) We need to work out target contribution from the given target profit of £30,000 and then we can work out how many units we need to sell to achieve the target contribution. If target profit is £30,000 and fixed costs are given as £15,000 then target contribution must be £45,000. The initial launch phase is 20,000 units, and so we need to work out contribution per unit after this and then we can divide this by target contribution to know how many units to sell. The problem is that we have a reducing variable cost due to the learning curve for every batch we produce and so therefore we have to work out the average unit cost for every batch. Ave unit cost between 20,000 and 30,000 units: (30,000 x £54.06) – (20,000 x £56.10) 10,000

=

£49.98

Ave unit cost between 30,000 and 40,000 units: (40,000 x £52.71) – (30,000 x £54.06) 10,000

=

£48.66

Contribution per unit between 20,000 and 30,000 = £57 - £49.98 = £7.02 No. of units that need to be sold to reach target contribution = £45,000 / £7.02 = 6,410 Contribution per unit between 30,000 and 40,000 = £57 - £48.66 = £8.34 No. of units that need to be sold to reach target contribution = £45,000 / £8.34 = 5,396

197 | P a g e

B2 -2 F plc (CIMA P1 May 2005) Part (a) (i) Current system (absorption costing) Absorption of overhead £880,000 ÷ £8m sales value of orders = 11% 11% of overhead will be charged every £1 of sales Order A £1,200 x 11% = Order B £900 x 11% =

£132.00 £99.00

Part (a) (ii) Activity based approach Cost drivers Invoice processing (25% x £280,000) ÷ 8000 order/invoices = (75% x £280,000) ÷ 28000 invoice lines =

£8.75 per invoice £7.50 per invoice line

Packing Small package (given) Large package (given)

£25.00 per small package £32.00 per large package

Delivery (£40,000 ÷ 1000 deliveries) ÷ 6 small packages = (£40,000 ÷ 1000 deliveries) ÷ 12 small packages =

£3.33 per small package £6.67 per large package

(£180,000 - £40,000) ÷ 350000 miles =

£0.40 per mile

Other overheads £200,000 ÷ 8000 orders =

£25.00 per order

198 | P a g e

Order

A £

B £

Invoice processing £8.75 per invoice £7.50 per invoice line

£7.50 x 2 =

8.75 15.00 £7.50 x 8 =

8.75 60.00

Packing

small package

25.00 large package

32.00

Delivery

small package £0.40 x 8 miles

Other overheads £25 per order

3.33 large package 3.20 £0.40 x 40 miles

6.67 16.00

25.00

25.00

80.28

148.42

Part (b)

Tip: A good report format will make your report be well presented and provide good structure for your answer. Part (b)(i) in short requires the advantages and disadvantages of ABC in comparison to absorption costing, so if you know what these are it will be the backbone to answering this part to the question. A good approach also would be to think of good examples from the calculations in part (a) to provide value within your report e.g. why the proposed system has led to a better or worse understanding of how costs are driven by the different resources consumed, such as administration or transport.

REPORT To: Management of F Plc From: Management Accountant Date: DD/MM/YY 1.0 Introduction The purpose of this report is to assess the strengths and weaknesses of the proposed activity approach for F Plc and recommend actions that the management might consider in the light of the new data produced using the activity based approach.

199 | P a g e

2.0 Strengths of the proposed activity based approach The current approach using absorption costing, took the total budgeted selling a distribution cost for the period and divided this by the budgeted list price of drugs supplied, in order to find a single overhead absorption rate. The new activity based costing approach looked in more detail about what causes the selling a distribution cost for the period to be driven. It worked out many ‘cost drivers’ rather than just sales value. It therefore obtained a more accurate way of looking at how overhead is driven and should therefore give a more accurate picture when costing and pricing for different orders of pharmaceuticals. For example order A using absorption costing would charge £132 of overhead, where as with the proposed system of ABC would charge only £80.28. This charge is lower due to recognising that order A consumes less resources of F Plc to deliver it.  

It required less invoice processing than order B e.g. only two lines on the invoice, meaning less resource of staff or equipment used to administrate it. It was a small package and therefore costs less to transport per package, due to a lorry having the ability to carry more packages per load. It also required less distance to be transported and this was reflected also in the cost of the order.

The strengths of the proposed activity based approach  More efficient management of resources by understanding what drives selling and distribution overhead incurred. For example management by ensuring all lorries carry full loads, can help reduce the frequency of deliveries to help minimise transport cost.  Better costing information for planning, control or decision making. An outsourcer could supply an activity externally and this can be benchmarked to internal costs for a decision to be made. A decision to accept or reject a price offered by a customer for a long-term contract, can be compared to the cost of supplying it more realistically.  More realistic pricing to customers in future, to cover overhead being incurred by them.  Better profitability analysis of different customers and their orders. 3.0 Weaknesses of the proposed activity based approach To implement a new system even though it could be useful, can be time consuming and a more expensive system to run and maintain, due to the greater complexity of how overhead is now allocated to orders. Investment will be required for ‘bespoke’ information systems, does this additional cost justify the benefit?

200 | P a g e

It still assumes fixed overhead is driven by volume as does absorption costing. The nature of fixed overhead is that it is ‘fixed’ it is assumed it does not vary significantly with an activity level of any kind. As an example it would be naive perhaps for management to believe that less administration of an order will mean less overhead incurred, given that most of this cost is staff overhead e.g. fixed salaries regardless of whether staff are processing orders or idle. 4.0 Recommendations The current approach although relatively simple, does not truly reflect the way an order consumes the resources and therefore overhead of F Plc. Charging 11% of the list price of an order does little in the way of recognising that the larger a package, the more invoice processing it requires and the longer the distance it needs to be carried, then the higher the cost of supplying it. It also requires the full £8 million list price of pharmaceuticals to be supplied in order to recover the £880,000 overhead fully. My recommendations would therefore be as follows. 1. If the new system is not costly to introduce then it perhaps should be, but before this decision is made it is imperative that research is done fully in terms of price comparisons between the current and proposed system of charging customers. Just because it reflects better consumption of resources within the price ultimately charged, does not mean it should be introduced especially if the new system will create prices well above prices charged for similar orders by the competition. Customers may perceive the idea as far too complex. 2. If the system is considered too costly or complex by the management or customers, then perhaps the following alternative should be considered. a) Charging a standard additional surcharge to a customer if delivery is outside a certain radius e.g. within 10 miles, 10 to 25 miles and above 25 miles. b) Charging a standard additional surcharge to a customer for multiple orders or larger packages handled. If this is this case the introduction of a relatively less complex and costly system can be maintained. At least customers will have a better idea when it comes to how much they will be charged or quoted. However again research needs to be done, it is important that standard surcharges when levied, do not put customers off, especially if competition does not do the same.

201 | P a g e

I would not recommend using the proposed ABC system as a way of pricing orders for customers, as I consider it too complex, but I would recommend it perhaps internally within F Plc when it comes to understanding how to reduce consumption of resources and improve efficiency. I would also recommend it for customer profitability analysis when it comes to making better management decisions about whether or not to supply a customer e.g. is the customer losing money for F Plc? Such a system can be maintained through a simple spreadsheet when doing so and therefore will not require significant investment. Signed Management Accountant B2 – 3 KL (CIMA P2 Nov 2006) Part (a) Total or full cost plus pricing This approach takes the variable costs involved in manufacturing the product and adding an apportioned amount of fixed overhead using an absorption costing or activity based costing system. This is known as the total or full cost. Then by adding a % mark up which will represent the profit earned from selling the unit this will give us the total or full cost plus price. This is the selling price of the unit. Advantages  Ensures fixed overheads are recovered in the long-term as it is included in the selling price.  Suitable if you already have an absorption costing system in place but without such a system it would be difficult to know how much to absorb of the fixed overheads in each of the different products.  Good for long-term profitability analysis as fixed overheads would be aimed to be recovered fully in all products sold.  Adequate profits are made without the need to seek out a selling price that would maximise profits as a profit mark up is included in the price.  Easy and quick to produce prices for products and therefore can be delegated to junior staff. Disadvantages  The selling price selected may not suit the market price and demand conditions as it ignores competition.  The method of sharing fixed overhead over the product range is subjective as a fixed overhead cost driver must be selected to apportion the costs. The driver may not be representative of how fixed overheads were actually spent.  Different organisations will use different overhead absorption methods for example an activity based costing approach versus a traditional absorption costing approach. 202 | P a g e



Apportionment of fixed overhead tends to be irrelevant in decision-making this is because they are historical costs and do not effect the net benefit to the company. The company should look at marginal revenue and marginal costs only as these will change as a result of our decision.

Marginal cost plus pricing This approach is more likely to be used with traders within the retail sector or when a company is at full capacity due to a limiting factor (relevant costing or minimum pricing). Marginal cost plus pricing recognises that fixed cost are not relevant to decision making and therefore ignores it in its pricing. Therefore to achieve the selling price under this approach we add the marginal or variable costs plus a % mark up together. The MD’s concern over whether sales may be lost if prices are set too high is important and using a marginal cost plus pricing approach may mean that prices are set lower and are more competitive whilst still earning some contribution towards fixed overheads. Advantages   

There is a far better understanding of cost as we are only looking at those costs that change as a result of our decision that being marginal costs only. It is useful when we only have information regarding variable costs only. Good for short-term decision-making as it ignores historical fixed costs.

Disadvantages   

Ignores fixed overhead in the long-term and therefore this would result in long term losses. Fixed overheads need to be recovered over the long term; however the mark up may be large enough to include this and profit. The selling price selected may not suit the market price and demand conditions as it ignores competition. It maybe all you can use due to the inability to apportion fixed overhead.

Part (b) Product Labour hours per unit Budgeted production Total labour hours

W $40 / $10 = 4 15,000 15,000 x 4 = 60,000

X $30 / $10 = 3 24,000 24,000 x 3 = 72,000

Y $50 / $10 = 5 20,000 20,000 x 5 = 100,000

Total labour hours = 60,000 + 72,000 + 100,000 = 232,000 Overhead absorption rate = $1,044,000 / 232,000 = $4.50 per labour hour

203 | P a g e

Material Labour Overhead Total cost per unit

W $35 $40 $4.50 x 4 = $18 $93

X $45 $30 $4.50 x 3 = $13.50 $88.50

Y $30 $50 $4.50 x 5 = $22.50 $102.50

Part (c) W 15,000 Production levels 15,000/500 = 30 Number of batches 30 x 4 = 120 Supplier orders 15,000 x 5 = 75,000 Machine hours

X 24,000 24,000/400 = 60 60 x 3 = 180 24,000 x 8 = 192,000

Y 20,000 20,000/1,000 = 20 20 x 5 = 100 20,000 x 7 = 140,000

Overhead absorption rate: Material ordering = $220,000 / (120 +180 + 100) = $550 per order Machine set up = $100,000 / (30 + 60 + 20) = $909 per batch Machine running = $400,000 / (75,000 + 192,000 + 140,000) = $0.98 per hour General facility = $324,000 / (75,000 + 192,000 + 140,000) = $0.80 per hour Cost per unit Material ordering Machine set ups Machine running General facility Direct material Direct labour Total cost per unit

W ($550 x 120) / 15,000 = $4.40 ($909 x 30) / 15,000 = $1.82 ($0.98 x 75,000) / 15,000 = $4.90 ($0.80 x 75,000) / 15,000 = $4 $35 $40 $90.12

X ($550 x 180) / 24,000 = $4.13 ($909 x 60) / 24,000 = $2.27 ($0.98 x 192,000) / 24,000 = $7.84 ($0.80 x 192,000) / 24,000 = $6.40 $45 $30 $95.64

Y ($550 x 100) / 20,000 = $2.75 ($909 x 20) / 20,000 = $0.91 ($0.98 x 140,000) / 20,000 = $6.86 ($0.80 x 140,000) / 20,000 = $5.60 $30 $50 $96.12

Part (d)

Current system ABC Difference

W $ 93.00 90.12 0.12

X $ 88.50 95.64 (7.14)

Y $ 102.50 96.12 6.38

It seems that for product W using Activity based costing (ABC) or the current system produces very similar figures for unit cost, however for product Y costs are less under ABC when compared to the current system, but this is not the case for product X where it seems that there are more costs allocated under ABC than the current system. 204 | P a g e

ABC looks in more detail about what causes fixed overhead to be incurred and works out many ‘cost drivers’ rather than just labour or machine hours or products produced, all driving overheads to be incurred. It is used in order to obtain a more accurate way of looking at how fixed overhead are driven and should give a more accurate picture when costing products, budgeting or valuing stock. ABC can improve profitability by: 

Managing resources more efficiently through the understanding of what drives fixed overheads.



Thereby better costing information for planning and control e.g. how different products consume different resources or the production of flexed budgets based upon activity based budgeting (ABB).



More realistic and competitive pricing to cover overheads.



Better profitability analysis of product range.

In the case of KL, profitability could be improved as follows: Reducing the number of set ups for product X would reduce set up costs. Currently under ABC $2.27 is allocated to product X for set up costs which is the most out of the product range. Product X has the highest annual production and it would make sense if these batches were larger and thus reduction in set up costs. Product Y may have costs allocated to it under the current system which are not driven by the activities in this product. This is seen by the lower cost per unit under ABC. Using the fairer costs allocation under ABC this would mean that a lower selling price could be charged and thereby increasing sales but still maintaining the same unit profitability. General facility costs are said to be driven by number of machine hours. It is not said in the question what the costs are but they may include maintenance to the building or warehouse, and if this was the case machine hours does seem like an arbitrary driver. It may be more sensible to leave such costs out of the unit cost calculation when looking at profitability as it would only serve to distort the overall cost and reduce the meaningfulness of the figures.

205 | P a g e

B2 – 4 Retail outlet (CIMA P2 Nov 2007) Part (a) Option 1

Weeks 1 to 4 Weeks 5 to 8 Weeks 9 to 12 Weeks 13 to 16

Sales lost None 30% 20% 10%

Weeks 1 to 12 there would be a 10% discount on the price of basic and canned foods.

Basic foods Newspapers Frozen foods Canned foods Lost contribution for 1 week

$ 200 300 950 1,200 2,650

Relevant costs for option 1 Weeks 1 to 4 lost contribution (4 x 2,650) Week 1 to 4 freezers not used (4 x 100) Redecoration costs Weeks 5 to 8 lost contribution (30% x 2,650 x 4) Weeks 9 to 12 lost contribution (20% x 2,650 x 4) Weeks 13 to 16 lost contribution (10% x 2,650 x 4) Discount on selling price weeks 5 to 8 (10% x (800 + 2,400) x 70%) x 4 Discount on selling price weeks 9 to 12 (10% x (800 + 2,400) x 80%) x 4 Discount on selling price weeks 13 to 16 (10% x (800 + 2,400) x 90%) x 4 Total

$ 10,600 (400) 2,500 3,180 2,120 1,060 896 1,024 1,152 22,132

206 | P a g e

Option 2 Relevant costs for option 2 Closure for ½ a week lost contribution ($2,650 - $100) x 0.5 Redecoration costs Product movement costs Lost contribution on frozen foods (W1) Lost contribution on canned foods (W1) Total

$ 1,275 3,500 1,000 2,550 6,840 15,165

Conclusion: Option 2 has the lower losses. Workings (W1) Lost contribution For 6 weeks the floor space will be restricted to 40% of original size = 40% x 400m² = 160m² No reduction in floor space for “Basic Foods” and “Newspapers and Magazines” therefore 60 m² is left for “Frozen Foods” and “Canned Foods”. We should do those products that give us the most profitability that being contribution per m² between “Frozen Foods” and “Canned Foods”. Frozen foods contribution per m² = 950 / 100 = $9.5 per m² Canned Foods contribution per m² = 1,200 / 100 = $6 per m² We should do “Frozen Foods” first and then “Canned Foods” if we have any space left. We can only do “Frozen foods” in complete freezer units. There are 4 freezer units, therefore each unit uses 4 / 100m² = 25m². We can only do 2 freezer units so therefore 50m² is given to “Frozen Foods” and the rest being 10m² is given to “Canned Foods”. Therefore lost contribution: “Frozen Foods” = ($950 - $100) x 50m²/100m² x 6 weeks = $2,550 “Canned Foods” = $1,200 x 190m²/200m² x 6 weeks = $6,840

207 | P a g e

Part (b) Traditional absorption costing takes the total budgeted fixed overhead for a period and divides by a budgeted (or normal) activity level in order to find the overhead absorption rate. Activity based costing (ABC) looks in more detail about what causes fixed overhead to be incurred and works out many ‘cost drivers’ rather than just labour or machine hours or products produced, all driving overhead to be incurred. It is used in order to obtain a more accurate way of looking at how fixed overhead is driven and should give a more accurate picture when costing products, budgeting or valuing stock. ABC is a method which lends itself to a retail company environment where there are a wide range of products being bought and sold which require different conditions in storage and display. For example frozen foods and chilled foods require refrigeration costs where as canned foods do not require this only storage in normal room temperature conditions. Main steps in ABC 

Group types of fixed overhead together, if they are driven by the same type of activity or ‘driver’. These are known as “cost pools”. For example quality control supervisor and quality control rent and overhead, grouped together as both would be driven perhaps by the number of inspections forecast for the period.



Calculate from these cost pools a fixed overhead “cost per driver”, for example similar to an overhead absorption rate when compared with traditional absorption costing.



Absorb fixed overhead by using ‘many cost drivers’ rather than just one or two ‘cost drivers’ or overhead absorption rates as with traditional absorption costing this would give management a better idea as to how different products produced consume resources and therefore overhead.

Activity Material procurement Material handling Quality control

Cost driver No. of purchase orders No. of movements No. of inspections

Advantages of ABC ABC would allow efficient management of resources by understanding what drives fixed overhead incurred.

208 | P a g e

There would be better costing information for planning and control for example how different products consume different resources or the production of flexed budgets based upon activity based budgeting (ABB). When an organisation is formulating their long term strategic decisions, such as product pricing, mix of products, discontinuance, launch or promotion of existing products and the launch of new products, ABC allows for realistic pricing to cover overheads and therefore superior profitability analysis on product ranges. ABC’s strength lies in the fact it allows accuracy over costs and drivers for products and as a result a sensible pricing strategy is achieved. It more specifically gives a good long term understanding of the variable costs being very relevant for decision making. However ABC information must be put into perspective as these are historic costs and cannot be used alone to predict future costs. They should be used as a starting point and other internal and external information should be used to determine future costs. All costs are variable in the long term and subject to change.

209 | P a g e

Solutions – Section A Part C Budgeting and Management Control C1 -1 Solicitors firm (CIMA P2 May 2010) Part (a) In the present budgeting system the senior partner estimates demand for the year while the divisional manager creates the cost budgets to support demand. The cost budgets are submitted for review by the senior partner but any amendments to the budgets are included without consultation with the divisional partners. This illustrates that the senior partners do not really allow divisional partners to participate in the budgeting process as they are not consulted on budget revisions. This can de-motivate staff leading to a decline in their productivity and efficiency as they feel their expert opinions are not important and are simply ignored. The divisional partners would feel that they have wasted their time in creating the budgets and would begin to feel that they are not part of the process to improve the organisation. There is great danger of a divide being created between the senior partner and the divisional partners. The current approach may also see divisional partners start to find excuses not meeting targets just to prove to the senior partner that their budgets were correct and the changes imposed by the senior partner were wrong. Sub-optimisation or dysfunctional behaviour may occur for example the personal objectives of the divisional partner are not aligned with the business objectives. Part (b) Only 2 are needed to be explained from the following:    

Training days per staff – measures the level of investment that the firm has committed to improving staff skills to support the needs of clients. Staff turnover – measures the number of staff leaving the firm indicating that staff are perhaps dissatisfied with the firms approach to supporting the needs of staff. Training costs per staff – measures the amount of money the firm have set aside for staff training purposes. Time taken between client first enquiry and first meeting – measures efficiency and flexibility of the firm to clients needs.

210 | P a g e

C1 – 2 DW (CIMA P2 Nov 2010) Part (a) An annual budgeting system is the process of using existing budgets as a guide to prepare the budget for the next 12 months which normally coincide with financial year end of the company. Rolling or continuous budgeting is when the budget is updated on a regular and frequent basis. The method is to add a further period immediately to the budget when an earlier period has expired, for example if Jan to Mar 2005 is the first three months of the yearly forecast, once this has expired then Jan to Mar 2006 will be created and added. Two of the key differences between these two systems are: 

Rolling budgets always provide a budget for the full period as they get updated, unlike annual budgets which don’t get updated as the periods expire. This enables management and so enables better long term planning.



Rolling budgets are good for adaptive planning, for example there is a greater chance that the budgets will be regularly updated to take account of changes within the environment the organisation is operating within if used.

Part (b) The new depot manager has two concerns that he raises about the current system: Budgets are out of date Annual budgets are set on annual basis and are not revisited on a regular basis. This means that it is not possible to see if the budget is being met or even if current economic circumstances have made the budget out of date and not applicable. Rolling budgets would allow regular comparison to the actual outcomes more frequent update of forecasts where necessary due to unanticipated changes in the economy. This would keep the budgets more accurate, reliable and meaningful. Operational and strategic decisions cannot be taken Rolling budgets always provide a budget for the full period as they get updated and approved my management, unlike annual budgets which don’t get updated as the periods expire. This enables depot mangers to take decisions for the early part of next year.

211 | P a g e

Annual budgets only get updated once a year and proves difficult for depot mangers to make decisions until the budget has been approved by management which maybe close to the year end restricting decisions being taken by depot mangers in the early part of next year. Operational decisions and control must however be dealt with carefully as these entail the day to day actions of the company and any differences in actual performance must be noted to ensure that improvements can be made. Rolling budgets should not eliminate these variances by revising them as it would mean that information about inefficiencies cannot be isolated and improved. Strategic decisions and control would benefit from rolling budgets as it help to ensure that failing strategies are abandoned in favour of successful strategies. Rolling budgets would allow frequent revisits to the strategy to ensure that it is being met and if not then the original strategy being revised. C1 – 3 JYT (CIMA P2 May 2011) Target costing is a strategy which seeks the selling price of a product which consumers are willing to pay, being the price that the product should be sold for in given market forces. Then deducting a desired level of benefit or profit for the organisation in order for the manufacture to be commercially viable, and then the product be manufactured within the value left over thereby becoming the budgeted costs or target costs. Market price to achieve desired market share

XX

TARGET COST (balancing figure)

(XX)

Desired profit

XX

The target cost is normally achieved over the long term using learning and experience curve efficiencies and may take several years and so in the meantime the desired profit is squeezed to compensate. Target costing is not simply a cost reduction exercise but a quality improvement strategy over the long term. It sacrifices short term profitability for long term profitability and combines the use of JIT, TQM, cost reduction, value analysis and benchmarking to achieve the target cost. Kaizen costing is the process of continuous improvement through small incremental steps rather than transformational changes. It also believes strongly in empowerment of employees to enable them to improve operations. Kaizen costing focuses on reducing variable costs of future periods below that of prior periods.

212 | P a g e

     

Human resources are your most valued asset and they should be involved in the search for perfection. They have superior knowledge as they are in the operation. This is the opposite of traditional systems where the managers develop standards. Incremental or gradual improvement. Perfection should be sought all the time and so there is always room for improvement. Cost reduction targets more frequent rather than traditional annual standard costs. Variance analysis used to help with kaizen costing. Investigations carried out if targets not met even if improvements have been made.

Unlike kaizen costing, target costing doesn’t focus on gradual cost reduction but starting from scratch and re-designing methods and processes to achieve large changes. Kaizen costing relies on all employees taking part in the process because it focuses on making small improvements by everyone. Target costing is a strategy implemented before the launch or production of a product unlike kaizen costing which is employed when production has commenced. C1 – 4 DVD (CIMA P2 May 2011) Part (a) (i) The sales mix variance shows the change in the different product lines being sold and the impact it has on profit or contribution.

DVD Blu-ray

Actual sales quantity 3,000 1,200 4,200

Actual sales at budget mix (W1) 2,800 1,400 4,200

Difference

Profit

Variance

200 (F) 200 (A)

$25 $95

$5,000 (F) $19,000 (A) $14,000 (A)

Total sales mix profit variance = $14,000 (A) Workings W1 – Actual sales at budget mix Revised demand for Blu-ray players of 1,500 units should be used in the budget instead of 1,000 units this is because of an unexpected growth in the total market for Blu-ray players. DVD = 3,000 / 4,500 x 4,200 = 2,800 Blu-ray = 1,500 / 4,500 x 4,200 = 1,400 213 | P a g e

Part (a) (ii) Units X (X) X x Standard Profit per unit X

Did sell (actual quantity sold) Should sell (original budget quantity sold)

Sales volume (profit) variance

The sales volume (profit) variance measures the difference between the original and flexed budgeted profit. It measures the impact on profit, when actual sale of units is more or less than the original budgeted sale of units. This method of calculation would be applied when absorption costing is used by the organisation. DVD sales volume variance = (3,000 – 3,000) x $25 = $0 Blu-ray sales volume variance = (1,200 – 1,500) x $95 = $28,500 (A) Total sales volume variance = $28,500 (A) Part (b) The planning variance is beyond the operational control of management and staff for example market size growth of Blu-ray player. The operational variance is normally within the control of management and now more realistic as a yardstick because calculations would include any revisions to standard, in the case of the Blu-ray players, the revised budgeted demand due to changes in the total market share. An example of an operational variance is the changes in the selling prices for the DVD and Blu-ray players resulting in variances which are controllable by management. Planning variances will help highlight variances between those which are controllable and those which are uncontrollable. For example the sales manager’s email boasts a favourable $19,000 variance however it is not strictly entirely caused by his influences of sales, some of it is due to the increase in total market share of Blu-ray players which is not in the control of the sales manager. The budget for Blu-ray players should have been 1,500 units and therefore resulting in an adverse variance. Planning variances help motivate managers and staff, for example avoids staff being blamed for faulty planning and gives a fairer reflection of any operational variances calculated when assessing any operational efficiencies or inefficiencies. Management and staff would be appraised more fairly for any favourable or adverse deviations that are within their control. Planning variances make use of realistic standards in order to measure performance gives better management information for control purposes.

214 | P a g e

C1 – 5 SFG (CIMA P2 May 2011) Part (a) Non-financial performance measures recognise that the constant drive to increase profitability can ultimately be self-defeating and that it is imperative that organisations do not put the needs of shareholders above all else. The primary goal of most profit seeking enterprises is to increase shareholder value through increasing profit; however nonfinancial performance measures are equally as important as they ultimately drive profitability and therefore shareholder value. For example level of customer satisfaction, innovation, quality and morale of the work force, all indicate how well the company is doing things that can lead to future profits. Long-term benefits of using non-financial performance measures will be improved profitability when these performance measures are combined with financial performance measures and are monitored and used for control purposes. Part (b) Exam tip: Please note that the question only asks for two reasons however we have provided a selection of possible reasons. Any two would suffice. 

  

Customer satisfaction surveys – measures the number of complaints or recommendations at the hotel, also giving a rating for service received from the hotel staff and mangers. Cutting back on customer service costs may save money in the short term but will impact the long-term reputation of the hotel and reduced repeat business. Training days per staff – measures the level of investment that the hotel has committed to improving staff technical and interpersonal skills. Staff turnover – measures the number of staff leaving the hotel indicating that staff are perhaps dissatisfied with the hotel mangers approach to supporting the needs of staff. The number of new hotel deals for accommodation and events – The more of these options are available created my by the managers the greater chance of increasing sales and repeat business. It may cost money to set up these deals and some money and in giving discounts or free “taster” days but in the long-term it would increase sales.

215 | P a g e

Part (c) An uncontrollable cost does not mean a manager being assessed should ignore it altogether. As an example if managers recognise there is an interest charged by head office based upon the capital employed used within a division, then to hold them more accountable could help improve efficiency by the minimisation of capital employed. In the case of head office charges, lack of accountability could encourage over consumption of these resources provided centrally. However it would be considered fairer if a manager were not assessed on costs which are not within their own control. This is likely to improve motivation and morale. The way to include non-controllable costs when assessing the performance of the manger on a fair basis is to put them into a separate section with in the report; however it is not always clear when determining controllable and non-controllable costs. Political arguments may ensue which are more subjective than objective when determining controllability. C1 – 6 Feedback and feedforward (CIMA P1 Pilot Paper 2005) An example of a feedback control system is a budgetary control system. This would gather information on past performance from the output of the system e.g. actual financial performance, and compare this to a predetermined standard or plan (budget) using any deviations e.g. variances, as a basis of improving future performance through control action taken. A feedback loop is where the output of a system is measured and fed back as input into the system in order to obtain a desired effect, often done intentionally, in order to control the behaviour of a system. In a double feedback loop, corrective action is not automatically taken. The output of the system is measured, however environmental factors will also be considered, along with internal feedback before any control action is taken. In a single feedback loop, the output is automatically compared to a predetermined standard; any exceptions and control action will be automatically taken. Feedback contrasted to feed-forward control is like closing the door after the horse has already bolted, in other words there is little you can do about it now, except try and rectify the situation to avoid it happening again. Feed-forward control is more prevention than appraisal, controlling a system by making adjustments now to the system in advance before any exceptions occur. It does this by trying to predict what will happen in the future. Feedback can be transformed into feed-forward control by being more proactive and predictive as to what will happen in the future, rather than being reactive or backward looking by historical reflection on the past.

216 | P a g e

C1 – 7 Profit centre managers (CIMA P1 Pilot Paper 2005) Tip: The behavioural aspects of budgeting is concerned with how budgets or standards affect people within an organisation. Poor performance could be attributable to the method of implementing budgets e.g. ignoring the ‘human side’ of participation or the introduction of a standard centrally that is unrealistic or unobtainable can de-motivate staff. A profit centre is when a manager will be accountable for creating sales revenue and controlling cost. The division or centre is assessed on profit earned. Advantages of participation  Greater motivation for the manager and because of this targets are more likely to be met. It will also be more interesting and more involving for the manager increasing job satisfaction.  Participation means that targets are more likely to be accepted by the manager e.g. less conflict due to setting targets themselves.  The manager maybe more up to date in terms of the market environment and therefore the targets they set could be more realistic than if imposed centrally. Disadvantages of participation If managers are asked to participate in setting their own budgets there is the possibility they will include “slack” or “padding” within the budget e.g. have less sales units forecast than could realistically be achieved. This is the inevitable downside of human behaviour e.g. underestimate to avoid blame if they do under achieve. Participation could create a slower process to formulate the budget because more consultation will be required. The manager could be inexperienced financially. Training would be required in order for them to participate effectively. C1 – 8 Balance scorecard (CIMA P1 May 2005)

Tip: Below are many possible answers for each perspective, the question states that you are to provide only one performance measure for each perspective and give a reason to support each measure.

A new approach to strategic management was developed in the early 1990's by Drs. Robert Kaplan (Harvard Business School) and David Norton. The balanced scorecard suggests that we view the organisation from four perspectives.

217 | P a g e

   

Customer perspective e.g. what must we do right for our customers and what do they value? Internal perspective e.g. what must we excel at or improve internally to satisfy shareholders and customers? Innovation and learning perspective e.g. how can we innovate and improve value? Financial perspective e.g. how do we satisfy shareholders and create value for them?

Customer perspective      

Percentage of sales which are repeat business. The number of complaints (analysed by reason). The percentage number of customers complaining. Average time to settle insurance claims. Average of satisfaction ratings from customers e.g. quality of service from 1-5 (5 being the highest). Percentage market share.

The reason for all the above measures is that they would indicate customer satisfaction for the quality of service customers are receiving. Internal perspective    

Percentage number of quotations and renewals delivered to customers within three days of processing their details. Number of complaints from customers regarding inaccurate information, late or lost paperwork. Average time to settle insurance claims. Staff turnover, percentage staff retention or staff absenteeism.

The reason for all of the above measures is that they would indicate failures in the insurance company’s internal processes which may impact on customer satisfaction. High staff turnover indicates the replacement frequently of staff which can impact upon the efficiency and effectiveness of internal activities. Innovation and learning perspective    

Training expenditure or training days per employee. New insurance products launched in the past 6 months, compared with the competition. Percentage of staff suggestions for improvement, actually used by management. Percentage of sales revenue each year from new products launched.

Training expenditure indicates the ‘learning’ or staff development perspective within the insurance company and the remaining three are good measures for innovation. 218 | P a g e

Financial perspective       

Share price growth. Percentage of market share. Dividend per share. Earnings per share. Sales or profit growth. Return on investment (ROI). Economic value added.

All the above measures concentrate on maximising financial value for a shareholder. C1 – 9 Participation in budgets (CIMA P1 May 2005) Tip: Only three circumstances briefly explained are required. Six possible answers are included below. Circumstances where participation leads to poor performance If managers are asked to participate in setting their own budgets there is the possibility they will include “slack” or “padding” within the budget e.g. have less sales units forecast than they could realistically achieve, given a reasonable level of effort. This is the inevitable downside of human behaviour e.g. underestimate to avoid blame if they were to under achieve. Participation could create a slower process to formulate a budget because more consultation is required. This will inevitably increase the cost and time of formulating budgets, leading to budgets which are late or costly to produce. The manager could be inexperienced financially. Therefore they may not be able to contribute effectively if they were to participate. This could lead to inaccuracies when standards or targets are developed. In a stable environment where revenue or costs are certain there would be little benefit in allowing a manager to participate. All this would do is add to the complexity of formulating the budget which would be unnecessary e.g. with a university, central government revenue could be fixed and therefore could not be influenced by a manager. The manager may not understand the market or industry environment very well and therefore the targets they set could be unrealistic than if imposed centrally. Managers could be demotivated due to going through the motions but with no feeling of real control or influence over the budget they submit e.g. often even with participation centralised decisions will often amend variables which are included. If job satisfaction or morale is diminished because of this then performance could suffer.

219 | P a g e

C1 – 10 Beyond budgeting (CIMA P1 May 2005) Part (i) Hope and Fraser argued that the traditional approach to budgeting should be abandoned for the following reasons:  

The budget process is too rigid and requires conformance to it with not enough flexibility. With the constantly changing business environment, managers need to be having more up to date information to help them make decisions. The budget process is often too bureaucratic, internally focussed and time consuming.

Fixed budgets don’t work today. A budget is a too static an instrument and locks managers into the past, something they thought last year that it was right. Managers instead need to be able to adapt constantly their priorities and put their resources where they can create most value for customers and shareholders. The beyond budgeting approach may include the following: 

Use of rolling budgets focussing on cash forecasts and not cost control.



Budgets revised more frequently and a shorter time horizon when forecasting.



Performance measures using frameworks like the balanced scorecard, developed to achieve improvement, with benchmarking for continuous improvement, rather than an internal focus on actual versus budget and cost control.



Benchmarking for continuous improvement.

Part (ii) The beyond budgeting approach should lead to  

A more dynamic performance management process that will enable organisations to be more responsive and thus create more value. A management model that enables greater devolution of responsibility that leads to increased ‘customer’ satisfaction through continuous improvement.

In the case of W Limited the environment is intensely competitive and likely that constant change is taking place in the market for computer games e.g. frequent innovation. W Limited by using long-term detailed budgets, may not be responding to these changes occurring due to being too internally focussed and reflecting too much on the past rather than what is occurring in the market at present. Planning for a three year time horizon may add little value due to the environment changing so fast.

220 | P a g e

By the creation of more external performance measures, perhaps using the balanced scorecard it could be monitoring more effective information, such as pricing, innovation and customer satisfaction. By the use of rolling cash budgets it could be more in touch with liquidity rather than too much focus on internal cost control. I would recommend changing the current budgeting system by developing more external measures that will help W Limited continually adapt to changes within the dynamic environment they face. I would also recommend a shorter time horizon when planning and more frequent revision to budgets. Greater participation by those who are more in touch with the market environment will also help plan more effectively. C1 – 11 J Limited (CIMA P1 Nov 2005) Part (i) Tip: The current method of budgeting used by J Ltd is incremental budgeting, this approach updates the budget for each period, by taking the previous period as a base and adding a certain percentage on top of this to allow for growth and inflation. Zero based budgeting (ZBB) is a method of budgeting which requires each cost element within a budget, to be specifically justified as though it was being under taken for the very first time. Without approval, the budget allowance would be zero. Therefore each cost, every period, must be ‘justified’ before it can be included in the budget, with employees encouraged to find alternative ways of accomplishing the same but for less money, questions are asked such as; 1. 2. 3. 4.

Do we really need this activity or expenditure? What would happen if we removed it? Are there alternatives that could eliminate it or replace it? How much will alternative ways that have been identified cost and what are the benefits?

Part (ii) Tip: You would need three of the below problems (or others) to earn full marks. Problems that may arise when implementing ZBB Management may think too short-term in view when removing items from the budget. Entrepreneurial spirit will be needed from staff in order to add value to the process of ZBB, this may require a change of staff culture given the existing incremental approach being used. Change management will need to be carefully planned for to introduce ZBB techniques. 221 | P a g e

Training would be required in order for staff to participate effectively. Current information systems may not be compatible to help with a ZBB approach. ZBB will be a time consuming and costly system to organise and administrate once introduced. Many of the decision packages that will be evaluated, rely heavily on qualitative rather than quantitative decisions, requiring subjectivity when making decisions to remove or add items for inclusion within the budget. Many decisions therefore will be hard to make and fundamental mistakes could arise. C1 – 12 ST plc (CIMA P1 Nov 2005)

Tip : A maximum of three marks are available for each requirement within part (e), therefore you need to produce a brief response to include perhaps two issues for part (e) (i) and three issues for part (e) (ii) or vice versa. The solutions below include many possible answers as to what could have been discussed.

Part (i) ERP integrates all manufacturing and related departments and functions for the entire organisation into one single database, this enables various departments to share information and communicate with each other in a real time environment. It is an extension of MRP II. 

   

The budget setting process will now be more effective because it will include an understanding of other resource or activities rather than just production related activities and how these relate to one another e.g. marketing, finance and HR budgets. The budget setting process will incorporate a more effective cash-flow forecast for the entire organisation. Because more enterprise wide activities are integrated, it would improve the efficiency of the budget setting process e.g. less management time involved in trying to link different activities together. It should lower the cost of the budget setting process because all resources and activities are integrated into one whole information system. ERP should allow when planning better ‘what if?’ or sensitivity analysis for the whole organisation e.g. increasing or decreasing the volume of sales or production activities, will give a more effective understanding of the impact this will have on a greater number functions within the organisation.

222 | P a g e

Part (ii)   

ERP operates within a real time environment, therefore you would have continuous exception reporting (variances) in a real time environment. This would allow better control rather than traditional periodic review. Less management time and expense of collecting information for comparison to the budget or standard due to better integration of systems. Less analysis work for management due to a greater number of exception reports produced from one single system.

C1 – 13 W Limited (CIMA P1 Nov 2005)

Tip: You would need to discuss both systems of setting targets from the perspective of the senior management and finance department for 2 marks. Also around three issues for how to alleviate this situation described. Five possible ways have been provided for how to help alleviate the situation.

Poor performance by staff when meeting budgets can often be attributable to the method of implementation. Ignoring the ‘human side’ of participation or the introduction of a target that is either unrealistic or unobtainable can de-motivate staff, this causing more harm than good. Getting the balance right in terms of an ‘achievable standard’ or target is essential during the planning stage of the budgeting process. It is however a problem in reality to actually define an ‘attainable standard’ especially if the environment W Ltd operates within is dynamic or uncertain. It seems likely that senior management may be trying to impose a tighter budget to improve performance by using more challenging targets, perhaps more an ideal standard e.g. a target attained under very favourable conditions. The finance department requires a standard that is more realistic or accurate e.g. an attainable or expected target, that can be achieved with a reasonable level of effort from staff. Practical action the coordinator could take to alleviate the situation described. 1. Use both challenging and realistic targets, but link higher financial reward to the achievement of the more challenging targets and moderate incentive for the easier targets. 2. Allow staff to participate when setting targets or budgets, this can help improve motivation, reduce frustration and increase their job satisfaction. 3. Clear trust and communication developed between management and staff, as well as the avoidance of being over critical by management if more challenging targets are not achieved.

223 | P a g e

4. Good planning to ensure whatever standards or targets that are used, they are realistic and achievable. Periodic review of standards or targets are essential, as well as consultation with staff during this process. 5. The use of rolling rather than periodic budgeting if a high degree of uncertainty exists when setting targets. Rolling budgets are good for adaptive planning e.g. more likely to be regularly updated to take account of changes within the environment W Ltd is operating within. C1 – 14 T plc (CIMA P1 May 2006) Part (i) Tip: Below are many possible answers for the two areas to benchmark, the question states that you are to provide only one performance measure for each area and give a reason. The detection of false claims    

Training expenditure (or training days) per employee to aid detection of false claims. Percentage false claims detected each year. Percentage of staff suggestions for improving the method of detecting false claims actually implemented. Percentage of claims verified by on site inspection, police or fire reports.

Each of the above measures would indicate whether more or less false claims are being detected each period or whether T Plc is doing enough to detect false claims. The speed of processing claims       

Percentage number of claims by customers processed within 14 days. Average of satisfaction ratings from customers from 1-5 (5 being the highest) e.g. speed of service to finalise claims. Number of complaints from customers regarding inaccurate information, late or lost paperwork when handling claims. Average time to settle insurance claims. Number of claims handled per member of staff. Number of claims handled per week. Staff turnover, percentage staff retention or staff absenteeism.

The reason for all of the above measures is that they would indicate the insurance company’s efficiency in processing claims. High staff turnover indicates the replacement frequently of staff which can impact upon the efficiency of internal activities.

224 | P a g e

Part (ii) All performance measures and related information about detecting fraud, should be gathered, summarised and analysed. This information then compared with relevant industry information available, or other insurance companies which may participate as partners in a benchmarking exercise. Such information as the rate of detection, staff training and internal procedures should be compared externally. Internal benchmarking could also be used to compare information between other claims departments, as well as trend analysis of these performance measures compared over time. For the efficiency of processing claims, once a claim is received it should be dated and date stamped when finalised. All information should be recorded in a computerised environment to allow automated and real time information to monitor performance measures and analyse trends over time. Similar information for comparison should be obtained from other insurance companies, trade journals, media an customers complaining to ensure T Plc is doing enough in terms of the norm or industry standard for settling claims. For both methods standards should be implemented to be used as a yardstick to view improvement over time. Internal groups should also be established in order to implement improvements recommended each time benchmarking is undertaken. C1 – 15 Product M (CIMA P1 May 2007) Part (i) Labour rate planning variance (Actual production should take) x (old rate - revised rate) (680units x (900hrs / 600units)) x (£30.00 - £31.20) 1,020hrs x £1.20 Operational labour efficiency variance Actual production did take Actual production should take 50hrs x £31.20 (revised rate)

£1,224 (A) Hrs 1,070 1,020 50 £1,560 (A)

Part (ii) 1. Helps highlight variances between those which are controllable and those which are uncontrollable.

225 | P a g e

2. Helps motivate managers and staff e.g. avoids staff being blamed for faulty planning and gives a fairer reflection of any operational variances calculated when assessing any operational efficiencies or inefficiencies. Management and staff would be appraised more fairly for any favourable or adverse deviations that are within their control. 3. Use of realistic standards in order to measure performance give better management information for control purposes. C1 – 16 QBD (CIMA P1 Nov 2007)

Opening inventory Production (balance) Less: Sales (given) Closing inventory*

Qtr 1 £ 5,500 9,900 15,400 10,000 5,400

Qtr 2 £ 5,400 12,200 17,600 12,000 5,600

*Closing inventory for: Qtr 1 = 45% x £12,000 = £5,400 Qtr 2 = 40% £14,400 = £5,600

Opening inventory Material purchases (balance) Less material usage (W1) Closing inventory* Value of Purchases (£) Purchases above x £6 per Kg W1 Material usage Production 9,900 x 1.5 Kg =

Qtr 1 Kg 4,500 14,925 19,425 14,850 4,575

£89,550

14,850 Kg

*Closing inventory for Qtr 1 would be 25% of Qtr 2 production needs. Therefore: 12,200 units x 1.5 Kg = 18,300 Kg. 18,300 Kg x 25% = 4,575 Kg.

226 | P a g e

C1 – 17 Budgetary planning and control (CIMA P1 Nov 2007) Note: Only THREE reasons are required – SIX possible reasons are given below. 

Budgets are a constraint on doing anything different as they stifle innovation and creativity.



Budgets are costly systems and consume large amounts of management time to set up and analyse.



Budgets have limited use in a dynamic and fast changing environment as they would only serve to reflect on the past and not the future, and so therefore they would be always out of date.



Budgets are generally too internal in focus, ignoring external variables, which more importantly should be being monitored for example the focus on sales targets rather than customer satisfaction.



Budgets create barriers within departments due to the feeling of competition for resources, and do not encourage fluidity between departments which is necessary in a rapidly changing environment.



Budgets are too short-term in focus for example they may just look at a year.

C1 – 18 JIT systems (CIMA P1 Nov 2007) The JIT philosophy requires that products should only be produced if there is an internal or external customer waiting for them. Traditionally manufacturers stockpiled and were not concerned whether or not there was any demand for the products. The main aim was to have enough stock for each stage of production. This would be able to sustain any sudden surges in demand but as a result of this there would be an excess of raw materials, work in progress and finished goods being carried. JIT aims ideally for zero stock e.g. raw materials delivered immediately at the time they are needed, no build up of work-inprogress, finished goods only produced if there is a customer waiting for them. It aims to try to achieve this by doing the following:       

Closer relationships with suppliers required Smaller more frequent deliveries need to be managed in order to produce Cell manufacturing Reduction in set up time (eliminating non value added activities) Higher quality machines perhaps multi-purpose with regular maintenance Involvement and training of staff to maintain flexibility Elimination of defects 227 | P a g e

In conclusion within a JIT environment lead time is paramount due to the absence of stock e.g. a chase demand strategy of meeting customer demand. Total quality management (TQM) will help improve response times by a more efficient and flexible organisation. C1 – 19 Feedback and forward (CIMA P1 Nov 2007) An example of a feedback control system is a budgetary control system. This would gather information on past performance from the output of the system e.g. actual financial performance, and compare this to a predetermined standard or plan (budget) using any deviations e.g. variances, as a basis of improving future performance through control action taken. Feed-forward control is more prevention than appraisal, controlling a system by making adjustments now to the system in advance before any exceptions occur. It does this by trying to predict what will happen in the future. Cash budgets would be an example where the budget would forecast ahead and allow for planned expenditures, highlight any cash shortages and cash surpluses. Action can be taken now to prevent problems occurring in the future. Feedback contrasted to feed-forward control is like closing the door after the horse has already bolted, in other words there is little you can do about it now, except try and rectify the situation to avoid it happening again. C1 – 20 Nursing homes (CIMA P1 Nov 2007) Administration costs for period 3 These are all fixed and we need only apply the relevant index to the cost. Therefore: £100,000 x 104/100 = £104,000 House-keeping costs for period 3 These are all variable and so it is dependent on patient days as well house-keeping costs. Therefore: £125,000 x 90/100 x 106/100 = £119,250

228 | P a g e

Nursing costs for period 3 This cost is semi-variable and so it is important that we find out the variable element from the fixed element by using the high low method. However we must first compare like with like and so we will need to deflate period 2 nursing costs to obtain the period 1 nursing costs. Therefore: Period 1 prices = £324,000 / 1.08 = £300,000 Variable costs per index point in period 1 = (£300,000 - £280,000) / (125 – 100) = £800 We now use either nursing costs at the higher level of £300,000 or at the lower level of £280,000 to work out the variable and fixed costs within. Period 1 Semi-variable cost £300,000

Variable costs Fixed costs

Variable cost Fixed cost £800 x 125 = £100,000 £200,000

Period 1

Period 2

x 125 x 108 £80,000 100 100 x 108 £200,000 100 £280,000

x 90 x 105 £108,000 100 100 x 105 £216,000 100 £324,000

Period 3 £102,060 £226,800 £328,860

Summary of period 3 budget

House-keeping Nursing Administration

£ 119,250 328,860 104,000 552,110

229 | P a g e

C1 – 21 Participative budgeting (CIMA P1 May 2008)

Tip: Only three behavioural consequences are required. Nine possible answers are included below. 

If participation is allowed then it would motivate the project team as their ideas of budgeting would be valued and it would mean that the budget would be more likely to be met by the team.



Participative budgeting would also be more likely to be accepted by the team as they themselves have helped to devised it.



Team members may be aware of more relevant and up to date reactions in the market which may be crucial in making informed decisions.



If managers are asked to participate in setting their own budgets there is the possibility they will include “slack” or “padding” within the budget e.g. have less sales units forecast than they could realistically achieve, given a reasonable level of effort. This is the inevitable downside of human behaviour e.g. underestimate to avoid blame if they were to under achieve.



Participation could create a slower process to formulate a budget because more consultation is required. This will inevitably increase the cost and time of formulating budgets, leading to budgets which are late or costly to produce.



The manager could be inexperienced financially. Therefore they may not be able to contribute effectively if they were to participate. This could lead to inaccuracies when standards or targets are developed.



In a stable environment where revenue or costs are certain there would be little benefit in allowing a manager to participate. All this would do is add to the complexity of formulating the budget which would be unnecessary e.g. with a university, central government revenue could be fixed and therefore could not be influenced by a manager.



The manager may not understand the market or industry environment very well and therefore the targets they set could be unrealistic than if imposed centrally.



Managers could be demotivated due to going through the motions but with no feeling of real control or influence over the budget they submit e.g. often even with participation centralised decisions will often amend variables which are included. If job satisfaction or morale is diminished because of this then performance could suffer.

230 | P a g e

C1 - 22 Rolling budgets (CIMA P1 May 2008) Rolling or continuous budgeting is when a further period is added immediately to the budget when the earliest period has expired for example if Jan to Mar 2005 is the first three months of the yearly forecast, once this has expired then Jan to Mar 2006 will be created and added. Rolling budgets are good for adaptive planning e.g. more likely to be regularly updated to take account of changes within the environment the organisation is operating within. Advantages  There is always an up to date twelve month budget available to management as the budget is adjusted on a frequent basis.  Rolling budgets are very useful when there is uncertainty, for example cash budgets are normally rolling as you can spot early warning signals quicker.  There is regular assessment and amendment, to keep the forecast more accurate.  Decisions made using the information from the rolling budget should be far more accurate and effective as the budget continually up dated. Disadvantages To create and continue to update the budget is time consuming to management and staff A rolling budgetary system is a costly system to organise and administrate. It may not be necessary for some company’s who operate in a stable industrial environment, as the costs may outweigh the benefits of up to date information.

231 | P a g e

Solutions – Section B Part C Budgeting and Management Control C2 -1 M plc (CIMA P1 May 2006) Part (a) Tip: The budget is based upon 6,400 labour hours and is being compared to actual expenditure based on 7,140 labour hours. This is not comparing expenditure on a like for like basis. Budgeted expenditure needs to be flexed to represent a budget based upon 7140 labour hours and you are given information in terms of how costs behave in order to this.

Budget

Flexed Budget

Actual

Assembly labour hours

6,400

7,140

7,140

Assembly labour Furniture packs Other materials Variable overhead Total variable cost

$ 49,920 224,000 23,040 34,560 331,520

$ 55,692 249,900 25,704 38,556 369,852

W1 W2 W3 W4

$ 56,177 W1 205,000 24,100 76,340 W5 361,617

$ 9,000 W4 27,000 W4 36,000 2,050 W1 38,050

$ 9,000 27,000 36,000 2,050 W1 38,050

Fixed overhead HQ fixed overhead Stepped fixed cost Managers fixed salary Total fixed cost Total cost

$ 9,000 18,500 27,500 2,050 29,550 361,070

407,902

399,667

Variance

$ 485 44,900 1,604 37,784 8,235

(A) (F) (F) (A) (F)

$ nil nil nil nil 8,235 (F)

W1 Assembly labour The original budget has removed the $2,050 managers fixed salary ($51,970 - $2,050) = $49,920, this cost has then been prorated to reflect this cost at 7,140 rather than 6,400 hours ($49,920 x (7,140 ÷ 6,400) = $55,692. The actual cost of assembly labour $58,227 has also removed the $2,050 managers fixed salary ($58,227 - $2,050) = $56,177. We would now have a better comparison due to the assembly labour analysed between fixed and variable cost. 232 | P a g e

W2 Furniture packs Given no other information, furniture packs within the flexed budget have been prorated on the basis of 7140 rather than 6400 hours ($224,000 x (7140 ÷ 6400) = $249,900. W3 Other materials Given no other information, materials within the flexed budget have been prorated on the basis of 7,140 rather than 6400 hours ($23,040 x (7,140 ÷ 6,400) = $25,704. W4 Overhead analysis between fixed and variable cost Hours $ 10,000 90,000 7,500 76,500 2,500 13,500 Variable cost per hour

$5.40

The high low method has been used to derive the variable and fixed cost element for overhead cost. When assembly hours exceed 7,000 hours fixed cost increases. The above workings are based on 7,500 and 10,000 assembly hours therefore the fixed cost included in the total cost would be the same. 

Variable cost equals the change in total costs ($13,500 ÷ 2,500 hours) = $5.40.

Therefore budgeted and flexed budgeted variable overhead has been calculated on the basis of $5.40 an assembly hour. 

Fixed cost is the balance of the total cost using either 7,500 or 10,000 assembly hours above to work this out; given fixed overhead will be the same for both activities.

$90,000 = Fixed cost + ($5.40 x 10,000 hours). Therefore fixed cost equals $36,000 at any activity over 7,000 assembly hours. Given the variable cost of $5.40 per assembly hour would remain constant (no other information to assume otherwise). The fixed cost at 5,000 hours would be $54,500 = Fixed cost + ($5.40 x 5,000 hours) $27,500 rather than as above $36,000. The marginal or stepped fixed cost above 7,000 assembly hours would therefore be $36,000 - $27,500 = $8,500. The fixed overhead within the statement above has also been broken down into the fixed $9,000 central headquarter charge and the remaining stepped fixed cost for better analysis.

233 | P a g e

Actual fixed costs for April were as budgeted therefore no variances for this period. W5 Actual variable overhead The question states that actual fixed cost is the same as budgeted fixed cost. Therefore if actual overhead was as stated in the question $112,340 and actual fixed overhead included within this was $36,000 (based on assembly labour hours being greater than 7000) then actual variable overhead for the period would have been $112,340 - $36,000 = $76,340. Part (b) (i) The revised statement is useful for the following reasons. 1. The original format supplied by M Plc did not provide a like for like comparison when calculating variances. This is because the original budget provided in the scenario had not flexed the budgeted costs; to represent what costs should have been based upon the actual assembly hours of 7140. The revised format will allow a more accurate comparison between budgeted and actual costs for control purposes. 2. The original format supplied by M Plc did not sub-divide costs into the classification of either variable or fixed. By the classification of variable and fixed cost within the revised format, it will allow management to understand cost behaviour between costs which may rise and fall with an activity level and those which remain constant. This will aid more effective planning, control and decision making by central headquarters. 3. The original format supplied by M Plc did not sub-divide fixed overhead into what was controllable by the department e.g. stepped fixed cost, and what was uncontrollable e.g. the headquarter central overhead charged. The revised format will allow a fairer assessment of the department, by the avoidance of criticising the department for costs which are not within there own control. Part (b) (ii) Whilst assembly labour cost may very well have a strong correlation with labour hours especially if piecework payment schemes are used, this is unlikely for the relationship between furniture packs and other materials. Furniture packs and other materials is more likely to be correlated with the type of furniture being assembled not direct labour hours. For example a dining room table and chairs maybe more complex to assemble than a wardrobe, however may require less finished timber. This maybe evidenced by the revised flexed budget statement prepared.

234 | P a g e

It can be seen that both the furniture packs and other material variances were favourable $44,900 and $1,604 respectively, where as the assembly labour variance was $485 adverse (which I assume if no change in the actual rate paid to staff when compared to standard rate, this is explained by efficiency). It seems that assembly was more complex and took longer by staff, but the furniture packs and materials used were far less. If there were a relationship between material usage and labour efficiency, we would have seen a greater dependency between these variances e.g. if one is favourable, so too would be the other. It would be a good idea for management to investigate a more effective cost driver for furniture packs and other materials, than labour hours when preparing budgets. Part (c) Advantages of allowing participation  Greater motivation for the assembly department manager and his assembly staff. Because of this targets are more likely to be met. It will also be more interesting and more involving for the manager increasing his job satisfaction.  Participation means that targets are more likely to be accepted by the manager and assembly staff e.g. less conflict due to setting targets themselves.  The manager or assembly staff maybe more aware of the environment they work within e.g. how long different types of furniture take to assemble and how much timber and material they would consume. The new manager does seem to have many years experience working within other assembly departments. Therefore the targets set could be more realistic than if imposed centrally. Disadvantages of allowing participation If managers or assembly staff are asked to participate in setting their own targets there is the possibility they will include “slack” or “padding” within the budget e.g. budget longer for how long it would take to assemble different types of furniture. This is the inevitable downside of human behaviour e.g. over estimate to avoid blame if they were to under achieve. Participation could create a slower process to formulate a budget because more consultation will be required. This will inevitably increase time and cost of formulating a budget. The manager seems to have little experience previously of working with budgets. Training would therefore be required in order for him to participate effectively. I feel that the manager should be allowed to participate in budget setting, however clear trust, communication and consultation between him and headquarters needs to be developed. Feedback at frequent intervals, as well as the avoidance of being over critical by headquarters will all help improve motivation and performance within the assembly department.

235 | P a g e

C2 – 2 RF Ltd (CIMA P1 May 2007) Part (a) Mth1 £

Mth 2 £

Mth 3 £

Total £

RECEIPTS Sales (W1) Capital Total receipts

2,940 16,250 19,190

10,180

15,545

10,180

15,545

28,665 16,250 44,915

PAYMENTS Material (W2) Labour (W3) Variable O/H (W4) Fixed O/H (W5) Total payments

0 6,105 1,332 3,750 11,187

3,515 5,940 2,184 5,625 17,264

3,420 6,666 2,318 5,625 18,029

6,935 18,711 5,834 15,000 46,480

8,003 0 8,003

(7,084) 8,003 919

(2,484) 919 (1,565)

(1,565) 0 (1,565)

Net cashflow Bal b/f Bal c/f Workings (W1) Sales

Sales (units x selling price) Receipts in 1st month (20%) and 2% discount given Receipts in 2nd month (45%) Receipts in 3rd month (25%) Total

Mth 1 £ 15,000 3,000 – 60 = 2,940

2,940

Mth 2 £ 17,500 3,500 – 70 = 3,430 6,750

Mth 3 £ 20,000 4,000 – 80 = 3,920 7,875

10,180

3,750 15,545

236 | P a g e

(W2) Material Mth 1 Units 1,500 0 1,500 350 1,850

Mth 2 Units 1,750 (350) 1,400 400 1,800

Mth 3 Units 2,000 (400) 1,600 420 2,020

£1.90 x 1,850 = £3,515

£1.90 x 1,800 = £3,420

£1.90 x 2,020 = £3,838

£3,515

£3,420

Forecast sales Opening inventory Closing inventory Production Material cost (£1.90 x units) Material payments

Mth 4 Units 2,100

(W3) Labour

Production units Labour cost (£3.30 x units)

Mth 1 1,850

Mth 2 1,800

Mth 3 2,020

£3.30 x 1,850 = £6,105

£3.30 x 1,800 = £5,940

£3.30 x 2,020 = £6,666

(W4) Variable O/H

Production units Variable O/H cost (£1.20 x units) Payments 60% in 1st month 40% in 2nd month Variable O/H payments

Mth 1 1,850

Mth 2 1,800

Mth 3 2,020

£1.20 x 1,850 = £2,220 £ 1,332

£1.20 x 1,800 = £2,160 £ 1,296 888 2,184

£1.20 x 2,020 = £2,424 £ 1,454 864 2,318

1,332

(W5) Fixed O/H

Fixed costs (£75,000 / 12) Payments 60% in 1st month 40% in 2nd month Fixed O/H payments

Mth 1 £ 6,250

Mth 2 £ 6,250

Mth 3 £ 6,250

3,750

3,750 1,875 5,625

3,750 1,875 5,625

3,750

237 | P a g e

Part (b) (i) If the cost was £1.50 there would be saving of £0.40 per unit.

Production cost savings

Mth 1 1,850 x £0.40 = £740

Mth 2 1,800 x £0.40 = £720

Mth 3 2,020 x £0.40 = £808

£740

£720

Savings received Total saving

£1,460

New total budget cash flow = Savings received + Current total budget cash flow £1,460 + -£1,565 = (£105) Part (b) (ii) If the cost was £2.20 there would be an extra cost of £0.30 per unit.

Production extra cost

Mth 1 1,850 x £0.30 = £555

Mth 2 1,800 x £0.30 = £540

Mth 3 2,020 x £0.30 = £606

£555

£540

Extra costs to pay Total extra cost

£1,095

New total budget cash flow = Extra costs to pay + Current total budget cash flow -£1,095 + -£1,565 = (£2,660) Part (c) Tip: A good report format is essential as it will present your answer well and provide good structure. In addition there maybe up to 5 marks given for a good report format style. Part (c) in short requires the benefits of “what if” when preparing cash budgets, so if you know what these are it will be the backbone to answering this part to the question. A good approach also would be to think of good examples from the calculations in the previous parts which will give value within your report.

REPORT To: Management of RF Ltd From: Management Accountant Subject: The benefits of “what if” analysis Date: 19th May 2007

238 | P a g e

1. Introduction The purpose of this report is to discuss the benefits or otherwise of performing “what if” analysis when preparing cash budgets. 2. “What if” analysis “What if” analysis looks at varying or changing the key variables to see how the outcome would change. These changes would be due to revisements of estimations or probabilities and typically might be material costs or demand. Spreadsheets are the main tool to help create a “what if” analysis which allows you to manipulate key figures and giving you instant feedback as to the impact on the outcomes. Flexible budgets are an example of “what if” analysis where you have figures for different levels of demand or production, allowing you too appreciate how profit might change as a result of these different levels. Using our calculations from part (b) we can see that by increasing direct material cost by 16% to £2.20 per component there is a 70% fall in total budget cash flow to negative £2,660. However a 21% decrease in direct material cost to £1.50 per component will result in a 93% increase in total budget cash flow to negative £105. It can be seen here that direct material costs is very sensitive and a small change will result in large change to total budget cash flow. 3. Benefits of “what if” analysis “What if” analysis allows for better planning and can be ‘flexed’ at the end of a period to correspond and be compared to actual results achieved, giving better information for control purposes It gives management a better understanding of the sensitivity of costs and revenues to small changes and therefore a better understanding of risk. It allows management to scrutinise different scenarios such as worst case, expected case and best case. “What if” analysis lends itself to the use of spreadsheets by making calculations more accurate, faster, easier to update changes and cuts down on the time of management to calculate and analyse the information The use of spreadsheets means that budgets are paperless and can be sent to and used by multiple users. 4. Limitations of “what if” analysis “What if” analysis does not account for qualitative information, only looking at changes to quantitative information.

239 | P a g e

The changes that are made in “what if” analysis are in isolation and do not give an understanding of what would happen to budgeted cash flow if multiple changes happened at the same time. What if analysis does not give management any idea of the probability of these changes happening and therefore no true understanding of exposure to risk. I hope you have found this report useful but should you require any further assistance or have any questions please do not hesitate to contact me. Signed Management Accountant C2 – 3 Trackit (CIMA P1 May 2008) Part (a) Cash budget for Q Months Receipts Sales (W1) Total receipts

1 $ 20,160 20,160

2 $ 65,240 65,240

3 $ 148,820 148,820

Total $ 234,220 234,220

Payments Direct materials (W2) Direct wages ($10 per unit) Direct variable overheads (W3) Fixed overheads (W4) Total payments

0 14500 18,850 42,000 75,350

106,800 16500 31,600 52,500 207,400

104,640 21200 39,110 52,500 217,450

211,440 52,200 89,560 147,000 500,200

Opening cash flow Net cash flow in the month Closing cash flow

250,000 194,810 (55,190) (142,160) 194,810 52,650

52,650 250,000 (68,630) (265,980) (15,980) (15,980)

240 | P a g e

Workings (W1) Sales receipts Selling price of tickets are $140 each Months

1 $ 140,000

2 $ 210,000

3 $ 280,000

20,160

30,240

40,320

35,000

52,500

20,160

65,240

56,000 148,820

1 $ 87,000

2 $ 99,000

3 4 $ $ 127,200 147,600

Opening inventory Purchases (balancing figure)

0 106,800

19,800 104,640

25,440 131,280

Closing inventory (20% of next month production)

(19,800)

(25,440)

(29,520)

87,000

99,000

127,200

0

106,800

104,640

Variable overheads

1 $ 29,000

2 $ 33,000

3 $ 42,400

Variable overheads (65% in month) Variable overheads (35% in next month) Variable overheads paid

18,850 0 18,850

21,450 10,150 31,600

27,560 11,550 39,110

Sales Immediate payment (15% of sales and 4% discount) 1 month later (25% of sales) 2 months later (40% of sales)

(W2) Direct materials Direct materials are $60 per unit Months Material costs used in the month

Material costs used in the month Materials paid

(W3) Direct variable overheads Months

241 | P a g e

(W4) Fixed overheads Fixed overheads are $840,000 per annum Months Fixed overheads (spread equally)

1 $ 70,000

2 $ 70,000

3 $ 70,000

Fixed overheads (60% in month) Fixed overheads (15% in next month) Fixed overheads paid

42,000 0 42,000

42,000 10,500 52,500

42,000 10,500 52,500

Part (b) (i) Total cost of direct material for the three month period is $211,440 as per the cash budget in part (a). The cost of the specialist component is included within material cost at $40 per unit. Total unit material cost is $60 which includes the cost of the component. Therefore total cost of the component is $211,440 x 40/60 = $140,960. If the unit cost of the component is now $32 then the total cost of the components will now be $140,960 x $32/$40 = $112,768. This will result in component cost savings of $140,960 - $112,768 = $28,192 decreasing total net negative cash flow from $265,980 to $237,788. Part (b) (ii) If the unit cost of the component is now $50 then the total cost of the components will now be $140,960 x $50/$40 = $176,200. This will result in component cost increase of $176,200 - $140,960 = $35,240 increasing total net negative cash flow from $265,980 to $301,220. Part (c)

REPORT To: From: Date: Subject:

1.0

Owners of Q Management Accountant 20th May 2008 Profitability of Q

Introduction

The purpose of this report is to offer advice about the profitability of your business and the prospects revealed by your business plan. 242 | P a g e

Profitability Annual forecast sales are 27,700 units resulting in total sales of $3,878,000 and profit of $545,000 if the component cost is $40 per unit. This gives a net profit margin of 14%. The break-even level of sales (the amount of units sold where all fixed costs have been recovered and no profit or loss is made) would be 16,800 units and the margin of safety is 39% which is good as this means that sales could fall by nearly 40% before the company started to make losses. If the specialist component cost fell down to $32 per unit then total profit would be $766,600 but if the component cost went up to $50 per unit then total profit would be $268,000. The business is profitable given the best and worst case scenarios for the cost of the specialist component. 1.2

Cash budget

The opening cash position in month 1 is $250,000 however given the expenditure over the next three months it seems that this alone will not be enough to meet cash consumption. An overdraft would be needed of approximately $16,000 to cover cash requirements to Month 3. If the unit cost of the component is now $32 then the total cost of the components will now be $112,768. This will result in component cost savings of $28,192 decreasing total net negative cash flow from $265,980 to $237,788. If the unit cost of the component is now $50 then the total cost of the components will now be $176,200. This will result in component cost increase of $35,240 increasing total net negative cash flow from $265,980 to $301,220. 1.3

Business plan and component costs

The business plan presents an optimistic projection of sales and costs and therefore it is essential to review the assumptions for behind the plan to ensure that they are sensible and realistic. There is uncertainly about the cost of the specialist component and this has major impact on profitability and cash flow. It is essential that this cost is stabilised to get a more reliable cash budget and profit analysis. It would appear that cash flow will be a major problem in the opening months and is vital that additional financing such as an overdraft is arranged in the short term, because in the long term the business will return profits.

243 | P a g e

Having a list of preferred suppliers or a single supplier will help stabilise the cost of the component. Giving the supplier an exclusive contract of components needs should help in agreeing a unit cost. In addition good relations with the supplier will ensure that quality of service and material as well cost is achieved. I hope you have found this report useful and should you have any further questions please do not hesitate to contact me.

Management Accountant Workings Forecast sales for the year = 1,000 + 1,500 + 2,000 + 2,400 + 2,600 + (7 x 2,600) = 27,700 units Component cost Contribution per Trackit Total contribution Fixed costs Total profit Break-even units Margin of safety

$32 $140 - $90 + $8 = $58 $1,606,600 $840,000 $766,600 $840,000 / $58 = 14,483 48%

$40 $140 - $90 = $50 $1,385,000 $840,000 $545,000 $840,000 / $50 = 16,800 39%

$50 $140 - $90 - $10 = $40 $1,108,000 $840,000 $268,000 $840,000 / $40 = 21,000 24%

C2 – 4 X plc (CIMA P1 Nov 2006) Part (a)(i) Note: It is not a requirement to produce the production budget for Qtr 5 but this information will be useful when working out the closing inventory for Material B. Qtr 1 Opening inventory* Production (balance) Less: Sales (given) Closing inventory

675 2190 2865 2250 615

Qtr 2 615 1930 2545 2050 495

Qtr 3 495 1770 2265 1650 615

Qtr 4 615 1810 2425 2050 375

Qtr 5 375 1490 1865 1250 615

244 | P a g e

*Opening inventory at the start of Qtr 1 would be the closing inventory from the previous quarter brought forward; therefore this would be 30% of Qtr 1 sales (0.3 x 2250 = 675). Part (a)(ii) Qtr 1 Kg Opening inventory Material purchases (balance) Less material usage (W1) Closing inventory* Value of Purchases (£) Purchases above x £7 per Kg W1 Material usage Production 2190 Production 1930 Production 1770 Production 1810

2956.5 6219 9175.5 6570 2605.5

Qtr 2

Qtr 3

Qtr 4

2605.5 5574 8179.5 5790 2389.5

2389.5 5364 7753.5 5310 2443.5

2443.5 4998 7441.5 5430 2011.5

£43,533 £39,018 £37,548 £34,986

x 3 Kg = x 3 Kg = x 3 Kg = x 3 Kg =

6570 5790 5310 5430

*Closing inventory for Qtr 4 would be 45% of Qtr 5 usage (production Qtr 5 1490 x usage 3 Kg = 4470 Kg). 4470 Kg x 0.45 = 2011.5 Kg. Part (b) Material A has been identified as a limiting factor (or principle budget factor) e.g. a scarce resource which is in short supply. X Plc should try firstly to identify any further suppliers, reduce wastage levels and run down any inventory for material A it has in the meantime. Assuming more material cannot be sourced X Plc should conduct a limiting factor analysis on its different products sold in order to maximise contribution (and therefore profit). X Plc will maximise contribution by allocating material A to products that earn the highest contribution per unit of material A consumed. Normally sales demand is the most binding constraint and therefore the sales budget is normally produced first and the production budget second (production driven by sales). However material A in this case would be the most binding constraint, it should be the first operating budget prepared, then the production budget (because material A restricts production). Once budgeted production levels have been determined, all other budgets such as sales, other material, labour and overhead can be prepared.

245 | P a g e

Part (c) Flexible budget statement Original Flexed Budget Budget 7700 units 7250 units Skilled Labour (4hrs x £15) x units Semi-skilled labour (6hrs x £9) x units Variable overhead (10 hrs x £2.18) x units Fixed overhead (£14.55 per unit)

462,000 415,800 168,000 112,000 1,157,800

435,000 391,500 158,050 105,488 1,090,038

Actual Results 7250 units

Total Variance

568,750 332,400 185,000 105,000 1,191,150

-133,750 59,100 -26,950 488 -101,112

(A) (F) (A) (F) (A)

Fixed overhead 40% of overhead is fixed therefore 40% x £280,000 = budgeted fixed overhead £112,000. Fixed overhead absorption rate per unit £112,000 ÷ 7700 budgeted units = £14.55 per unit. Within the flexed budget you could assume fixed overhead is £112,000 rather than £105,488. Because this is an absorption costing company I would assume the fixed overhead absorbed would have been (7250 units x £14.55) £105,488 and therefore the over absorption for the period £488. Variable overhead 60% of overhead is variable and varies with total labour hours. 60% £280,000 = £168,000. Total labour hours forecast 7700 units x 10 hours per unit = 77000 hours. £168,000 ÷ 77000 hours = £2.18 per labour hour. Part (d) Incremental budgeting is the process of using current and past budgets as a guide and adding or subtracting from these budgets to arrive at income and expenditure for a future financial period. The main problems with this system is that it can encourage wasteful expenditure and misallocate resources. Zero based budgeting (ZBB) is a method of budgeting which requires each cost element within a budget, to be specifically justified as though it was being under taken for the very first time. Without approval the budget allowance would be zero.  ZBB attempts to remove for every financial period any obsolescent or inefficient spending, (slack or ‘budget padding’), as a process it forces management and employees to avoid wasteful spending. Incremental budgeting normally updates expenditure by taking the previous period as a base and adds a certain percentage on top of these figures to allow for growth and inflation, it therefore can compound inefficient or wasteful spending over time.

246 | P a g e

 ZBB concentrates on the future and challenges past activities making staff more flexible to environmental change. Entrepreneurship as a culture is not normally encouraged with incremental budgeting.  ZBB can increase staff motivation due to a budgeting system that is more interesting and more involving. Part (e) Rolling or continuous budgeting is when a further period is added immediately to the budget when an earlier period has expired e.g. if Jan to Mar 2007 is the first three months of the yearly forecast, once this has expired then Jan to Mar 2008 will be created and added. Rolling budgets are good for adaptive planning e.g. more likely to be regularly updated to take account of changes within the environment the organisation is operating within. Advantages for X Plc  Always an up to date budget available for management. This means the budget is less likely to be out of date due to it being frequently revised or amended during the budgeting period.  Useful for when an organisation if facing high uncertainty when forecasting e.g. cash budgets if rolling can spot early warning signals of cash shortages much quicker. X Plc does face major uncertainties with regard to increased levels of competition and issues concerning the availability and cost of material and staff, these factors will cause problems when trying to forecast accurately. Once the budget has been established at the beginning of a period, it is then constantly amended on account of developing circumstances.  Regular assessment and amendment keeps the forecast and standard cost information more accurate within X Plc. This should prevent a variance arising from external and uncontrollable circumstances, giving more meaning to the variances which remain and the operational variances more attributable to staff and management effort during the period.

247 | P a g e

Solutions – Section A Part D Control and Performance Measurement of Responsibilty Centres D1 -1 EVA and RI (CIMA P2 Pilot paper 2005)

Tip: Economic value added was developed by Stern Stewart & Co. ‘The economic value created by a division in a given period of time’ EVA = Net cash operating profit after tax (‘adjusted’ for accounting distortions e.g. add back depreciation) less Economic depreciation (a reflection of market values) less Amortised R&D, advertising and goodwill less (‘adjusted’ capital employed* x cost of capital) * using the economic replacement cost of assets. Tip: Residual income (RI) is calculated by taking the profit a manager earns for a division less a ‘notional interest charge’ for the investment within the division e.g. the return generated from the division less a finance charge from the holding company or head office using a cost of capital engaged within it. Profit after tax Capital employed x cost of capital Residual income

X (X) X

Similarities with EVA and RI   

Both are absolute rather than relative measures. Both deduct a finance charge for the cost of capital used within a division. Both exclude interest payments in arriving at profit due to using a notional interest charge instead e.g. the cost of capital.

Differences with EVA and RI 

EVA uses the replacement not historical accounting cost of assets when calculating a finance charge by head office.

248 | P a g e





The profit calculated under both methods are different. EVA uses economic not accounting profit, it adjusts for accounting distortions such as adding back accounting depreciation and deducting economic depreciation instead, based upon the replacement cost of assets. EVA would also amortise R&D, advertising or goodwill over a useful economic life when arriving at profit. EVA capitalises costs such as R&D, advertising or goodwill and includes these in the replacement cost of assets to apply the finance charge. This is because EVA considers such expenditure as long-term assets building for the future.

D1 – 2 Controllability principle (CIMA P1 Pilot Paper 2005) The controllability principle is concerned with assessing performance based upon measures that can be controlled only by a manager and omitting any items which are uncontrollable. As an example the head office or holding company could ensure it does not include and therefore evaluate a manager on the head office overhead they apportion or interest charges they apply centrally. Arguments for the controllability principle  It would be considered fairer by a manager if they were not assessed on costs which are not within their own control. This is likely to improve motivation and morale.  If a manager was assessed on costs which were not within their control, it could be argued there would be little they could do about these costs any way, even if exceptions were reported. Arguments against the controllability principle Political arguments may occur over such costs which are more subjective than objective when determining controllability. It is not always black and white when determining controllability from uncontrollability. Just because a cost is uncontrollable does not mean a manager being assessed should ignore it altogether. As an example if managers recognise there is an interest charge by head office based upon the capital employed used within a division, then to hold them more accountable could help improve efficiency by the minimisation of capital employed. In the case of head office charges, lack of accountability could encourage over consumption of these resources provided centrally. D1 – 3 Transfer pricing (CIMA P1 Pilot Paper 2005) The following problems could arise when divisions in different countries buy and sell to one another and a transfer price needs to be determined.

249 | P a g e

Exchange rates Often the buying and selling division within a group will use different currencies and these can fluctuate from one day to another. The difficulty in these circumstances is what transfer price to actually set e.g. which exchange rate do you use. Another problem can arise when settling liabilities between the two divisions e.g. exchange gains and losses occur when payment arises. Taxation Different countries may set taxation rates higher or lower than other countries for profit earned by resident companies. In cases where the buying divisions country is higher taxation compared to the sellers, a higher transfer price can shift profit away from the buyer to the seller, due to a higher cost incurred. This will help reduce the tax liability of the group. Some countries also apply withholding tax on profits repatriated as dividend by an overseas subsidiary to its parent. Protectionism Certain counties may restrict the flow of goods between divisions due to import quotas in place or a tariff could be applied to the price of each good sold. In cases where tariffs are applied, it could be important to minimise the transfer price to the buying division in the overseas country to minimise this tax liability. Other problems could include a restriction on foreign currency available to a buying division e.g. exchange controls, when settling intra-group liabilities. In these cases netting off liabilities between divisions before settlement takes place may help avoid such restrictions. Worldwide prices of other suppliers Due to exchange rate fluctuations a buying division may find it cheaper to buy externally from another supplier rather than the selling division. If the marginal cost of producing each unit for the selling division is lower than the price the buyer pays externally, this would not maximise group profit.

250 | P a g e

D1 – 4 EVA (CIMA P1 May 2005) Part (i) ‘The economic value created by a division in a given period of time’ EVA = Net cash operating profit after tax (‘adjusted’ for accounting distortions e.g. add back depreciation) less Economic depreciation (a reflection of market values) less Amortised R&D, advertising and goodwill less (‘adjusted’ capital employed* x cost of capital) * using the economic replacement cost of assets.

Economic value added was developed by Stern Stewart & Co, a US management consultancy firm. EVA is an absolute measure of the economic financial wealth generated by a division or organisation over time. It deducts a finance charge using a cost of capital, applied to the replacement cost of assets used by the division or organisation. EVA capitalises costs such as R&D, advertising or goodwill and includes these in the replacement cost of assets to calculate the interest charge. This is because EVA considers such expenditure as long-term assets building for the future. EVA uses economic not accounting profit, it adjusts for accounting distortions such as adding back accounting depreciation and deducting economic depreciation instead, based upon the replacement cost of assets. EVA would also amortise R&D, advertising or goodwill over its useful economic life when arriving at economic profit.

251 | P a g e

Part (ii)

Tip: Only three reasons briefly explained are required for how EVA might affect the behaviour of managers. Five possible answers are included below.

EVA may affect the behaviour of divisional senior executives in the following ways. 1. They would concentrate their investment decisions on maximising shareholder value or financial wealth of their shareholders. 2. They would concentrate on the maximisation of cash or contribution which is more likely to maximise shareholder value e.g. EVA can not be manipulated by a manager’s choice over the accounting policies they might use. 3. They would concentrate on long-term decisions as opposed to short-term decisions e.g. with relative measures like return on investment (ROI) often new investments deliver low profit and have high accounting book values in the early years. This often discourages managers in the short-term from undertaking investment due to a low ROI. 4. Because a finance charge is applied against the replacement cost of assets, it forces managers to use and invest in assets more efficiently. 5. EVA will not discourage expenditure on long-term assets building for the future such as marketing or research and development. This is because these items will not be deducted entirely when arriving at economic profit, instead amortised over the period of the expenditures useful economic life. This would lead to perhaps greater EVA when compared to the measure of accounting profit. With accounting profit it is more likely the entire cost would be deducted and therefore could deter a manager if assessed on accounting measures such as residual income or return on investment. D1 – 5 WD, PD & TD (CIMA P1 May 2005) Part (i) WD Two thirds of output sold by WD is to PD therefore the transfer price it sells at will determine significantly the profits and return on investment (ROI) of WD. It is possible that WD may not be controlling its cost due to cost-plus pricing used, which may add a mark-up upon actual not standard cost. This would mean that WD may not be controlling its costs, passing on any inefficiency by charging higher prices to PD.

252 | P a g e

The behavioural consequences in this case is that WD could in fact manipulate the internal system of cost-plus pricing used by increasing its own costs in order to add a mark-up and earn higher profit absolutely. As an example if 25% mark-up was used £100 cost + 25% mark-up would earn £25 profit per unit of timber, but if cost was increased to £150 a 25% mark-up would earn £37.50 profit per unit of timber. This will not improve group profit. WD may therefore have no incentive to control cost and due to two thirds of its output sold to PD, it may concentrate solely on supplying WD rather than external customers due to potentially more profit. This will not improve group profit. PD The policy of C Plc is that PD must buy all its output from WD and sell all its output to TD. It seems that PD is wholly reliant on both of the other two divisions, an intermediary in the supply chain. The same consequences of behaviour as WD can apply here, that is there would seem to be no incentive for PD to control cost, as well as any inefficiency of WD passed on to TD in the form of higher prices. There is also the behavioural consequences of PD and WD minimising much needed investment in non-current assets as a way of improving return on investment. Under investment in assets causes the net book value of assets to decrease over time. Even if profit remains static ROI will improve, yet the managers of both divisions will have done very little to improve financial results. TD Due to TD only purchasing items it sells from PD, it is highly reliant on the cost and quality of products made by PD to earn profit. Due to the last division in the internal supply chain it could end up paying for the inefficiencies of the other two divisions, this effecting it’s ability to be competitive. This would be incredibly frustrating for the manager of TD, especially if products can be procured from external suppliers at far cheaper prices, as well as increasing product range and quality if they did this. Morale and motivation in this division maybe low as a consequence, as sales and profit earned are highly reliant on the cost, quality and range of products sold to them by PD, they have very little control over these factors due to only selling items purchased from PD.

253 | P a g e

Part (ii) If the internal transfer price for items sold by PD to TD were at market price e.g. similar to a price on the open market, many of the frustrations of TD could be avoided. At least TD will buy at a fair market price and would not be frustrated by the fact it could procure products from external suppliers cheaper. Also there would be greater incentive for PD to control cost, the only way to improve profit would be to control cost and improve efficiency, due to its prices set by the open market. The problems of this approach is that it still may do very little to motivate PD to improve quality, or increase or innovate its products made, due to the fact that TD according to company policy will buy all of its products from PD anyway. It would also frustrate the manager of division PD, if inefficiencies are still being passed on by WD due to cost-plus pricing still applying between these two divisions. There is also the difficulty of determining what is a definitive market price for PD products sold. There is good argument that PD should in fact be controlled as a cost rather than profit centre, due to the fact it does not buy or sell outside the group. If this were the case then supply should take place at the marginal cost of production according to economic theory. Perhaps at standard rather than actual marginal cost as an incentive for PD to control its efficiency and cost. There is the added advantage in this case that TD could purchase items made by PD at far less than market price, this could allow them to sell to garden centres or similar outlets at reduced cost improving sales volume and profit, however this also could be dangerous due to the fixed cost of PD not being considered. If marginal costing was used there would also be little incentive for the manager of PD to supply, as marginal cost would not cover the fixed costs of the PD division and no profit could be earned. Perhaps instead either full cost-plus (using a standard full cost) or a two-part tariff system should be used instead. A two-part tariff system could include an element of profit to give PD the necessary motivation to supply, as well as control its own cost e.g. a fixed fee to cover fixed cost and some profit, and a charge for each unit supplied to cover the marginal cost of production. D1 – 6 G group (CIMA P1 May 2007) A transfer price must be selected by group however it must recognise the value expected by both the buying and selling divisions. The price set should encourage internal transfer between divisions as this would serve in the best interests of the group as a whole and not just for themselves. The recognition of a satisfactory transfer price will also allow for a better understanding of performance of each division by head office.

254 | P a g e

If there is an external market price for a product that could be transferred internally the transfer price should be set at this price less any savings made by transferring internally. This would mean that there would be an incentive by both divisions to transfer internally. If there is a limitation on external demand then the transfer price should be set at the marginal cost needed to manufacture the product, but a bonus incentive is needed to manufacture and transfer. A two-part tariff system could include an element of profit to give the necessary motivation to supply, as well as control costs e.g. a fixed fee to cover fixed cost and some profit, and a charge for each unit supplied to cover the marginal cost of production. Participation of the divisions in the creation of the bonus scheme means that it would be more likely to be accepted and there would be less conflict because they had accepted and developed the scheme themselves. The scheme should be clear and simple to understand by all staff and should motivate and reward divisions on a responsibility accounting basis. This means that they should be assessed on those costs and revenues that they can only control. D1 – 7 Digital equipment (CIMA P1 May 2008) Part (a) To measure technology leadership we could compare the number of new products launched each year versus the expenditure on research and development. This will tell us how effective innovative ideas developed by the company have been by looking at whether or not they have gone into commercial production for sale to customers. Part (b) The data that should be collected would include customer ratings and feedback logged when support is sought by customers. This could be done on the phone after support has been given or on the internet if support is web-based. This would help determine how satisfactory support given to customers is. Part (c) Benefits of internal benchmarking over external benchmarking (please note just three are needed):  Internal benchmarking information has been created by the company and not a third party like external benchmarking information, therefore internal benchmarking information may not be more accurate and timely than external benchmarking information.

255 | P a g e

 Internal benchmarking information has specifically created for the company unlike external benchmarking information, and so comparisons made with internal benchmarking information are more relevant.  External information maybe incomplete as it is difficult to get information about all competitors but internal information is complete as it is generate by the company and there are no restrictions to access.  Internal information will allow continuous improvement by sharing ideas between managers this is not possible with mangers of other companies. Part (d) Reasons why ROI may not be a good performance measure (please note just three are needed): 

The biggest drawback of ROI is that it is an accounting and not a cash-based measure, which it can be manipulated. The choice of accounting policy (stock valuation or depreciation) can also distort and create different profit and fixed asset levels within divisions.



ROI creates an incentive not to invest in the company’s long-term future and creates short-termist behaviour by divisional managers. Under investment in fixed assets causes the net book value of assets to decrease over time. ROI improves over the life of an asset where little or no reinvestment takes place.



If profits remain static then ROI will improve, yet the manager may have done very little in terms of improving results.



Managers may also be over zealous to cut back expenditure in order to improve the profit of the division e.g. advertising or training of staff, and this can jeopardise the longer-term profit of the business.



ROI may create political arguments over such costs as head office apportioned overhead or interest charges by head office which have direct negative impact on ROI.



Goal incongruent decisions maybe taken for example where an asset generates a positive net present value but would fail on the criteria of ROI used by the manager, and hence the project is rejected.

256 | P a g e

Solutions – Section B Part D Control and Performance Measurement of Responsibilty Centres D2 -1 Y and Z (CIMA P1 Nov 2005) Part (a) Tip: Total assets less current liabilities (TALCL) or ‘net assets’ is also equal to shareholders equity + long-term liabilities. It is imperative to read the question; the operating statements are for a single month (October) therefore profits before tax must be annualised in order for return on investment to be calculated. Return on investment (ROI) or Return on capital employed (ROCE) ROI =

Operating profit before interest and tax Capital employed (TALCL)

Division Y 15.0% =

(£0.122 million x 12 months) (£9.76 million net assets)

Division Z 20.0% =

(£0.021 million x 12 months) (£1.26 million net assets)

Discussion of relative performance    

Division Z has the highest return on investment (20%) in comparison to division Y (15%). Both divisions exceed the target of 12% per annum set by the parent company. However division Y will be at greater risk if the target return on investment is increased. Both are profitable and generate a positive contribution for the group. In absolute terms division Y is the largest division in terms of net assets and generates a greater absolute profit than division Z (£122,000 compared to £21,000 per month). This is almost six times the level of absolute profit in comparison to division Z.

257 | P a g e







Both divisions operate in similar markets however division Z has almost the same absolute level of variable cost as division Y, even though its sales revenue is almost half the amount. Division Y has variable cost to sales of 38.3% (£0.345m ÷ £0.9m) and division Z 56.2% (£0.312m ÷ £0.555m). This indicates that division Y looks more operationally efficient. Division Z has a much lower net assets value than division Y which could indicate that its assets are older and therefore more inefficient. Division Y has a greater level of apportioned central cost (£338,000 per month), which is almost twice the amount that division Z is charged. This arbitrary amount charged will effect the profitability of the two divisions by a great extent e.g. for division Z an 11.7% increase in apportioned central cost would reduce profit per month to zero (£21,000 ÷ £180,000). If the uncontrollability principle is applied and central apportioned cost were to be removed then the ROI of the two divisions would be as follows  Division Y (£0.46m x 12) ÷ 9.76m = 56.6%  Division Z (£0.201m x 12) ÷ 1.26m = 191.1%

More information will be needed for how central apportioned costs are allocated to each division, as well as information on the age of the net assets used within each division, to make a more effective comparison of financial performance between the two. Part (b) Tip: Residual income (RI) is calculated by taking the profit a manager earns for a division less a ‘notional interest charge’ for the investment within the division e.g. the profit generated from the division less a finance charge from the holding company or head office using a cost of capital. Accounting profit is calculated the same way as for return on investment (ROI). Profit before interest and tax Capital employed x cost of capital Residual income

X (X) X

Residual income in comparison to return on investment  

Absolute rather than a relative measure Deducts a finance charge for the cost of capital used for a division

Division Y Profit before interest and tax (£0.122 million x 12 months) Capital employed x cost of capital (£9.76 million net assets x 12%) Residual income

£m 1.464 (1.171) 0.293

258 | P a g e

Division Z Profit before interest and tax (£0.021 million x 12 months) Capital employed x cost of capital (£1.26 million net assets x 12%) Residual income   

£m 0.252 (0.151) 0.101

Even though division Y has a lower return on investment (15%) compared to division Z (20%), it does create greater wealth for the group in terms of the absolute size of residual income it earns. This is something that return on investment considered in isolation will not demonstrate because it is a relative not absolute measure of return. The implications of this information is that it demonstrates that division Y contributes greater wealth to the profits of the group and therefore its shareholders. It is a superior measure when contrasted to return on investment. However one single measure by itself will never allow a complete understanding of financial performance.

Part (c) Tip: The biggest drawbacks of both ROI and RI are that they are accounting not cashbased measures. Such financial measures can create short-term behaviour by divisional managers. If divisions under invest in non current assets, this causes the net book value of net assets or capital employed to fall in value over time. If profits remain static both ROI and RI will improve, yet the manager would have done little in terms of improving financial results. The choice of accounting policies used e.g. stock valuation, depreciation methods or the way central costs are apportioned, will also distort and create different profit and net asset levels within divisions. Managers may also be over zealous to cut back expenditure in order to improve the profit of the division e.g. advertising, training, research and development, and this can jeopardise the long-term profit of the division. These methods also can frustrate managers and can cause political argument occur over the allocation of cost centrally such as central apportioned overhead or interest charges. The below gives a comprehensive listing of the advantages and disadvantages of using the two methods, however one mark, up to a maximum of 3 would have been awarded for each brief comment you make. The level of comprehension within the solution below would therefore not be needed.

259 | P a g e

Advantages of ROI  A relative measure so different sized divisions can be compared better than RI when assessing financial performance.  Well understood by users of accounts.  Forces the manager to be efficient with resources (assets) used. Disadvantages of ROI Disincentive to invest in net assets to improve ROI. ROI improves over the life of an asset where little or no reinvestment takes place. Goal incongruent decisions where a new investment generates a positive net present value, but would fail on the criteria of ROI used by the manager. This is because new projects often will have low profit and high net book values in the early years of investment. An accounting not cash based measure therefore ROI can be distorted. Advantages of RI  Consistent with profit maximisation and an absolute rather than relative measure.  Brings home the idea about cost of finance for a manager.  Unlikely when contrasted with ROI to act as a disincentive to invest e.g. as long as profit is earned it should improve RI. Disadvantages of RI An accounting not cash based measure therefore RI can be distorted. Cannot compare divisions of different sizes very well. May discourage investment in net assets in order to lower the interest or finance charge applied to a division. RI improves the older net assets become e.g. a lower finance charge when applied to the historical cost of assets within the division.

260 | P a g e

Tip: Only a few sentences about two further methods of assessment would be required for one mark each. Possibilities could include the following.     

Controllability principle applied when calculating ROI or RI e.g. ignoring central costs apportioned. Cash-based methods such as throughput accounting, net present value or economic value added. Variance analysis and budgetary control through exception reporting. Ratio analysis e.g. profitability, liquidity and investor ratios. Other non-financial ratios e.g. sales per square metre, number of complaints, staff turnover, market share, sales growth, new customers or repeat business. Using performance frameworks like the balanced scorecard.

Other methods of appraising divisions Economic value added is an absolute cash based measure of the economic financial wealth generated by a division over time. It deducts a finance charge using a cost of capital, applied to the replacement cost of assets used by a division. They method concentrates on the maximisation of cash or contribution which is more likely to maximise shareholder value. EVA can not be manipulated by a manager’s choice over the accounting policies they might use. A new approach to strategic management was developed in the early 1990's by Drs. Robert Kaplan (Harvard Business School) and David Norton. The balanced scorecard suggests that we view an organisation from four perspectives.    

Customer perspective Internal perspective Innovation and learning perspective Financial perspective

261 | P a g e

D2 – 2 FP (CIMA P1 May 2006) Part (a)

Tip: A good approach here would be to work out a standard cost for the service department for each repair, breaking down the standard cost into variable (marginal) and fixed cost. This will help understand the principles of how servicing is being charged by the service department and also make the profit calculations easier.

Standard cost of one repair Parts Labour (3 x £15 per hour) Variable overhead (3 x £10 per hour) Variable (marginal) cost Fixed overhead (3 x £22) Full cost per repair 40% mark up on full cost Standard price

£ 54.00 45.00 30.00 129.00 66.00 195.00 78.00 273.00

Repairs carried out by the service department (at full cost plus 40%) Sales Service FP Department Department Group £ £ £ Sales 120,000 W1 136,500 W3 120,000 W5 Cost of sales 136,500 W2 97,500 W4 97,500 W6 Profit -16,500 39,000 22,500 W1 2000 x £60 W2 500 x £273 W3 500 x £273 W4 500 x £195 W5 2000 x £60 W6 500 x £195

262 | P a g e

Repairs carried out by the service department (at marginal cost) Sales Service Department Department £ £ Sales 120,000 W1 64,500 W3 Cost of sales 64,500 W2 97,500 W4 Profit 55,500 -33,000 W1 2000 x £60 W2 500 x £129 W3 500 x £129 W4 500 x £195 W5 2000 x £60 W6 500 x £195

FP Group £ 120,000 W5 97,500 W6 22,500

Repairs carried out by RS

Sales Cost of sales Profit W1 2000 x £60 W2 500 x £180 W3 500 x nil W4 500 x £66 F/OH per unit W5 2000 x £60 W6 (500 x £180) + £33,000 F/OH

Sales Service Department Department £ £ 120,000 W1 0 W3 90,000 W2 33,000 W4 30,000 -33,000

FP Group £ 120,000 W5 123,000 W6 -3,000

Part (b) (i) The transfer price for repairs undertaken by the service department on behalf of the sales department, will determine significantly the profits of the sales department. If full cost plus a mark up is being used, it is possible that the service department may not control its cost and pass on any inefficiency, by charging higher prices to the sales department. In effect the more inefficient the service department becomes, the higher the cost and therefore price it would charge to the sales department. The sales department paying for these inefficiencies by reduced profit, due to the higher price.

263 | P a g e

Another problem of full cost plus is that the price charged maybe too high, ignoring competition in terms of how much other companies would charge for the same service. The price charged by the service department is currently £273 per repair to the sales department. Compared with what RS would charge this would be £180 per repair. If these departments are to run as profit centres, the sales department may go elsewhere to improve its own profitability. This would actually be a goal incongruent decision because it would lower overall profitability for the group. Fixed cost would be incurred regardless by the service department, whether it supplied the service department or not. The actual cost to the group in relevant costing terms would be the internal marginal cost of each repair, currently budgeted at £129. If the sales department started to use RS the external marginal cost to the group would be £180, therefore the group profit would fall by (500 repairs x (£180 - £129) = £25,500. The group would therefore be worse off. Part (b) (ii)

Tip: One mark will be available for each relevant issue you discuss. Six possible solutions have been provided below.      

Does RS have a solid track record for delivery e.g. references from other satisfied customers in terms of the quality of service they provide should be sought. The financially stability of RS should be investigated e.g. FP should obtain previous sets of accounts and obtain credit ratings. Does RS have the resources required to work with FP e.g. will they cope with peak periods when demand for there services may be high. Will RS be increasing prices above £180 per repair in the long-term? Is any of the fixed overhead currently committed by the service department avoidable, if there is an internal reduction in demand for there services. Can the service department find external work to fill any spare capacity caused by a decline in internal demand?

It will be important to have a good service level agreement (SLA) with RS that gives a minimum level of service expected and offers rescission of the contract, should certain conditions be broken by RS e.g. for non-performance or a low standard of service over a period of time.

264 | P a g e

Part (c)

Tip: A profit centre is when a manager will be accountable for both creating revenue and controlling costs. The sales and service departments will therefore be assessed on profit earned. One mark will be available for each relevant issue you discuss. Many possible solutions have been provided below, it is important you give both advantages and disadvantages not just one or the other.

Advantages of profit centres  Linking financial reward to the achievement of profit targets set for staff will be more motivating for the department managers, encouraging them to increase sales for their departments and be more efficient in controlling costs.  A quicker response to changes in the market environment by departments due to having greater autonomy to make decisions. Greater decentralisation of decision making will also give greater responsibility to each manager improving their job satisfaction.  Due to less centralised decisions it should create a culture of greater entrepreneurial spirit and initiative from managers within departments.  Less bureaucracy and centralised interference by senior management should provide greater efficiency of decision making by managers in each department.  Senior management can free up time consuming affairs of running each department and concentrate on the corporate rather than business strategy of each department. Disadvantages of profit centres Each department maybe run from the individual managers view rather than the corporate point of view. As an example the sales department may choose RS as a supplier over the service department leading to dysfunctional behaviour e.g. loss of group profit. Because of less senior involvement there maybe less control and coordination of activities within each department. Profit centres may duplicate many activities such as human resource management, finance or IT, placing less reliance on centralised activities being offered. This could increase fixed overhead for the group as a whole. For centralised services there could be a central overhead charge by senior management for these services which often creates political argument and may frustrate managers if these charges effect their own profit. Because of more delegation to middle management this may add further fixed overhead of running FP due to greater supervision required. It may also create less efficiency and consistency of decisions within departments.

265 | P a g e

The department managers may have little experience previously of working autonomously; this could lead to poor decisions being made. Training may be required in order for them to participate effectively. D2 – 3 ZZ group (CIMA P1 Nov 2006) Part (a) Income statements Sales (W1) Cost of sales Variable costs (W2) Contribution Fixed costs Profit Less finance cost (W3) Residual income ROCE (W4) Operating profit margin (W5) Asset turnover (W6) Working 1

Division X £ 100,000

Division Y £ 270,000

(50,000) 50,000 (15,000) 35,000 (6,000) 29,000 58.33% 35.00% 1.67

(144,000) 126,000 (100,000) 26,000 (11,000) 15,000 23.64% 9.63% 2.46

Sales

Division X would sell a the external market price of £20 per unit Currently 3,000 units transferred to division Y and 2,000 units sold externally. This gives the current sales of {(3,000 x £10) + (2,000 x £20)} £70,000 Revised sales would be the entire 5,000 units are sold at £20 = £100,000 Division Y sales would remain unchanged as they don’t transfer internally. Working 2

Variable costs

Division X – remain the same at £50,000 (£10 per unit marginal cost) Division Y – 3,000 units transferred from Division X would now cost £20 per unit. Current variable costs Less 3,000 x £10 Add 3,000 x £20 Revised variable costs

£114,000 (£30,000) (£60,000) £144,000

266 | P a g e

Working 3

Finance charge

Capital employed x cost of capital Division X - £60,000 x 10% Division Y - £110,000 x 10% Working 4

= £ 6,000 = £11,000

Return on capital employed (ROCE)

ROCE = Profit / capital employed

x 100%

Division X - £35,000 / £60,000 x 100% Division Y - £26,000 / £110,000 x 100% Working 5

= =

58.33% 23.64%

= =

35.00% 9.63%

Operating profit margin

Profit / turnover x 100% Division X - £35,000 / £100,000 x 100% Division Y - £26,000 / £270,000 x 100% Working 6

Asset turnover

Sales / capital employed Division X - £100,000 / £60,000 Division Y - £270,000 / £110,000

= =

1.67 2.46

Part (b) Performance measures Residual income ROCE Operating profit margin Asset turnover

Division X before (£1,000) 8.33% 7.14% 1.17

Division X after £29,000 58.33% 35.00% 1.67

Division Y before £45,000 50.91% 20.74% 2.46

Division Y after £15,000 23.64% 9.63% 2.46

The proposed change in policy will benefit Division X greatly and but at the expense of Division Y. Division X’s revenue and therefore profit increases by £30,000. This is because they are now selling 3,000 units at £10 extra (£30,000). This therefore increases their residual income to a positive £29,000. Their return on capital has increased hugely from 8.33% to 58.33%. The operating profit margin has also increased to 35% from 7.14% and their asset turnover is much improved.

267 | P a g e

However Division Y’s performance is not so good. With the increased cost of component C, their variable costs have increased to £144,000 from £114,000. This is the £30,000 increase which has been passed on from Division X. The result of these increased costs has resulted in lower residual income (only £15,000), the ROCE is more than half its original value at 23.64%. The profit margin has reduced from a healthy 20.74% to a depressing 9.63%. Their asset turnover remains the same. The result of altering the transfer price to £20 per component will be great for Division X as their performance measures will be greatly improved. Therefore the managers of Division X will really want to push for this new proposal. However for Division Y, their performance measures will be vastly reduced, resulting in lower moral. Therefore Division Y may choose to source their component C from elsewhere at cheaper rates. This will lead to goal incongruence which is not in the best interest of the group. Part (c) (i) With the new proposal, the managers of Division X will want to set the transfer price at the same rate as the external market price of £20 per unit. This will improve their financial performance immensely. Division Y will lose out if the transfer price is set at £20 as their performance measures deteriorate drastically. So therefore division Y managers will want to negotiate a lower transfer price. If Division X does not agree to a lower price, Division Y may purchase component C externally. The marginal cost to the group of producing component C is £10 and if Division Y purchases externally at a price higher than £10, the group as a whole is losing out. If Division Y does source component C externally, Division X will have spare capacity. Currently there are only 3,000 units of external demand, which means that there will be 2,000 units of spare capacity. If the fixed costs cannot be avoided, this again means that ZZ group as a whole is losing out and it will impact the bottom line profit. A good transfer price is one where both divisions are happy with and it doesn’t impact the group as a whole in a negative way. This usually means that divisions buy and sell internally and do not source goods from outside the group if they can buy them internally. With the current situation it is unlikely that both divisions can agree on a suitable transfer price. This may cause hostility between both divisions leading to goal incongruence and low morale. The group may have to intervene to ensure that profitability of the group as a whole is not negatively impacted.

268 | P a g e

Part (c) (ii) A good way of pleasing both divisions where there is a problem of a suitable transfer price could be methods such as a dual pricing or two-part tariff system. These methods of transfer pricing ensures both divisions are happy and that they buy and sell to each other. A dual transfer price is achieved by setting one transfer price for Division X and another transfer price for Division Y. The transfer price for Division X to sell will be set at the external market rate and the transfer price for Division Y will be set at the marginal cost of producing component C. The difference between the two transfer prices would need to be reconciled by head office, which is one of the major drawbacks of this method as it is very time consuming. A two-part tariff system is where a fixed charge per period is given to the seller – Division X - irrespective of the amount of units transferred by the seller plus a fixed rate (at marginal or variable cost) charged for each unit transferred. Such a system would include an element of profit to give Division X the necessary motivation. Such a system aims to ensure the seller covers the fixed cost of production, and receives a selling price for each unit supplied to cover the variable or marginal cost of production. Both of these transfer pricing policies would give autonomy to Division X and Y. However agreeing a transfer price can be very time consuming especially if the divisional managers are not experienced in this area. Some involvement by the group may be necessary to ensure that negotiations go ahead and that both divisions do agree. D2 – 4 Computer manufacturer (CIMA P1 Nov 2007) Part (a) 

 

The European subsidiary has offered price $95 to supply the chips to American subsidiary and it would earn the same contribution (see last paragraph). Therefore the contribution earned from this would be the same as to external sales ($105 - $60) $45. The spare capacity will be used resulting in higher fixed costs. Assume all the American chips are delivered (300,000) and the rest sold to externally.

Increase in European contribution Increase in fixed costs Increase in European profits

200,000 x $45 $26m - $20m

$9 million ($6 million) $3 million

Extra cost to American subsidiary

300,000 x ($95 - $90)

($1.5 million)

Net increase in profit on group

($3m - $1.5m)

1.5 million 269 | P a g e

Part (b) (i) Assume all orders to American fulfilled first 300,000 units and remaining go to external sales 500,000 units. This is a loss of 100,000 units sold externally. Current contribution from external sales (105-60) per unit Lost contribution due to sales internally – 100,000 units x $45 Additional fixed costs Total additional contribution required ($4.5m + $6m) Contribution required per unit sold to America ($10.5m / 300,000 units) Variable cost of selling a unit to America ($95 - $45) Transfer price

$45 $4.5 million $6 million $10.5 million $35 $50 $85

Part (b) (ii) The return on assets = profit / annual fixed costs x 100% This is currently 35% Profit / $26m x 100% = 35% Profit = $26m x 35/100 = $9.1 million Therefore a profit of $9.1 million needs to be generated. Target profit Add back fixed costs Total contribution ($9.1m + $26m) Contribution from external sales (500,000 units x $45) Contribution required from American sales (35.1 – 22.5) Contribution per chip to American ($12.6m / 300,000) Variable cost of selling a unit to America ($95 - $45) Transfer price

$9.1 million $26 million $35.1 million $22.5 million $12.6 million $42 $50 $92

Part (c)

REPORT To: From: Date: Subject:

Managing Director Accountant November 2007 Transfer pricing and performance measures

This report will discuss issues raised by the directive and the introduction of the performance measures. 270 | P a g e

Introduction The American subsidiary currently sources the chips externally paying $90 per chip for the requirement of 300,000 chips. The European subsidiary has spare capacity of 200,000 chips and here there is an opportunity to source the chips internally rather than sourcing from external suppliers. The group’s profit increases by $1.5 million if the European supply chips to America for the price of $95. This is a significant increase which will benefit the organisation. Transfer Pricing A good transfer price is one where both divisions are happy with and it doesn’t impact the group as a whole in a negative way. This usually means that divisions buy and sell internally and do not source goods from outside the group if they can buy them internally. The issue of supplying chips internally is what price to transfer them at. Here the needs of the both organisations require careful consideration as not to de-motivate the managers. The American subsidiary is currently buying externally at $90 and this is effectively the maximum they will pay. So the European can only really charge a maximum transfer price of $90. The minimum price the European’s could charge is $85 {part b(i)}, so a range from $85 to $90 exists. However if the manager’s performances are going to be measured based on returns, then the minimum price the European will want to charge is $92 and this would clearly not be acceptable to the Americans. From the group’s perspective the transfer price is based on opportunity cost approach (or relevant costing) to ensure goal congruence Minimum price to a seller (the European subsidiary) • •

Marginal cost of each unit sold normally if seller is not at full capacity or no intermediate market exists Marginal cost + opportunity cost e.g. normally the external market price for the product sold if an intermediate market exists or the division is at full capacity

Maximum price for a buyer (the American subsidiary) Lower of • •

Market price the buyer could obtain the product from elsewhere Market price the buyer sells the product for, less any further costs the buyer has before resale of the goods

271 | P a g e

What the above method will do is to set a range; so long as it is profitable to do so for the group e.g. a range actually exists, any price within this range once set will then ensure goal congruence e.g. seller and buyer operate in the best interests of the group as a whole. The actual transfer price within this range would normally have to be found by politics and compromise between the two managers. Performance measures Setting performance measures based on profit will encourage goal congruence. The transfer price is $85 and this would be acceptable to both parties. This would also result in higher profits for the group. If the return on assets is used as a performance measure the transfer price will be too high for the American subsidiary to accept $92 and this will result in goal incongruence. This means chips will be sourced from outside the group resulting in lower profits. A single performance measure is not an ideal way to measure performance of divisions as this could result in short term decisions being taken which do not benefit in the long term. A range of performance measures with both financial and non financial aspects is better in the long term for the group. Conclusion The European subsidiary should use its spare capacity and sell chips to the American subsidiary at $85. However the loss of external sales of 100,000 units by the European could have a negative impact on future sales. Increasing capacity will help increase profits. A range of performance measures should be used to encourage goal congruence and long term success of the group. Part (d) Different countries have varying tax rates. A multi-national company can set up subsidiaries in different countries to take advantage of these tax rates through transfer pricing to reduce their overall tax charge. Where there are high tax rates in a country, goods can be supplied to that subsidiary at a high transfer price to reduce profits, from a subsidiary in a low tax rate country. The idea is to increase profits in lower tax rate country and decrease profits in a higher tax rate country. This can be done through transfer pricing.

272 | P a g e

National tax authorities thoroughly investigate accounts of these types of companies to ensure that a market price has been used and that the profits have not been manipulated by transfer pricing to reduce tax payments. Fines and penalties can be imposed on organisations who deliberately try to manipulate profits by the improper use of transfer pricing. D2 – 5 Perfumes and cosmetics (CIMA P2 May 2010) Part (a) (i) Return on capital employed (ROCE) =

Profit before interest and tax (PBIT) Capital employed

x 100%

ROCE is also referred to as return on investment (ROI) and return on net assets (RONA). ROCE measures profitability and shows how well the business is utilising its capital to generate profits. Process

Profit & loss $

Capital employed $

B

18,800

800,000 x 0.8 = 262,144

Monthly ROI %

Annualised ROI %

18,800 / 262,144 = 7

7 x 12 = 84

(15,550) / 320,000 = (5)

(5) x 12 = (60)

(5,000) / 32,212 = (15)

(15) x 12 = (180)

5 2

C

(15,550)

500,000 x 0.8 = 320,000 10

D

(5,000)

300,000 x 0.8 = 32,212

Part (a) (ii) Capital equipment bought in each division has been purchased at different points in time compared to other divisions; therefore it means that the ROI comparison between divisions will be inconsistent. This is due to the fact that the price paid will be affected by inflation and in addition depreciable years will be different between divisions. Some of the costs are not within the control of the divisional managers namely head office costs which are apportioned arbitrarily; therefore it would be unfair to measure performance based on these divisional profits. Transfer prices are being used between divisions which are a mark up on the total costs of the process. This is unfair as the receiving division has no control over the costs of the supplying division on which the transfer price calculated. The receiving division’s performance will be affected by the costs of the supplying division.

273 | P a g e

Division C and D have fixed costs occurring regardless of the volume received from division B, therefore the managers of C and D have little control over their activity and in addition they are highly geared and so dependent on high levels of activity in order to cover their fixed costs. Part (b) (i) If investment was undertaken: Extra income generated 1,500 litres $30,000 from abnormal losses at $20 per litre Abnormal sales forgone ($750) 1,500 litres x $0.50 Increase in depreciation (W1) ($11,333) $16,666 - $5,333 Loss in 2010 ($15,550) Profit $2,367 ROI = ($2,367 / $1,000,000) x 100% = 0.24% Annualised ROI = 0.24% x 12 = 2.88% If investment was not undertaken: Fall in depreciation (W1) $5,333 - $4,267 Loss in 2010 Loss

$1,066 ($15,550) ($14,484)

ROI = [($14,484) / $256,000] x 100% = (5.66%) Annualised ROI = (5.66%) x 12 = (67.89%) Workings (W1) – Depreciation Year

NBV b/f

2008 2009 2010 2011

$ 500,000 400,000 320,000 256,000

Deprecation (20% reducing balance) $ 100,000 80,000 64,000 51,200

NBV c/f $ 400,000 320,000 256,000

274 | P a g e

Monthly depreciation charge on old machine in 2010 = $64,000 / 12 = $5,333 Monthly depreciation charge on old machine in 2011 = $51,200 / 12 = $4,267 Monthly depreciation charge on new machine in 2011 = ($1,000,000 x 0.2) / 12 = $16,666 Part (b) (ii) This investment decision shows a small increase in ROI of 2.88% per annum and should be accepted if the increase is adequate. There maybe objections from division C as the manager may feel that this is not enough and there is a good chance it may cause a fall in ROI if the figures used are not accurate. The choice should be based on using NPV based on the company’s cost of capital. If it is positive then the investment should be accepted. Using ROI the profits can be manipulated because it includes some non-cash based items such as depreciation. For example the depreciation rate could be decreased to increase the profit and thereby increasing the ROI. NPV uses relevant costing and so only includes those cash flows that will change as a result of the investment decision and so there is less scope for manipulation. ROI ignores the time value of money as future cash flows are worth less today and that they should be discounted to reflect this. NPV takes account of the time value of money as it applies discounting to future cash flows to give them same real value if they were received today. Part (c) Manager of division B The manager of B would be in favour of the current transfer pricing policy as all of his output is transferred to C or D and furthermore a by-product is created (due to this process) which is sold externally. All of his costs are covered and receives a profit through the transfer price which is budgeted cost plus 15% mark up. The problem with this is that there is no incentive for manager B to control any inefficiencies in his process as he knows that all costs will be paid by the transfer pricing policy, furthermore, if there are any efficiencies made he will be rewarded for it. Manager of division D The manager of D can buy the material D externally for $7.50 per litre or pay $9.20 per litre internally from B. It is easy to see that the manger of D is going to purchase externally as it is cheaper than buying from B, and so the manger of D is not in favour of the current transfer pricing policy.

275 | P a g e

Group perspective It is important that divisions transfer material between them as far as possible otherwise there maybe materials that the group make which are not used internally but instead the same items are being bought externally at a high cost, leading extra unnecessary costs. Clearly the manager of D needs to be encouraged not to make the sub-optimal decision of purchasing material externally when it is being made by division B. Furthermore, the material D being produced by B is part of a joint process and so in any event will be made and so if it is not transferred to division D then it will be wasted. There would be no change to the costs of process B and external costs spent by the group will rise. The transfer price between division B and division D should at least match the market price of $7.50 and so division D would be encouraged to accept the transfers from division B. One way to do this is perhaps by reducing the costs being apportioned to the manufacture of material B and so therefore division B can charge a lower price to division D. A dual pricing approach can be adopted to help in this situation, as this would mean we give the transfer price to each of the divisions that would want in order to agree to transfer. For example we allow division B to transfer at $9.50 but give a transfer price of $7.50 to division D. This would encourage the transfer and the difference in the prices can be resolved though head office re-apportioning it as charges to all divisions. D2 – 6 SWZ (CIMA P2 Nov 2010) Part (a)

Return on capital employed (ROCE) =

Profit before interest and tax (PBIT) Capital employed

x 100%

The ROI for the three years: Year 2008 2009 2010

ROI $ 40 / 400 = 10% 56 / 320 = 17.5% 62 / 256 = 24.2%

276 | P a g e

There is a very good improvement of the ROI over the last 3 years from 10% up to nearly 25%. Inflation has been removed according to the question and the figures the same for turnover and cost of sales throughout the 3 years which means there has been no increase in products sold over the last 3 years. The gross profit has been constant at 40% and therefore indicating no change in quantity sold and prices. Operating costs appear to be falling over the last three years, but if depreciation is removed it shows that operation costs have increased by $4,000 in 2009 and more significantly in 2010. In conclusion the improvement in ROI over the last three years is largely down to the depreciation policy rather than improved performance by the division. Part (b) The investment results in a positive NPV of $24,536 and so from a group company perspective it will be accepted by head office to go ahead, however from the perspective of the manger of S division it will depend on whether or not ROI of the division is improved. This is because his performance is assessed on ROI. To assess this we will look at the divisional ROI if the investment is undertaken compared to the divisional ROI if the investment is not undertaken. If investment was undertaken: Sales Cost of sales ($240,000 x 90%) Gross profit Other operating costs (W1) Pre-tax operating profit Capital invested at the end of the year (W2) = $252,800

$ 400,000 216,000 184,000 (97,200) 86,800

ROI = ($86,800 / $252,800) x 100% = 34.3% If investment was not undertaken: Sales Cost of sales Gross profit Other operating costs (W3) Pre-tax operating profit

$ 400,000 240,000 160,000 (85,200) 74,800

Capital invested at the end of the year ($256,000 - $51,200) = $204,800 ROI = ($74,800 / $204,800) x 100% = 36.5%

277 | P a g e

Workings (W1) – Other operating costs Other operating costs Less: Depreciation ($320,000 x 20%) Add: New depreciation (W2) Total

$ 98,000 (64,000) 63,200 97,200

(W2) – New depreciation and capital invested at the end of the year $ Current total NBV at the end of 2010 256,000 Less: NBV of machine sold (40,000) Add: Cost of new machine 100,000 316,000 Less: New depreciation (316,000 x 20%) (63,200) Revised total NBV at the end of 2011 252,800 (W3) – Other operating costs Other operating costs Less: Old depreciation ($320,000 x 20%) Add: New depreciation ($256,000 x 20%) Total

$ 98,000 (64,000) 51,200 85,200

Part (c)

Profit before interest and tax (PBIT) Capital employed x head office % interest charge Residual Income

If investment was undertaken: Pre-tax operating profits Notional capital charge ($252,800 x 8%) Residual Income

$ 86,800 (20,220) 66,580

If investment was not undertaken: Pre-tax operating profits Notional capital charge ($204,800 x 8%) Residual Income

$ 74,800 (16,380) 58,420

X (X) X

Difference is an increase of Residual Income if investment undertaken of $8,160

278 | P a g e

D2 – 7 DE company (CIMA P2 May 2011) Part (a) Division E could have sold a further 42,000 components but was constrained by capacity and had to transfer 70,000 components internally to division D. Division E therefore lost out on achieving the market price for 42,000 components, and therefore the transfer price on these to division D should be market price. The rest of the components should be at variable cost. Analysis

Number of components Variable cost Sales value

Internal at variable cost 28,000

Internal at market price 42,000

External

Total

70,000

140,000

$000 28,000 28,000

$000 42,000 65,100

$000 70,000 108,500

$000 140,000 201,600

Part (b) External demand is very important as it determines what profits are accrued to divisions D and E internally. Division E has sold 28,000 components at variable cost to division D. It could have sold them externally at $1550 and could have made a profit instead of making no profits through the internal transfer. If division D had bought externally rather than internally then it would have had to spend $43.4m to obtain the 28,000 components transferred to them at variable costs. This is an extra $15.4m. Division D appears to enjoy higher profits because of the items transferred at variable cost, which may not be fair as division E is receiving no recognition for the manufacture of these items. There should be some benefit accrued to division E and maybe not full market price as it cannot sell these components externally. For example if the profits were shared equally between the divisions each division would have a profit of $7.7m. If the market demand increased for components then the transfer price charged to division D would increase and thereby shifting more of the profits to division E. If market demand fell for the components then the transfer priced charged to division D would fall and thereby shifting more of the profits to division D. Part (c) (i) E’s capacity will increase by 10% and variable costs will fall by 20% if the new investment is undertaken, however this benefit is reduced because of the internal transfer policy.

279 | P a g e

Any increase in capacity for division E will have the effect of increasing external sales but at the same time reducing the volume of forgone external sales when transferring components to division D. Therefore there is no additional financial benefit for division E. Half of E’s components are sold to D and 28,000 of these are transferred at variable costs. Cost savings are therefore transferred to D because of the transfer pricing policy. E will only enjoy cost savings on those items which are sold at market value. Division E benefits: VC of items sold at market value = $140m x 80% = $112m per annum 20% cost savings every year = $112m x 20% = $22.4m PV of cost savings over 5 years at 8% cost of capital = $22.4m x 3.993 = $89.4m The cost of the investment is $120m with no residual value and the cost saving from investment is going to only be $89.4m, therefore it is not financially viable from division E’s perspective. Part (c) (ii) We need to compare the original situation to the revised situation from a group perspective. We already know the benefits that would accrue to division E if the investment was undertaken (see part (i) above) and therefore we now need to do the same for division D. Revised transfer value for division D: 42,000 components original cost of $42m x 80% 28,000 components market value of $1,550 per unit Total

= =

$33.6m $43.4m $77.0m

Original transfer value for division D = $28m + $65.1m = $93.1m Saving = $93.1 - $77.0m = $16.1m Total saving = $22.4m + $16.1m = $38.5m PV = $38.5 x 3.993 = $153.73m From a group perspective the investment of $120m is beneficial because the PV of $153.73m is greater than this cost.

280 | P a g e

Part (d) It should be fair and consider those items that can be controlled by the manager. It should simple to understand and easy to calculate so the impact of decisions taken by managers can be seen and measured.

281 | P a g e

View more...

Comments

Copyright ©2017 KUPDF Inc.
SUPPORT KUPDF